SlideShare a Scribd company logo
1 of 361
Primer Simulador
del Curso ENARM 2013
    CMN SIGLO XXI
Instrucciones
1. Preparar hoja de respuesta impresa.
2. Activar el simulador aplicando play.
3. Escribir nombre y hora de inicio en la hoja de
   respuesta.
4. Circular el reactivo de cada respuesta en hoja
   de respuesta.
5. El simulador se activará a las 8:00 hrs y se
   desactivará a las 12:00 hrs, la primera parte.
6. La segunda parte se activara a las 14:00 hrs y
   de desactivará a las 18:00 hrs.
Caso clínico No. 58 (ejemplo)
Femenino de 51 años, originaria de Campeche,
  acude control mensual por HAS y DM 10 y 5 años
  respectivamente, sus SV fueron TA 155/95
  mmHg, FC 51 lpm, FR 21 rpm. Glucosa 210 mg/dl,
  hemoglobina glucosilada 6, urea 17, creatinina
  1,1 mg/dl, refiere mareo ocasional, cefalea global
  acompañada de sueño y fatiga, frio y mareo
  ocasional, disnea de leves a moderados
  esfuerzos, se observa edema de miembros
  inferiores. Su tratamiento es captopril,
  metoprolol y glibenclamida, se realiza ECG (ver
  imagen).
Caso clínico No. 58 (ejemplo)
Caso clínico No. 58 (ejemplo)
PRENGUNTA
Se realiza ECG, considerando las manifestaciones que se
  han presentado, que conducta terapéutica considera
  más apropiada realizar de forma mediata?

RESPUESTA
a.- Incrementar los fármacos hasta meta de <120/80
   mmHg.
b.- Retirar metoprolol continuar con captopril y
   glibenclamida.
c.- Mantener glibenclamida, retirar metoprolol e iniciar
   losartan.
d.- Agregar hidroclorotiazida al manejo actual
Hoja de respuestas
d   51     a   b    c   d     101
d   52     a   b    c   d     102
d   53     a   b    c   d     103
d   54     a   b    c   d     104
d   55     a   b    c   d     105
d   56     a   b    c   d     106
d   57     a   b    c   d     107
d   58     a   b    c   d     108
d   59     a   b    c   d     109
d   60     a   b    c   d     110
Caso Clínico
• Femenino de 52 años, acude por cefalea occipital, que
  se incrementa con esfuerzos y estrés laboral, agrega
  mareo, intolerancia a ruidos e luz. AHF padre con IAM
  a los 40 años, madre con DM, HAS. APP tabaquismo,
  alcoholismo, vida sedentaria y obesidad grado II.
  AGO: eclampsia hace 30 años y anticonceptivos por
  15 años. EF: TA 145/109, 150/110, 145/105. FC 93
  lpm, FR 23. Palidez, ingurgitación yugular, ruidos
  respiratorios subcrepitantes, edema maleolar ++.
PREGUNTA No. 1
¿Cuál de los siguientes criterios es el más útil para
  para establecer el diagnostico?

RESPUESTA
a.- Cifras tensiónales altas en repetidas ocasiones.
b.- Sospecha de alteraciones de órgano blanco.
c.- Factores de riesgo presentes.
d.- Alteraciones de estudios paraclinicos.
PREGUNTA No. 2
Se realizaron estudios de laboratorio y gabinete con
los siguientes datos: glucosa 116, creatinina 1.6, acido
úrico 9, colesterol 256, triglicéridos 380, EGO:
proteinuria +, glucosuria +. Cual es su conducta
diagnostica mas adecuada a seguir?

RESPUESTA
a.- Realizar Rx de torax.
b.- Realizar ECG.
c.- Realizar ECCG.
d.- Prueba de esfuerzo.
PREGUNTA No. 3
Cual de los siguientes datos observados en la rx es
mas probable? (ver imagen)

RESPUESTA
a.- Cardiomegalia global.
b.- Hipertrofia ventricular derecha.
c.- Edema agudo pulmonar.
d.- Hipertrofia ventricular izquierda.
PREGUNTA No. 4
Cual de los siguientes datos observados en el
ECG es más probable? (ver imagen)

RESPUESTA
a.- Hipertrofia predominio septum.
b.- Hipertrofia ventricular izquierda.
c.- Hipertrofia ventricular derecha.
d.- Hipertrofia auricular izquierda.
PREGUNTA No. 5
Considerando el fondo de ojo cual es el estadio con
la clasificación de Keith - Wagener - Barkeren el que
se encuentra? (ver imagen)

RESPUESTA
a.- Grado I.
b.- Grado II.
c.- Grado III.
d.- Grado IV.
PREGUNTA No. 6
Considerando la clasificación JNC 7 en que estadio se
encuentra el paciente?

RESPUESTA
a.- Normal.
b.- Prehipertensión.
c.- Hipertensión estadio I.
d.- Hipertension estadio II.
PREGUNTA No. 7
• De los factor de riesgo presentes cual es el más
  importante para el desarrollo de HAS en este caso?

RESPUESTA
a.- Eclampsia previa.
b.- Uso de estrógenos.
c.- Familiar con IAM.
d.- Familiar con HAS.
PREGUNTA No. 8
Cuál de los siguientes esquemas eligiría como inicial
  presenta mayor utilidad para el paciente?

RESPUESTA
a.- Losartan y hidroclorotiazida.
b.- Metoprolol, hidroclorotiazida y nifedipina.
c.- Captopril, hidroclorotiazida y metoprolol.
d.- Lisonapril y amlodipino.
PREGUNTA No. 9
  Cuáles son los lineamientos para el manejo en tercer
  nivel de los pacientes con hipertensión arterial de
  difícil control en el primer nivel de atención, presente
  en este caso?

RESPUESTA
a.- Falta de recursos para continuar con el manejo.
b.- Datos sugestivos de daño a órgano blanco.
c.- Requerir más de 3 fármacos.
d.- Presencia de crisis hipertensivas.
PREGUNTA No. 10
¿Cuáles son los cambios bioquímicos es más probables
  por los datos clínicos en el caso descrito?

RESPUESTA
a.- Aumento de angiotensina II.
b.- Aumento de endotelinas.
c.- Aumento de noradrenalina.
d.- Disminución de tromboxano A2
PREGUNTA No. 11
De los siguientes cambios funcionales cual es
  predominantemente menos dependiente de los
  factores fisiopatogenicos?

RESPUESTA
a.- Hiperactividad simpática.
b.- Aumento del volumen sanguíneo.
c.- Alteración del FNA.
d.- Expansión del volumen intracelular.
PREGUNTA No. 12
Cuáles de las siguientes medidas es menos adecuada
  para garantizar el adecuado control, vigilancia y
  seguimiento en este paciente?

RESPUESTA
a.- Toma de presión arterial al menos 2 veces al mes.
b.- Envió oportuno a segundo nivel al diagnostico inicial.
c.- Evaluación oftalmológica para manejo de retinopatía.
d.- Realización de ecocardiograma al menos una vez al
   año.
PREGUNTA No. 13
Cuál de los resultados de los índices o clasificaciones
  siguientes es más firme para la referencia del
  paciente antes descrito a segundo nivel de atención?

RESPUESTA
a.- Clasificación de keith, Wagener y Barker.
b.- Índice de KDOQI.
c.- Índice de Sokolow-Lyon.
d.- ATP III.
PREGUNTA No. 14
Cuál de las siguientes complicaciones es mas probable
  esperar en el caso considerando sus manifestaciones
  clínicas, daño a órgano blanco y paraclinicos
  observados?

RESPUESTA
a.- Infarto agudo al miocardio.
b.- Accidente vascular cerebral.
c.- Insuficiencia cardiaca congestiva.
d.- Insuficiencia renal crónica.
PREGUNTA No. 15
2 años después la paciente continuo presentando
  evolución maligna de la retinopatia hipertensiva, su
  gasto urinario fue de 45 ml/hr, creatinina 6, Urea 85,
  Cuál de los siguientes cambios histológicos renales es
  menos probable observar?

RESPUESTA
a.- Dilatación de los podocitos.
b.- Esclerosis glomerular.
c.- Necrosis fibrinoide glomerular.
d.- Fibrosis intersticial
PREGUNTA No. 16
Cuáles son las siguientes alteraciones es menos
  frecuentes que se presentan de la retina en la
  hipertensión arterial?

RESPUESTA
a.- Papiledema del nervio óptico.
b.- Amaurosis.
c.- Manchas algodonosas.
d.- Hemorragias.
PREGUNTA No. 17
The pharmacodynamic effect what more useful for
  nephroprotection in treating hypertension?

ANSWER
a. - Reabsorption of K+.
b. - AGT inhibitors
c. - AGT II antagonism.
d. - Blocking calcium channels.
PREGUNTA No. 18
What is the most useful strategy for the rehabilitation
  of the current case nephrological disorders?

ANSWER
a. - Maintain strict blood pressure levels.
b. - Monitoring K/DOQI semester.
c. - Reduced weight, lipids and snuff.
d. - Decrease sympathetic nervous system activity.
PREGUNTA No. 19
Cuál es la estrategia predictiva más útil para seguir la
  evolución de las alteraciones oftalmológicas del caso
  actual?

RESPUESTA
a.- Mantener los niveles de tensión arterial estricto.
b.- Monitorización de K/DOQI semestral.
c.- Reducción de peso, lípidos y tabaco.
d.- Disminuir la actividad del sistema nervioso
   simpatico.
CASE REPORT

Male is 78 years old who attends to consultation by
 joint pain in his left knee during the scan you identify
 that your blood pressure are as follows, TA 3 times,
 145/90, 155/90 and 150/91 mmHg, the patient
 denied symptoms associated with tensional figure.
 The patient has a history of a transient ischemic
 episode 6 months ago.
PREGUNTA No. 20
What is action to take to handle this case?

ANSWER
a. - Weight reduction and salt.
b. - Hydralazine 25 mg every 24 hrs.
c. - Captopril and hydralazine.
d. - Enalapril and hydrochlorothiazide.
PREGUNTA No. 21
What is the purpose of the BMI for the treatment of the
  patient?

ANSWER
a. - Keep the body index between 20 and 25 kg / m2.
b. - Maintain systolic blood pressure at least 140 mmHg.
c. - Keep diastolic blood pressure less than 90 mmHg?.
d. - Reduce the risk of MI or stroke.
PREGUNTA No. 22
What is the treatment of choice for providing
  protection of the patient considering the history of
  transient cerebral ischemia?

ANSWER
a. - Enalapril 10 to 20 mg.
b. - Losartan 50 mg
c. - Hydrochlorothiazide 12.5 to 25mg.
d. - Amlodipine 5 to 10 mg.
PREGUNTA No. 23
The patient was evaluated by a cardiologist and
  returned for continuous control, within the measures
  submitted by the same monitor blood pressure was
  supine, sitting and standing. What is the reason for
  this measure.

ANSWER
a. - A measure that guarantees the figures.
b. - Search orthostatic symptoms.
c. - Check the treatment effect.
d. - Does not have a demonstrable effect.
CLINIC CASE
A 47 year old white female with diabetes mellitus type II
  presents to the clinic with an erythematous rash in her
  axillae and groin of one month duration. Over the
  counter cortisone has failed to clear her rash. She
  denies using any deodorants or creams.

PREGUNTA No. 24
What is the best treatment option for this diagnosis?

ANSWER
a.- Erythromcin.
b.- Trimethoprim-sulfamethoxazole.
c.- Ketoconazole cream.
d.- Ciprofloxacin.
CASO CLINICO
Neonato de 11 días de vida, previamente sana,
  ingresada por dificultad respiratoria, mala
  coloración, rechazo de tomas y decaimiento.
  Constantes vitales; TA de 95/65 mmHg, FR de
  160 lpm, FR de 90 rpm y saturación de
  oxígeno del 94% con FiO2 del 30%. Se
  auscultan ruidos crepitantes e hipoventilación
  basal izquierda. La radiografía de tórax
  evidencia infiltrado basal izquierdo.
PREGUNTA No. 25
Cual es la conducta a seguir?

RESPUESTA
a.- Amoxicilina mas gentamicina.
b.- Trimetoprim mas sulfametoxazol.
c.- Cetriaxona.
d.- Vancomicina.
CASO CLINICO

RN de 36 SDG obtenido por cesarea por
  desprendimiento de placenta. Al nacimiento
  presenta Apgar 8/9, peso en -0,89 DE, longitud en
  +1,75 DE, sin datos del perímetro cefálico. Con 48
  horas de vida ingresada por ictericia en su hospital
  de referencia presenta hipoglucemias no cetósicas
  (glucemias entre 25-45mg/dl), insulinemia máxima
  registrada: 7μU/ml coincidiendo con glucemia de
  33mg/dl.
PREGUNTA No. 26
Cual es la conducta a seguir para identificar la causa ó
  antecedentes de la hipoglucemia en el paciente?

RESPUESTA
a.- Verificar Diabetes Mellitus en la madre.
b.- Realizar escaneo abdominal.
c.- Verificar funcionamiento hepático.
d.- Verificar funcionamiento suprarrenal.
CASO CLINICO
Se trata de recién nacido de 3 horas de nacido, el
  cual nace prematuro, de madre adolecente con
  cervicovaginitis y parto distócico, inicia con
  taquipnea, dificultad respiratoria, con aleteo
  nasal, con disociación toraco abdominal, con
  mala tolerancia a la vía oral, con hipotermia, Los
  datos radiográficos de control se observo
  persistencia de imágenes radiológicas en ambos
  campos pulmonares tipo infiltrado, con síndrome
  de fuga, el recuento de leucocitos fueron
  predominio eosinofilos.
PREGUNTA No. 27
Considerando las condiciones clínicas así como
  antecedentes patológicos, cual es el agente
  etiológico más probable?

RESPUESTA
a- Clanmydia
b.- Klepsiela
c.- Candida
d.- Garnerella
PREGUNTA No. 28
Cual es el tratamiento farmacológico en espera de los
  resultados de cultivo?

a.- Ampicilina y amikacina.
b.- Ampicilina y gentamicina.
c.- Ceftriaxona y vancomicina.
d.- Imipenem y Cefotaxima.
CASO CLINICO

Se trata de paciente de 35 semanas de gestación de 5
  días de nacido, con ruptura prematura de membrana
  el cual requirió reanimación intensiva debido a pagar
  2/6, se observo fiebre materna durante el trabajo de
  parto, la madre vive en medio rural, al nacer
  presento apnea, crisis convulsivas, vomito, distermia,
  taquicardia, disnea y deterioro, hiporreactivo con
  piel marmórea, distención abdominal, acude por
  flacidez, falta de respuesta, sin respuesta al medio
  externo.
PREGUNTA No. 29
Cuales de los diagnósticos diferenciales es el menos
  frecuente?

RESPUESTA
a.- Síndrome de adaptación del recién nacido.
b.- Taquicardia transitoria del recién nacido
c.- Hemorragia ventricular.
d.- Hipertiroidismo congenito.
CASO CLINICO
Se encuentra paciente de 42 años de edad el cual se
  conoce hipertenso con buen apego a tratamiento,
  tiene antecedentes de endarteritis de takayasu,
  refiere cefalea global, con acufenos y fosfenos, refiere
  amaurosis fugax y dolor retrosternal ardoroso y
  opresivo, se observo soplo carotidea, disminución de
  la fuerza muscular, disminución de la sensibilidad en
  de miembros, disminución de pulsos, con hipotermia
  periférica, se observa desviación de choque de punta,
  galope, sin de edema pulmonar e imagen de
  ensanchamiento mediastinal. Sus constantes vitales
  fueron TA 210/119 mmHg, FC 123 lpm, FR 31 rpm.
PREGUNTA No 30
Cual de las siguientes complicaciones es la mas
  probable que se encuentra en este caso?

RESPUESTA
a.- Insuficiencias cardiaca aguda.
b.- Disección de la aorta.
c.- Bloqueo AV de tercer grado.
d.- Encefalopatia hipertensiva
CASO CLINICO

Masculino de 51 años con VIH, hepatitis C y
 alcoholismo de 30 años. Consume un litro de vodka
 al día durante 2 meses. El alcohol en sangre era
 269mg/dL, refiere náuseas, diarrea y dolor
 abdominal. Mala dentadura y eritema, atribuido a
 dermatitis seborreica crónica. Laboratorios e
 imágenes normales.
PREGUNTA No. 31
Cuál es la conducta terapéutica más importante
a seguir en este caso a su ingreso?

RESPUESTA
a.- Tiamina, ácido fólico y multivitaminas.
b.- Diazepam 20 mg/c 8 hrs. IV.
c.- Solución mixta 10 ml/kg/c 8 hrs.
d.- Carbamacepina 800 a 1200 mg en 24 hrs.
PREGUNTA No. 32
Presenta agitación, agresividad, desorientación,
respuesta alterada a estímulos y se presentaron
signos de inestabilidad autonómica.

RESPUESTA
a.- Diacepam 20 mg y haloperidol 5 mg.
b.- Haloperidol 10 mg y levomepromacina 25 mg.
c.- Diacepam 20 mg y loracepam 4 mg.
d.- Diacepam 20 mg, haloperidol 10 mg y
   metoprolol 50 mg.
PREGUNTA No 33
Los síntomas mentales y autonómicos con leve mejoría,
se observa "eritema sin brillo, de color rojo oscuro con
escamas sueltas en cabeza, cara, cuello, pecho y brazos,
que comorbilidad más probable presenta?

RESPUESTA
a.- Delirium tremens con síndrome de mala absorción.
b.- Delirium tremens y psicosis de korsakof.
c.- Delirium tremens y encefalopatía de wernicke.
d.- Delirium tremens y pelagra.
PREGUNTA No 34
Cual es el tratamiento a largo plazo más adecuado?
RESPUESTA
a.- Ácido valproico 250 mg, niacina 150 mg, tiamina 100
   mg y niacina 150 mg.
b.- Lamotrigina 100 mg, niacina 100 mg, ácido fólic0 30
   mg y hierro aminoquelado 0.800 mg
c.- Ácido valproico 250 mg, niacina 100 mg y sulfiram 250
   mg cada 8 hrs.
d.- Carbamacepina 300 mg cada 8 hrs, niacina 150 mg,
   tiamina 100 mg y riboflavina 4 mg.
.
PREGUNTA No 35
Luego del alta, volvió a beber y no llevo el
tratamiento. fue readmitido para desintoxicación,
presento la erupción eritematosa fotodistribuida.
Cuál es la complicación más frecuente en el SNC ?

RESPUESTA
a.- Daño pontino.
b.- Daño cerebelar.
c.- Daño frontotemporal.
d.- Daño diecencefalico.
PREGUNTA No 36
Ingresa a urgencias 2 años después, sin tratamiento, con
alcoholismo activo, a la EF disneico, diaforético, confuso,
delirante, ingurgitación yugular, tos con expectoración
asalmolenada, estertores crepitantes, galope, obesidad central,
reflujo hepatoyugular y edema de miembros inferiores. Cuál es la
conducta diagnostica inmediata más adecuada?

RESPUESTA
a.- Cuantificación de CD4, carga viral de VHC.
b.- Ultrasonografia hepática y vías biliares.
c.- Ecocardiograma transesofagico.
d.- Radiografía de torax.
PREGUNTA No 37
Considerando los estudios realizados cual de los
siguientes hallazgos nos proporciona una orientación
diagnostica mas adecuada?

RESPUESTA
a.- 200 CD4 y 400 copias /ml.
b.- Hepatomegalia, con aumento de ecogenicidad.
c.- Fracción de eyección < 30%.
d.- Evidencia de edema pulmonar.
CASO CLINICO

Mujer caucásica de 54 años de edad con dependencia
 del alcohol y dismotilidad esofágica se presentó a
 urgencias con diarrea, vómitos y pérdida de 15 kg de
 peso en seis meses, con estado mental alterado. En la
 TAC abdominal sugiere cirrosis. Por laboratorio se
 reporta anemia macrocítica, trombocitopenia,
 hipoalbuminemia y concentraciones elevadas de
 aspartato y alanina aminotransferasas.
PREGUNTA No. 38
Cual es la conducta a seguir más adecuada?

RESPUESTA
a.- Reposición de líquidos y suplementos vitamínicos.
b.- Diazepam 10 mg más haloperidol 10 a 20 mg.
c.- Concentrado eritrocitario y plasma fresco.
d.- Enema, neomicina y dieta hipoproteica.
PREGUNTA No 39
Recibió reposición de líquidos y suplementos vitamínicos,
   altas dosis de diazepam debido a su estado alterado
   con agitación. 3 días después se observa temblor en
   reposo, ataxia prominente e incapacitante, cuál opción
   sería más adecuada para una aproximación
   diagnostica?
RESPUESTA
a.- Realizar BH, QS, EGO, PFH.
b.- Realizar resonancia magnética.
c.- Electroencefalograma.
d.- Electromiografía y Potenciales evocados.
PREGUNTA No 40
IRM con hiperintensidades, el EEG: ondas lentas difusas.
  A la EF somnolienta, orientada en persona,
  perseverante y desorganizada, discurso pobre y afecto
  lábil además diarrea persistente y glositis. Cual es la
  conducta farmacológica mas adecuada?
RESPUESTA
a.- Rivastigmina y memantina.
b.- Niacina, tiamina, riboflamina, cianocobalamina.
c.- Lamotrigina, acido valproico, quetiapina.
d.- Ziprasidona, biperiden, clonacepam.
CASO CLINICO
Paciente masculino de 61 años de edad hipertenso mal
  controlado y diabetes mellitus, acude a urgencias por
  cefalea intensa, vomitos intenso, el paciente
  presenta    datos     compatibles     con    delirium
  caracterizado por agitación psicomotriz, presento
  vomito importante, su alteraciones neurologías no
  fueron focalizados y se presento lentamente,
  durante la exploración clínica presento disartria y
  hemiparesia, los signos vitales TA 205/120 mmHg.
PREGUNTA No. 41
cual es la complicación mas probable que presento el
  paciente?

RESPUESTA
a.- Hemorragia cerebral.
b.- Encefalopatia hipertensiva.
c.- Edema pulmonar.
d.- Meningoencefalitis.
PREGUNTA No. 42
Cual es la meta sobre la hipertensión arterial en la
  primera hora?

RESPUESTA
a.- Disminuir 25 % de la diastólica.
b.- Disminuir 50 % de la diastólica.
c.- Disminuir 25 % de la sistólica.
d.- Disminuir 50 % de la sistólica.
PREGUNTA No. 43
Cual es tratamiento de primera elección en este caso?

RESPUESTA
a.- Betabloqueadores.
b.- Nitroglicerina
c.- Nitroprusiato.
c.- Calcioantagonistas.
PREGUNTA No. 44
Se agrego al tratamiento diurético al manejo, el
  paciente presenta rahs cutáneo en alas de mariposa,
  cual es la causa de la presencia de este síndrome de
  lupus Like?

RESPUESTA
a.- Hidroclorotiazida.
b.- Clorotiacida.
c.- Furosemide.
d.- Hidralacina.
CASO CLINICO
Inicio con cefalea, visión borrosa, con dolor toraccico
   opresivo, el paciente es diabético e hipertenso con
   mal apego a tratamiento, la diatolica se observo en
   115, refirió disminución de la agudeza visual, con
   necrosis fribinoide, con hemorragia de la retina, y
   exudados algodonosos, sus antecedentes fueron
   tabaquismo, anemia hemolítica, con cardiomegalia
   sin datos de edema pulmonar. Uremia,
   Anteriormente presento hemorragia cerebral hace 2
   años.
PREGUNTA No. 45
El paciente recibió previamente al traslado recibió
  dihidropiridina sublingual, cual es el tratamiento mas
  adecuado en este caso?

TRATAMIENTO
a.- Nitroprusiato de sodio.
b.- Nitroglicerina.
c.- Diurético de asa.
d.- Dihidropiridina.
PREGUNTA No. 46
La paciente respondió bien al cabo de una semana de
tratamiento elegido, el delirio y ataxia habían mejorado
notablemente, afecto agradable y reactividad apropiada,
cual es el diagnostico actual con la respuesta observada?

RESPUESTA
a.- Encefalopatía de Wernicke.
b.- Psicosis de korsakoff.
c.- Deficiencia severa de niacina.
d.- Trastorno orgánico cerebral.
CASO CLINICO
Hombre caucásico de 61 años de edad, alcohólico con
cardiomiopatía dilatada, previo al ingreso bebió con
mala alimentación durante 3 meses continuos. A su
ingreso recibió clordiazepóxido, así como la tiamina y
ácido fólico todos los días. 3 días después recibió 5 mg
de haloperidol IM por agitación, continuó con
desorientación fluctuante y tuvo un episodio de hipoxia
de etiología incierta.
PREGUNTA No. 47
El paciente presento saturación del 90 %, rx de torax
   normal, presentaba erupción eritematosa con
   escamas en su puente nasal y la hiperpigmentación
   de la piel en áreas expuestas al sol, desorientado en
   lugar y fecha con marcha atáxica, cual es la conducta
   a seguir mas adecuada?
RESPUESTA
a.- Nicotinamida oral 100 mg cada 8 hrs.
b.- Neuroléptico bloqueador D2 tipicos.
c.- Diazepam 10 mg cada 8 hrs.
d.- Antipsicotico bloqueador D2 y 5Ht2a.
PREGUNTA No. 48
El tratamiento de la abstinencia al alcohol es un grave
problema debido a su alta morbilidad y mortalidad, los
pacientes desarrollan delirio y reciben altas dosis de
benzodiacepinas. Cual de las siguientes condiciones es
menos frecuente?

RESPUESTA
a.- Deshidratación y alteraciones electrolíticas.
b.- Trauma craneal y hematoma subdural crónico.
c.- Pancreatitis aguda y/o crónica.
d.- Deficiencia grave de niacina.
PREGUNTA No. 49
La vitamina B3 en el metabolismo funciona como una
coenzima o co-sustrato en una amplia gama de
reacciones redox, cual es la forma principal con la que
actúa?

RESPUESTA
a.- Hidrógeno nicotinamida adenina dinucleotido.
b.- Nicotinamida adenina fosfato.
b.- Nicotinamida adenina dinucleotido fostato de hidrogeno.
c.- Hidrógeno nicotinamida dinucleotido.
PREGUNTA No. 50
La niacina se encuentra naturalmente en forma de
nicotinamida y acido nicotínico donde 1mg de cada uno
representa NE (niacina uno). A cuantos días de deficiencia
de niacina se pueden presentar su sintomatología?

RESPUESTA
a.- 30 días aproximadamente.
b.- 60 días aproximadamente.
c.- 90 días aproximadamente.
d.- 120 días aproximadamente.
PREGUNTA No. 51

Cual es la prevalencia de deficiencia de niacina en los
pacientes con dependencia de alcohol con cuidados a
largo plazo y que murieron durante la hospitalización por
la deficiencia niacina con tratamiento potencialmente
reversible?

RESPUESTA
a.- 10 a 20 %.
b.- 20 a 30 %.
c.- 30 a 40 %.
d.- 40 a 50 %.
CASO CLINICO

• Femenino de 21 años de edad la cual acude a urgencias
  secundaria a perdida del estado de alerta, refiere
  acompañantes que se encontraba en clases cuando
  cayo de su propia altura sin causa aparente, a la
  exploración física se observa mucosas orales
  deshidratadas, palidez generalizada, piel reseca,
  cabello quebradizo y de caída fácil, observa un estadio
  tanner no correspondiente a la edad y al interrogatorio
  refiere pirosis, disfagia y vómitos ocasionales con trazas
  de sangre.
PREGUNTA No. 52
Considerando su impresión diagnostica al ingreso a
urgencias cual es su abordaje terapéutico?

RESPUESTA
a.- Inhibidor selectivo de la recaptura de serotonina.
b.- Administración de glucosa al 50 %.
c.- Administración de solución mixta.
d.- Omeprazol 40 mg/12 hrs IV.
PREGUNTA No 53
Considerando el padecimiento cual es la alteración
hidroelectrolitico cronica mas probable?

RESPUESTA
a.- Hiponatremia.
b.- Hipercalcemia.
c.- Hiperkalcemia.
d.- Hipokalemia.
PREGUNTA No. 54
Cual es la incidencia y prevalencia de este trastorno
Mental en mujeres?

RESPUESTA
a.- 8 por 100 000 y 0.3%.
b.- 10 por 100 000 y 3.0%.
c.- 15 por 100 000 y 2 %.
d.- 12 por 100 000 y 1.5 %
PREGUNTA No 55
Considerando el eje mas frecuentemente alterado en
esta patología que estudios seria confirmatorio?

RESPUESTA
a.- Perfil tiroideo.
b.- Perfil suprarenal.
c.- Perfil gonadal.
d.- Perfil de crecimiento.
PREGUNTA No. 56
Las anomalías en las glándulas suprarrenales y la
  glándula tiroides generalmente se invierte a la
  rehabilitación nutricional, cual es la segunda
  complicación mas frecuente de larga duración?

RESPUESTA
a.- Baja estatura.
b.- Infertilidad.
c.- Osteoporosis.
d.- Amenorrea
PREGUNTA No. 57
La paciente egresa por mejoría es enviada a psiquiatría,
  donde recibe tratamiento farmacológico, cual de los
  siguiente esquemas es el mas adecuado para el caso?

RESPUESTA
a.- Amitriptilina 150 mg en 24 horas.
b.- Fluvoxamina 100 a 300 mg en 24 hrs.
c.- Fluoxetina 20 a 80 mg por día.
d.- Velanfaxina 50 a 200 mg en 24 hrs.
PREGUNTA No. 58
La paciente regresa 8 semanas después de iniciado el
  tratamiento, refiere ansiedad con agitación leve,
  estreñimiento, cefalea pero no ha presentado mejoría
  del cuadro inicial, cual es su conducta ?

RESPUESTA
a.- Ajusta dosis según K/DOQI.
b.- Cambian a un ISRS sedante.
c.- Mantiene el tratamiento actual.
d.- Ingresa a para TEC.
PREGUNTA No. 59
Ingresa urgencias con crisis convulsivas, perdida de control
   de esfínteres, hiperreflexia, hipertermia, TA 160 lpm, 60
   rpm, palidez generalizada, ideas delirantes, pH 7.1, pCO2
   31, pO2 84, gasto urinario < 30ml, enzimas hepáticas y
   cardiacas    elevadas,     presentando     un    síndrome
   serotoninergico maligno cual es la conducta mas adecuada
   sobre el fármaco especifico?

RESPUESTA
a.- Vía aérea, oxigenación y ventilación mecánica.
b.- El carbón activado con sorbitol y lavado gástrico,
c.- Medidas generales de soporte y sintomáticas.
d.- Diuresis forzada ó diálisis y plasmaféresis.
CASO CLINICO
Masculino de 24 años, se encuentra en urgencias por
 referir inestabilidad emocional, taquicardia y opresión
 en el pecho, temor a ser secuestrado debido a sus
 conocimiento especiales, los familiares agregan que
 desde los 18 años dejo la escuela comenzó a
 comportarse extraño, se extravió 3 semanas,
 regresando sucio, mencionando que fue a buscar a
 dios para hablar con él, a la EF se observa falta de
 contacto visual, mutista selectivo, disperso, con
 rumiaciones, su constantes vitales se encuentran
 normales.
PREGUNTA No 60
Cual es la conducta terapéutica de segunda elección
  mas adecuada?

RESPUESTA
a.- Haloperidol 10 mg y levomepromacina 25 mg IM.
b.- Risperidona 2mg cada 12 hrs.
c.- Olanzapina 5 mg cada 12 hrs.
d.- Haloperidol 5 mg cada 12 hrs.
PREGUNTA No 61
Se yugulan síntomas motivo de ingreso se realiza cambio
  en la terapéutica para mayor apego así para disminuir
  la conducta autística, cual seria el fármaco a elegir?

RESPUESTA
a.- Haloperidol 10 mg en 24 hrs VO.
b.- Levomepromacina 25 en 24 hrs VO.
c.- Quetiapina 200 mg cada 12 hrs VO.
d.- Risperidona 3 mg cada 12 hrs VO.
PREGUNTA No. 62
Cual es el diagnostico mas probable?

RESPUESTA
a.- Esquizofrenia paranoide.
b.- Esquizofrenia hebefrenica.
c.- Esquizofrenia desorganizada.
d.- Esquizofrenia simple.
PREGUNTA No 63
El paciente regresa en 6 meses con un nuevo cuadro
   psicótico, se ingresa a urgencias, el familiar refiere
   que el paciente no quiere tomar el medicamento por
   los efectos adversos, de las siguiente manejos
   coadyuvantes es el mas adecuado para contrarrestar
   los efectos indeseables?

RESPUESTA
a.- Difenidramina 25 mg.
b.- Trihexifenidilo 5 mg cada 8 horas
c.- Clonacepam 2 mg cada 24 hrs
d.- Biperiden 25 mg cada 24 hrs.
PREGUNTA No 64
Considerando la naturaleza del fármaco agregado cual
  de los siguientes manifestaciones no son propias de
  este grupo farmacológico?

RESPUESTA
a.- Pupilas dilatadas, piel y mucosa oral secas.
b.- Elevación de la temperatura corporal.
c.- Bradicardia o bloqueos.
d.- Distensión abdominal y retención urinaria.
PREGUNTA No 65
Cual de las siguientes no es una contraindicación del
  Biperideno?

RESPUESTA
a.- Hipertrofia prostática benigna.
b.- Temblor esencial idiopático.
c.- Síndrome de QT largo.
d.- Enfermedad de alzheimer.
CASO CLINICO

Se trata de paciente femenino de 81 años que ingresa
  a urgencias por quemadura de segundo grado al
  cocinar en casa, la paciente llego 6 horas después
  ya que se encontraba sola en casa, a la EF se
  percibe olor a urea, descuidada, desalineada, se
  observa síntomas afásicos, apraxicos y amnésicos.
  (ver imagen)
PREGUNTA No. 66
Cual es el diagnostico mas posible con la clínica y
  gabinete?

RESPUESTA
a.- Demencia por tautopatia
b.- Enfermedad de pick.
c.- Demencia de cuerpo de lewy
d.- Demencia por enfermedad de pequeños vasos.
PREGUNTA No 67
El paciente muestra signos de demencia leve a
  moderada, cuál de los siguientes manejos tiene
  mayor utilidad sobre la reserva del núcleo de
  meynert, reduciendo la actividad glutamatergica?

RESPUESTA
a.- Memantina.
b.- Galantamina.
c.- Rivastigmina.
d.- Donepecilo.
PREGUNTA No. 68
Se realiza resonancia magnética cual de las siguientes
  características no es frecuente de este tipo de
  demencia?

RESPUESTA
a.- Atrofia generalizada.
b.- Atrofia de predominio frontal.
c.- Lesiones de núcleos basales.
d.- Atrofia periventricular.
PREGUNTA No 69
Considerando las alteraciones fisiopatológicas cual de
  las siguientes respuestas no corresponde a la
  actividad de las ovillos neurofibrilares y las aplacas
  amiloideas?

RESPUESTA
a.- Neurotoxicidad mediada por glutamato.
b.- Liberación de ROS y NOS.
c.- Depleción de GABA y Glicina.
d.- Liberación factores inflamatorios.
PREGUNTA No. 70
Acude a consulta por infección urinaria, al interrogatorio
  se observa descuidado con irritabilidad actitud
  alucinada, con tendencia a la somnolencia,
  incongruente, ideas irracionales y agresividad, perdida
  del núcleo semántico, se diagnostica delirio, cual es el
  mejor abordaje?

RESPUESTA
a.- Olanzapina 5mg dosis respuesta.
b.- Haloperidol 5 mg dosis respuesta.
c.- Risperidona 2 mg dosis respuesta.
d.- Clonacepam 2 mg dosis respuesta.
CASO CLINICO
Se trata de femenino de 19 años con tristeza, llanto
  fácil, ideas de minusvalia, acudió por insomnio,
  agresividad y falta de control de impulsos, la madre
  refiere que es muy cambiante, desde niña era muy
  problemática, aislada y poco flexible, al examen
  mental se observa renuente, no cooperadora, con
  tendencia a la irritabilidad, con llanto fácil, ideas
  suicidas y homicidas, con cambios de expresión
  frecuentes.
PREGUNTA No. 71
Se indico un ISRS, a las 6 semanas, la paciente refiere
  estar bien, con energía, quiere regresar a la escuela,
  al trabajo. La madre refiere que duerme poco, esta
  muy activa, desaparecieron los síntomas depresivos,
  considerando la respuesta al tratamiento cual es la
  conducta mas adecuada a seguir?

RESPUESTA
a.- Mantener el tratamiento.
b.- Retirar el tratamiento.
c.- Cambiar el tratamiento.
d.- Reducir el tratamiento.
PREGUNTA No. 72
3 meses después la paciente refiere sentirse mejor pero
  algo aburrida, la madre refiere que aun tiene
  insomnio, cambios de conducta, nuevamente
  presenta llanto con irritabilidad e inestabilidad con
  bajo control de impulsos, cual es la conducta a seguir?

RESPUESTA
a.- Llevar a dosis mínima toxica de eutimizante.
b.- Agregar un eutimizante de diferente grupo.
c.- Agregar antidepresivo triciclico.
d.- Agregar un agonista antagonista parcial D2.
PREGUNTA No 73
Cual es la complicación psiquiátrica mas frecuente
  considerando la edad de inicio así como el sexo de la
  paciente?

RESPUESTA
a.- Suicidio.
b.- Toxicomanías.
c.- Ansiedad.
d.- Trastorno de pánico.
CASO CLINICO
Masculino de 43 años de edad el cual acude a
 urgencias en la tarde, refiere dolor torácico,
 dificultad respiratoria, sudoración, temblor de
 miembros superiores, mareo y sensación de
 desmayo, estas manifestaciones se ha presentado
 en otros momentos, el paciente cuenta con
 antecedentes de importancia, como IAM el padre,
 DM en la madre, a la exploración física se observa
 obesidad grado III, leve palidez de tegumentos, SV
 150/95 mmHg, FC 102 lpm, FR 41 rmp.
PREGUNTA No. 74
Cual es la complicación psiquiátrica mas frecuente
  considerando la edad de inicio así como el sexo de la
  paciente?

RESPUESTA
a.- Suicidio.
b.- Toxicomanías.
c.- Ansiedad.
d.- Trastorno de pánico.
CASO CLINICO

• Male is 78 years old which comes in for joint
  pain in his left knee during the scan you
  identify that your blood pressure are as
  follows, TA 80/170mmHg, repeat the pressure
  test and remains such, the patient denied
  symptoms associated with tension figure. The
  patient has a history of a transient ischemic
  episode 6 months ago.
QUESTION No. 75
What is action to take to handle this case?

ANSWER
a. - Weight reduction and salt.
b. - Hydralazine 25 mg every 24 hrs.
c. - Captopril and hydralazine.
d. - Enalapril and lisonipril.
QUESTION No 76
What is the purpose of the BMI for the treatment of the
 patient?

ANSWER
a. - Keep the body index between 20 and 25 kg / m2.
b. - Maintain systolic blood pressure at least 140 mmHg.
c. - Keep diastolic blood pressure less than 90 mmHg?.
d. - Reduce the risk of MI or stroke.
QUESTION N0. 77
What is the treatment of choice            considering
 pharmacological history of the patient?

ANSWER
a. - Enalapril 10 to 20 mg.
b. - Losartan 50 mg
c. - Hydrochlorothiazide 12.5 to 25mg.
d. - Amlodipine.
QUESTION No 78
The patient is currently stable with blood pressure of
  120/80 mmHg, which of the following milestones
  must perform to shipping to third level?

ANSWER
a. - Perform QS, BH, EGO.
b.- Laboratories - more routine CXR and EKG.
c. - Seek first target organ damage.
d. - It is not necessary as it was a recent find.
CASO CLINICO
Se trata de masculino de 22 años de edad el cual es
  ingresado al servicio de urgencia debido a la
  presencia de cefalea intensa, palpitación,
  sensación de muerte inminente, diaforesis,
  refiere que ya le había ocurrido esto
  anteriormente pero no tan intenso, su tensión
  arterial fue de 140/100 mmHg. FC 115, FR 32, se
  observa agitado con dolor precordial resto de
  exploración sin datos por agregar, laboratorios
  de rutina sin datos patológicos.
PREGUNTA No 79
Considerando la presencia del cuadro clínico cual es la
  conducta a seguir mas adecuada?

RESPUESTA
a.- Evaluar metanefrinas /24 hrs.
b.- Medicina nuclar con MIBG.
c.- Prueba de supresión con clonidina.
d.- Prueba de supresión con glucagon.
PREGUNTA No. 80
Posterior a la realización del estudio anterior, se
  observo una imagen isointensa y continuaron los
  síntomas, cual es la conducta a seguir?

RESPUESTA
a.- Realizar búsqueda en cuello, torax y abdomen.
b.- Identificar CAS/CARS
c.- Supresion con Fluhidrocortisona.
d.- Realizar urografía escretora.
CASO CLINICO
• Female patient is 73 years old which has 20 years of
  hypertension and diabetes mellitus, with medical
  treatment but poor adherence pharmacologic and
  nonpharmacologic, is currently with the following scheme
  100mg/day hydrochlorothiazide, furosemide 20 mg / day,
  enalapril 60 mg / day, amlodipine 10 mg / day
  acetylsalicylic acid 100 mg / day, glipizide 20 mg in two
  divided doses, currently goes for occasional dizziness with
  falling height twice, has lost twice to disregard the people.
  Physical examination shows patient oriented, adequate
  alertness, with slight desihratacion with plethoric facies,
  observed 5 cm jugular venous distention, pulmonary rales
  fields predominantly in the bases, desdoblamiendo heard
  the second heart sound, mild hepatomegaly, lower limb
  edema + + +, which are hot. Vital signs: BP 105/80mmHg,
  HR 109 bpm, FR 31rpm. ECG is performed and shown tele-
  ray. Calculated an EF> 45%.
QUESTION No. 81
What are the clinical manifestations specific for
 diagnosis of the disease?

ANSWER
a. - Orthopnea, nocturnal dyspnea and cough.
b. - Edema, fatigue and chest congestion.
c. - Electrocardiogram and chest radiograph.
d. - Jugular venous distention, hepatomegaly and
   ejection fraction.
QUESTION No. 82
What is the risk factor that can lead to an unbalanced
 state in this case?

ANSWER
a. - Failure of treatment.
b. - OSA.
c. - Arrhythmia, anemia or bleeding.
d. - Class antiarrhythmic Ia and Ic.
QUESTION No 83
What are the clinical features most useful to perform
 a differential diagnosis of diastolic HF vs systolic
 HF?

ANSWER
a. - There are distinguishable.
b. - Dyspnea and edema.
c. - Tachycardia and tachypnea.
d. - Hepatic congestion.
QUESTION No. 84
What are the most useful diagnostic aids that have not
 been performed in patients to identify complications
 of the disease?

ANSWER
a.- Liver function test.
b. - Serum electrolytes.
c. - Stress tests.
d. - Echocardiogram.
QUESTION No. 85
What is the measure predominantly pharmacological
 intervene on neurohumoral response in sodium and
 fluid retention?

ANSWER
a. - Renin-angiotensin-aldosterone system.
b. - Arginine vasopressin system.
c. - Release of atrial natriuretic peptide.
d. - Inhibits transport system Na + Cl -.
QUESTION No 86
Before defining the pharmacological treatment of
  reversible causes which less likely is the case?

ANSWER
a. - Alcohol-induced myocardial depression.
b. - Nonsteroidal anti-inflammatory.
c. - State of elevated afterload.
d. - Using calcium channel blocker.
QUESTION No. 87
Which of neurohumoral changes that occur in heart
 failure is less likely in this case?

ANSWER
a. - Increased myocardial contractility.
b. - increased heart rate.
c. - Increase in venous tone.
d. - Decreased effective central blood volume.
CASO CLINICO
Ingresa paciente de 39 años de edad el cual ingresa
  a urgencias 4 horas antes, por presencia súbita
  de dolor retrosternal acompañado de presión
  toracicca “sensación de mucho peso sobre su
  pecho” que se presento en la madrugada,
  nausea, dificultad para respirar, refiere que la
  noche anterior comió abundante y consumió
  alcohol hasta la embriaguez, al ingreso se
  observa confuso, diaforético con palidez y
  cianosis distal, ingurgitación yugular, se ausculta
  estertores cripitantes bibasales, tercer ruido, TA
  90/60 mmHg, FC 132 lpm, FR 36 rpm, Temp. 36.3
  grados. Gasto urinario de < 20 ml/h, se
  observaron los siguientes estudios
PREGUNTA No. 88
Cual es la clase funcional de Kllip que presenta el
  caso?

RESPUESTA
a.- Clase funcional Killip I.
b.- Clase funcional Killip II.
c.- Clase funcional Killip III.
d.- Clase funcional Killip IV.
PREGUNTA No. 89

Cual es el pronostico para la vida en este caso
  considerando la clase funcional?

a.- Tasa de mortalidad del 6%.
b.- Tasa de mortalidad del 17%.
c.- Tasa de mortalidad del 38%.
d.- Tasa de mortalidad del 81%
PREGUNTA No. 90
Se calculo los siguientes resultados (LC) de 2
  L/m2/min. Considerando los elementos clínicos y el
  (LC), en la curva de starling cual es el diagnostico?

RESPUESTA
a.- Insuficiencia cardiaca.
b.- Edema agudo pulmonar.
c.- Choque cardiogenico.
d.- Choque distributivo.
PREGUNTA No. 91
Se realizó ecocardiograma transesofagico de urgencias,
  donde se reporta FE < 30%, cual es % de mortalidad
  a 6 meses del paciente?

RESPUESTA
a.- del 15 al 20 %
b.- del 20 al 25 %
c.- del 25 al 30 %
d.- 30 al 35 %
CASO CLINICO
Mujer de 70 años de edad, hipertensa, diabética de reciente
 diagnostico durante un examen de rutina y cursando con
 bacteriuria asintomatica, procuró atención médica por tos
 seca, disnea, inapetencia y desánimo. La paciente sabía que
 era portadora de hipertensión arterial desde la edad de 37
 años, de diabetes mellitus desde los 56 años y de
 hipertrigliceridemia. A los 56 años de edad, se quejó de
 palpitaciones taquicárdicas con duración de 20 minutos, sin
 síncope. Hacía uso de 75 mg de captopril, 50 mg de
 clortalidona, 600 mg de quinidina y 0,25 mg de digoxina. El
 examen físico reveló peso de 54 kg, altura de 1,49 m,
 presión arterial de 170/110 mmHg. En el examen físico, fue
 identificado soplo sistólico +/4+ en área mitral. Lo restante
 del examen físico no reveló alteraciones. El ECG mostró
 ritmo     sinusal,   sobrecarga     ventricular    izquierda,
 infradesnivel de ST en V5 y V6 y presencia de onda U.
PREGUNTA No. 92
Cual de los siguientes fármacos tiene mayor evidencia
  de efecto cardioprotector en esta condición del
  paciente?

RESPUESTA
a.- IECA´s
b.- ARA II.
c.- Digoxina.
d.- Clortalidona.
CASO CLINICO
Hombre de 62 años con HTA, IAM y DM tipo 2 diagnosticada.
  Es ingresado por pielonefritis con IR aguda (MDRD 21
  ml/min), interpretándose en el contexto de la propia
  infección, agravada por el consumo de antiinflamatorios no
  esteroideos (AINE) y persistiendo al ser dado de alta, es
  remitido a la consulta de nefrología presentando IR grado 4
  (MDRD 13,83 ml/min), proteinuria de 5 g/24 h y
  microhematuria persistente (urocultivos negativos y
  citología negativa para malignidad). En el resto de la
  analítica destaca: ANA, ANCA, anti-MBG negativos.
  Inmunoglobulinas, cadenas ligeras, complemento y
  proteinograma normales. Serologías VHC, VHB, VIH
  negativas. Ecografía abdominal: riñones de tamaño normal
  con cortical moderadamente ecogénica y sin evidencia de
  dilatación de la vía excretora. Fondo de ojo normal.
PREGUNTA No. 93
Cuál es la conducta más adecuada a seguir en el caso.

RESPUESTA
a.- IECA´s
b.- ARA II.
c.- Dialisis.
d.- Hemodialisis.
CASO CLINICO

Se trata de masculino de 46 años de edad el cual acude
  a consulta debido a que ha presentado un proceso
  infecciosos urinario ya tratado, refiere mejoría
  completa posterior a una semana de tratamiento
  antibiótico, a la exploración identifica un perímetro
  abdominal de 102 cms debido a ello realiza una
  glucosa periférica donde se obtiene 130 mg/dl.
PREGUNTA No. 94
Cual es la conducta a seguir para sustentar el
  diagnostico?

RESPUESTA
a.- Curva de tolerancia a la glucosa.
b.- Glucosa en ayuno.
c.- Determinar hemoglobina glucosilada.
d.- Glucosa pospandrial.
CASO CLINICO
Ingresa a urgencias masculino de 21 años de edad el
  cual se observa ansioso, agitado con actitud
  alucinada, a la exploración se encuentra diaforético
  con palidez generalizada, súbitamente el paciente
  pierde el estado de alerta y presenta parada
  cardiaca, se activa el código de alerta e inicia
  compresiones torácicas, los familiares solo refieren
  que lo golpearon en una fiesta, 10 minutos después
  presento los síntomas. Se observo el siguiente trazo
  del ECG. No hay respuesta, no se detecta pulso, ni
  constantes vitales. Se descarga con 360 j, se
  mantiene soporte por 20 minutos más.
PREGUNTA No. 95
No se percibe pulso pero se observa el siguiente trazo, cual
   de las siguientes causas es la mas probable en este caso?
RESPUESTA
a.- Neumotorax a tensión.
b.- Sobredosis por drogas.
c.- Tromboembolismo pulmonar.
d.- Hipovolemia.
CASO CLINICO

• Se trata de femenino de 28 años de edad que
  acude a planificación familiar para solicitar
  método, cuenta con antecedentes de
  importancia tabaquismo positivo, ingesta de
  alcohol social una vez al mes por lo menos,
  desconoce antecedentes de cáncer en la familia,
  refiere que tiene relaciones sexuales con su
  esposo de 3 a 4 veces por semana desde hace 3
  meses que se caso, es su primera pareja sexual
  desde hace 5 años.
PREGUNTA No. 96
Cuál es el método más apropiado para esta paciente?

RESPUESTA
a.- Inyectable mensual.
b.- Hormonales oral.
c.- Dispositivo intrauterino.
d.- Implante subdermico.
CASO CLINICO

Paciente femenino de 28 años de edad la cual se
  encuentra en pos-aborto de 5 dias, refiere que ya no
  presenta sangrado, acude a solicitar método
  anticonceptico, refiere que no quiere tener mas hijos
  por lo menos en 3 años, tiene 2 hijos, anteriormente
  empleaba hormonales orales y durante este método
  se embarazo, no cuenta con antecedentes de
  cervicovaginitis ni EPI.
PREGUNTA No. 97
Cual de los siguientes métodos de planificación familiar
  es el mas adecuado para este caso?

RESPUESTA
a.- Hormonal inyectado.
b.- Implante hormonal.
c.- Preservativo.
d.- Dispositivo intrauterino.
PREGUNTA No. 98
La paciente acude ha consulta por cambios del flujo
  menstrual, el cual refiere es obscuro y ha presentado
  manchado intermenstrual, a la exploración se
  observa moco cervical con sangre moderada, cual es
  la conducta a seguir?

RESPUESTA
a.- Retirar el DIU.
b.- Indicar un AINES.
c.- Indicar antibiótico.
d.- Indicar DOC.
PREGUNTA No. 99
La paciente acude 6 meses después por dolor abdominal en
   fosa iliaca izquierda que se irradia a hipocondrio y hacia
   tras, a la palpación se presenta dolor abdominal de
   predominio pélvico, acude además porque su DOC
   presento cambios inflamatorios inespecíficos, cual es la
   conducta a seguir por en este caso?

RESPUESTA
a.- Retirar DIU por probable EPI.
b.- Retirar DIU por resultado de DOC.
c.- Retirar DIU por probable embarazo.
d.- Retirar DIU por cambio de método.
CASO CLINICO
Se trata de paciente masculino RN de 60 minutos,
  obtenido por cesarea iterativa, demás de presentar
  sufrimiento fetal, por disminución de actividad
  intrauterina y disminuciones de FC de hasta 110 lpm,
  se observa con movimientos muy finos, al inicio
  presento cianosis leve y breve principalmente
  periungeal y oral, se apreciaron secreciones orales
  abundantes, a la exploración física se auscultan
  estertores bilaterales normales, peristaltismo poco
  audible y temperatura de 36.1 grado, se apreció
  expulsión de secreciones con tinte meconial durante
  la exploración.
PREGUNTA No. 100
Considerando el cuadro clínico cual es su conducta a
  seguir?

RESPUESTA
a.- Mantener al paciente en observación continua.
b.- Buscar posibles patologías ocultas.
c.- Realizar radiografia de torax.
d.- No hay datos patológicos son adaptativo.
PREGUNTA No. 101
Cual es la conducta terapéutica menos apropiada en
  este momento del caso?

RESPUESTA
a.- Mantener ayuno hasta identificar causa.
b.- Colocar en ambiente neutro.
c.- Administrar oxigeno por casco cefálico 40 %.
d.- Gasometria y oximetría de pulso.
PREGUNTA No. 102
Cual de los siguientes parámetros gasométrico no es
  normal esperarlo en el caso?

RESPUESTA
a.- pH < 7.34.
b.- PaCO > 45 mmHg.
c.- HCO3 > 40mEq/L.
d.- SpO2 < 95.
PREGUNTA No. 103
Se realizaron electrolitos séricos por continuar con rechazo
  al alimento y secreción moderada oral, todos los
  resultados resultaron dentro de parámetro normales, cual
  de las siguientes diagnósticos diferencias del acidosis
  respiratoria es el mas probable presentarse?

RESPUESTA
a.- Ventilación inadecuada alveolar.
b.- Desordenes musculares.
c.- Defectos pulmonares.
d.- Trastornos de las vías aéreas.
PREGUNTA No. 104
Considerando los valores gasométricos y de
  electrolitos, cual es la causa de la modificación del
  anion gap, del caso?

RESPUESTA
a.- Error de laboratorio.
b.- Toxinas no identificadas.
c.- Mielomas.
d.- Exceso de soluciones.
CASO CLINICO
Se trata de paciente masculino de 15 horas de vida
  extrauterina, el paciente nació vía cesárea, nació a la
  35 semanas de gestación, se administro
  maduradores a la madre debido a trabajo de parto
  distócico con ruptura de membranas 12 horas antes
  de la intervención quirúrgica, a la exploración física
  se observo taquipnea de 101 rpm, y signo
  moderados de dificultad respiratoria.
PREGUNTA No. 105
Considerando un probable diagnostico clínico
  realizado, cual de los diagnósticos diferenciales es
  menos probable encontrar?

RESPUESTA
a.- Síndrome de adaptación pulmonar.
b.- Síndrome de aspiración de meconio.
c.- Enfermedad de membrana hialina.
d.- Neumonía neonatal hospitalaria.
PREGUNTA No. 106
Se decide realizar radiografía de tórax, cuales son los
  datos no es probable espera encontrar para
  confirmar el diagnostico?

RESPUESTA
a.- Broncograma aéreo.
b.- Hilio congestivo.
c.- Derrame cisurales.
d.- Sobredistencion pulmonar.
PREGUNTA No. 107
Cuales son los factores de riesgo inducido para la
  taquipnea transitoria del recién nacido, cual es la
  mas probable en el caso clínico?

RESPUESA
a.- Sexo masculino
b.- Grande.
c.- Cesárea.
d.- Sedación materna.
PREGUNTA No. 108
Cual es el fenómeno fisiopatológico no es la mas
  adecuada para el caso?

RESPUESTA
a.- Falta de absorción de líquido amniótico.
b.- Falta administración de esteroides.
c.- Deficiencia ligera de surfactante.
d.- Edema pulmonar transitorio.
PREGUNTA No. 109
Considerando el diagnostico actual, el cual muestra
  signos de dificultad respiratoria, resultados de
  laboratorio normales al momento, FiO2 < 0.40
  descartando los diagnósticos diferencias, cuál es la
  conducta a seguir en este caso es menos adecuada?

RESPUESTA
a.- Monitoreo de signos vitales.
b.- Monitoreo de gases por punción cada 6 horas.
c.- Mantener glucosa y electrolitos.
d.- O2 por casco al 70 %
PREGUNTA No. 110
Tomando en cuenta la evolución del caso, cual es su
  pronostico?

RESPUESTA
a.- Bueno.
c.- Malo.
b.- Fatal.
d.- Excelente.
PREGUNTA No. 111
Considerando un probable diagnostico clínico
  realizado, cual de los diagnósticos diferenciales es
  menos probable encontrar?

RESPUESTA
a.- Síndrome de adaptación pulmonar.
b.- Síndrome de aspiración de meconio.
c.- Enfermedad de membrana hialina.
d.- Neumonía neonatal hospitalaria.
CASO CLINICO
Un RN de 3,6 kg nació a las 37 semanas de gestación
  hijo de diabética con un agente hipoglicémico oral.
  Obtenido por cesárea electiva con Apgar de 6/9.
  Desarrollo taquipnea inmediatamente después de su
  nacimiento y requirió oxígeno suplementario. En la
  gasometria presento; fueron PO2 de 57 mm Hg,
  PCO2 de 52 mm Hg, y pH de 7,31. El niño se mantuvo
  en oxígeno por campana. A las 2 horas el paciente se
  encuentra hipotónico, cianótico, con saturación del
  70 %. La Rx de observa rectificación, hiperclaridad e
  incremento de los espacios intercostales y
  congestión parahiliar.
PREGUNTA No. 112
Cual es la medida inmediata a seguir?

RESPUESTA
a.- Intubación orotraqueal.
b.- Alimentación por sonda orogastrica.
c.- Mantener un ambiente neutro.
d.- Realizar medidas de reanimación.
CASO CLINICO

• Paciente masculino 10 días de nacimiento, que se
  obtuvo en casa sin control prenatal, ingresa por
  diarrea, llanto, e irritabilidad, vomito en proyectil, en
  casa, refiere la madre que solo fue en una ocasión, al
  ingreso se observo hipotónico, llanto agudo, mal
  estado generalizado, con tono cervical no se observo
  rigidez, pero fontanela abombada, su peso fue de
  2050 grs, laboratorios con 150,000 de plaquetas,
  antecedentes de rinorrea hialina, se presento 35
  grados, irritabilidad, durante la exploración se
  observo pedaleo y chupeteo, dificultad respiratoria.
PREGUNTA No 113
Para establecer el diagnostico y conducir su
  terapéutica para el caso?

RESPUESTA
a.- Hemocultivo.
b.- Punción lumbar.
c.- Tomografía.
d.- Biometría hemática.
PREGUNTA No. 114
Se encuentra en espera de resultados, sin embargo el
  paciente se va deteriorando, cual es la agente
  etiológico mas probable para dar una terapéutica
  dirigida?

a.- H influenza
b.- E Coli.
c.- Listeria monocitogenes
d.- Estreptocco beta hemolito.
PREGUNTA No. 115
Cual es la conducta farmacológica más adecuada
  considerando la etiología?

RESPUESTA
a.- Ampicilina mas amikacina.
b.- Ampicilina mas metronizadol.
c.- Ampicilina mas vancomicina.
d.- Ampicilina mas cefotaxima.
CASO CLINICO
Varón de 33 años que sufrió un traumatismo torácico
  severo tras atropello. Precisó ventilación mecánica y
  fármacos inotrópicos. Presentaba un soplo de
  insuficiencia aórtica. La TAC mostro neumotórax y
  fracturas costales bilaterales, sin datos de disección
  aórtica. El USG evidenció insuficiencia aórtica severa con
  dilatación ligera de la raíz aórtica, se observó un velo
  coronariano derecho roto, que protruía hacia el tracto de
  salida del ventrículo izquierdo, e insuficiencia aórtica
  severa. En la pared anterior de aorta ascendente
  proximal, había una imagen de seudoaneurisma con
  rotura de capas íntima y media y se observaban colgajos
  medio-intimales protruyendo hacia la luz.
PREGUNTA No. 116
La triada de Beck consiste en:

RESPUESTA
a.- Hipotensión, ruidos cardiacos velados y distensión
   venas del cuello.
b.- Hipertensión, edema facial y ruidos cardiacos
   velados.
c.- Hipotensión, torax inestable y disfagia.
d.- Ruidos cardiacos velados, distensión venas del
   cuello y disfonía.
CASO CLINICO

• Un hombre de 31 años de edad, con antecedentes
  de DM, alcoholismo y tabaquismo.                  Con
  desprendimiento total de retina de su ojo derecho y
  la fotocoagulación con láser de su ojo izquierdo.
  Examen mostró etapa final de retinopatía diabética
  proliferativa con desprendimiento de retina
  traccional.     El    ojo    izquierdo     tenía    la
  neovascularización extensiva y la proliferación
  fibrovascular de la retina secundaria a la retinopatía
  diabética proliferativa avanzada.
PREGUNTA No 117
Cual de los siguientes factores es de mayor importancia
  para el desarrollo de la retinopatía diabética?

RESPUESTA
a.- Más de 5 años de evolución de la DM.
b.- Tabaquismo positivo intenso.
c.- Progresión rápida de la enfermedad.
d.- Depuración de creatinina menos de 60 ml/ minuto.
CASO CLINICO
Un paciente de 78 años, sin antecedentes de
  importancia y con un control periódico normal hace
  6 meses, comienza hace 24 horas con dolor
  quemante, en la región torácica posterior derecha,
  irradiado a flanco e hipocondrio homolaterales. La
  semiología pulmonar es normal y no presenta
  contractura muscular. En el abdomen no hay
  visceromegalias y presenta leve dolor a la palpación
  superficial en hipocondrio derecho. La radiografía de
  tórax y de columna dorsal y la ecografía
  hepatobiliopancreática son normales.
PREGUNTA No. 118
Cuál es el diagnóstico más probable?

RESPUESTA
a.- CCL.
b.- Espondilitis anquilosante.
c.- Herpes zoster.
d.- Pleuritis.
CASO CLINICO

• Un paciente de 87 años con antecedentes de
  bronquitis crónica e insuficiencia cardiaca, ha sido
  diagnosticado de colecistitis aguda litiasica. Tras 4
  dia de hospitalización en tratamiento con dieta
  absoluta, liquidos IV, piperacilina / tazobactam, el
  paciente continua con fiebre, dolor abdominal
  persistente     y    leucocitosis,   desihidratacion,
  hipeglucemia, hipotension, y taquicardia.
PREGUNTA No. 119
La actitud mas adecuada en este momento seria?

a.- Colecistectomia urgente.
b.- Drenaje biliar mediante colecistostomia percutanea.
c.- Sustituir la piperacilina/tazobactam por
     metronidazol+cefotaxima.
d.- Sustituir la piperacilina/tazobactam por
     amikacina+clindamicina.
CASO CLÍNICO

• Se trata de paciente de 24 años de edad la
  cual acude a solicitud de la pareja debido a
  que presenta secresion por meato urinario, de
  tipo ardoroso, sin embargo la paciente no
  presenta manifestaciones, ocasionalmente
  presenta descarga vaginal, la EF se observa
  leve descarga directa del cervix.
PREGUNTA NO. 120
Cual es el diagnostico mas probable que presenta la
  pareja?

RESPUESTA
a.- Gonorrea.
b.- Tricomoniasis.
c.- Candida
d.- Garnerella
PREGUNTA NO. 121
Cual es el diagnostico mas probable que presenta la pareja?

RESPUESTA
a.- Espectinomicina 2 gr.
b.- Penicilina procainica 4 millones previo probenicid 1 gr.
c.- Ciprofloxacilina 2 grs dosis unica.
d.- Ceftriaxona 2 gr. IM DU.
Pregunta No. 122
Cual de los siguientes agentes infecciosos no es
  diferencial en este caso?

RESPUESTA
a.- Toxoplasma.
b.- Ureplasma.
c.- Sifilis.
d.- Clanmidya
Pregunta No. 123
Acude a consulta la pareja de la paciente, cual presenta
  secresión en meato urinario, durante la exploración
  observa hiperemia conjuntival. Cual es el agente mas
  probable que se presenta en la mucosa
  subconjuntival?

RESPUESTA
a.- Toxoplasma.
b.- Ureplasma.
c.- Sifilis.
d.- Clanmidya
Pregunta No. 124
Se trata de paciente femenino que acude por descarga
  vaginal frecuente, sin mal olor, la pareja no presenta
  sintomatologias, no es ardorosa, ni ulceraciones, se
  presume que presenta una cervicitis por mycoplasma,
  cual es el medio mas adecuado para cultivar este
  agente?

RESPUESTA
a.- Caldo de argirina.
b.- Caldo de urea.
c.- Medio agar.
d.- Medio saburoe.
Pregunta No. 125
Se trata de paciente femenino que acude por descarga
  vaginal frecuente, a la revisión vaginal se observa
  ulceración en cérvix, la secreción es amarillenta, con olor
  desagradable, leve dolor a la palpación y negativo a la
  prueba de aminas, se envía secreción a cultivo, cual es el
  fármaco de elección en espera de resultados?

RESPUESTA
a.- Ceftriaxona
b.- Metronidazol
c.- Fluconazol
d.- 5 fluoracilo.
Pregunta No. 126.
Se trata de paciente femenino que acude por descarga
  vaginal frecuente, ardor vulvar con secresion de lesion
  ulcerosa, a la exploracion fisica se detecta cadena
  ganglionar inflamada, cual es su conducta a seguir?

RESPUESTA
a.- Aspirado de ganglios.
b.- Cultivo de secresiones.
c.- Realizar VDRL.
d.- Colposcopia.
Pregunta No. 127
Se trata de paciente femenino que acude por descarga
  vaginal frecuente, ardor vulvar, con bubon unilateral y
  ganglios palpables?

RESPUESTA
a.- Herpes genital
b.- Linfogranuloma venerio.
c.- Chancroide
d.- Sifilis secundaria.
Pregunta No. 128.
Se trata de paciente femenino que acude por descarga
  vaginal frecuente, ardor vulvar intenso con vesículas
  múltiples, acudió 5 días después de iniciado, intento usar
  una crema, la cual incremento las lesiones, a la
  exploración física se observa lesión corticoestropeada
  cual es el diagnostico mas probable?

RESPUESTA
a.- Herpes tipo I
b.- Herpes tipo II
c.- Herpes genital.
d.- Herpes zoster.
Pregunta No. 129.
Se trata de paciente femenino que acude por descarga
  vaginal frecuente, ardor y dolor en vulva, con
  inflamacion importante en ganglios, con bubon, por
  arriba de ligamento inginal, con el signo del surco
  dividido?

RESPUESTA
a.- Linfogranuloma venereo
b.- Herpes genital.
c.- Balanosis.
d.- Chancroide.
Pregunta No. 130.
Cual de las siguiente complicaciones es la menos probable?

RESPUESTA
a.- Queratosis.
b.- Elefantiasis
c.- Cervicitis crónica.
d.- Paralisis y demencia.
Pregunta No. 131
Se trata de femenino de 10 años que presenta lesiones
  sugerentes de hiperqueratinocitis, abundante con
  lesiones que sobrepasan la piel, algunas lesiones miden
  mas de 1 cm, lesiones umblicadas, tiene un aspecto
  perlado, sin descamación, con prurito, cual es su
  impresión diagnostica?

RESPUESTA
c.- Queratotenoma.
b.- Molusco contagioso.
c.- Lesiones de ducrey.
d.- Citomegalovirus.
Pregunta No. 132
Se trata de femenino de 21 años de edad que presenta
  cansancio, fatiga, adinamia, con linfadenopatia, hígado
  con tamaño dentro de lo normal, esplenomegalia grado
  I, con fiebre, cual es el agente etiológico mas probable?

RESPUESTA
a.- Mononucleosis.
b.- Citomegarovirus
c.- Hepatitis B
d.- VIH.
Pregunta No. 133
Se trata de femenino de 28 años de edad la cual acude por
  cansancio, fatiga, anorexia, adinamia, diarrea, dolor a la
  palpación abdominal de predominio cuadrante superior
  derecho, los cultivos de secreciones resultaron negativos
  para los agentes de ETS habituales?

RESPUESTA
a.- Mononuclesosis
b.- Virus herpes genital.
c.- Citomegarovirus
d.- Hepatitis B
CLINIC CASE
A 16 year old female familiar to you from previous yearly visitis
  was brought to your office today by her mother who is
  concerned about her frequent complaints of dizziness and
  weaknes. There is no hitory of syncope, but the patient often
  remarks that se feels as if she is about to “pass out”. Revies of
  systems is negative for fever, weight gain or loss, nigth
  sweats, chest pain, palpitations, increased work of breathing,
  vomiting or diarrhea. Weigth continues to track along the /
  5th percentile; height remains at the 50 th percentile, Vitial
  signs are within normal limits for age, including orthostatic
  blood pressure measurements. The physical examination is
  unremarkable except for some excess wear on the upper
  incisor.
QUESTION No. 134
Tests Na 134 mEq/L, K 2.8 mEq/L, Cl 83 mEq/L, HCO3
  35 mEq/L, creatinina 0.6 mg/dL, Urea 12 mg/dL,
  pH7.5, urine pH 9.0 This patient´s physical
  examination and laboratory values are most
  consistent with of the following condictions?

ANSWER
a.- Zinc deficiency.
b.- Renal tubular acidosis.
c.- Anorexia nervosa.
d.- Bulimia nervosa.
CLINIC CASE
An 8 year old female is brought to your office the
  weeked after july 4 th for evaluation of fever. The
  child was well until yesterday, when she complained
  of a headache. Overnight she developed a fever to
  39.9 C. and began complaining of achiness “all over”
  Thay also want you to look at a red place on her
  upper arm.
  You note an irregular annular about 4 cm across at
  the widest, with partial central clearinq. The areas is
  not scaly, and the child denies pain or pruritus
  associated with the lesion.
QUESTION No. 135
Antibiotic therapy doxycycline twice a day for 10 days,
  should be administered to prevent wich of the
  following complications of this patient´s infection?

ANSWER
a.- Encephalitis.
b.- Liver failure.
c.- Relapsing arthritis.
d.- Rheumatic heart disease.
CLINICAL CASE

A 9 year old male sustains an inversion injury to his
  ankie while playing soccer. He is untable to bear any
  weigth on the affected ankle, which is markedly
  swollen. Tendernes is most pronunced over the
  lateral malielu, particulary the distal fibular swelling
  but are otherwise normal.
QUESTION No. 136
Which of the following is the most likely result of this
 boy´s injury?

ANSWER
a.- Frature of the distal fibular epiphysis.
b.- fracture of the 5th metatarsal.
c.- Rupture of the peroneal tendon.
d.- Sprain of the calcaneofibular ligamente.
CLINIC CASE
During the initial assessment of a therm newborn, you note
  empty scrotal sacs on goth the right and the left. A small
  mobile mass the size of a typical newborn testis is
  palpated along the left inguinal canal; gentle traction
  does not result entry of the mass into the left scrotal sac.
  No similar mass is present on the right side.

The genital, rectal, and back examinations are otherwise
  unremarkable. A sonogram of the pelvis confirms the
  right testis in the inguinal canal. The left testis is in the
  abdomen. Orchiopexy is successful at 12 months of age.
QUESTION NO. 137
This patient is most at risk for which of the following
  complications?

ANSWER
a.- Orchitis.
b.- Testicular torsion.
c.- Testicular cancer.
d.- Testicular feminization.
CLINIC CASE

You are called to the nursery to evaluate o newborn
  infant. The infante is full-term but weighs only 2040
  grams. The liver is enlarged, and “blueberry muffin”
  spots are evident in the skin. You obtain computed
  tomography of the brain, which demonstrates
  periventricular calcifications. You suspect congenital
  infection with cytomegalovirus.
QUESTION No. 138
Which of the following is the most likely long-term
 complication of congenital CMV?

ANSWER
a.- Hearing loss.
b.- Keratoconjunctivitis.
c.- Hydrops fetalis.
b.- Cataracts and trachoma.
CLINIC CASE

A 46 year old male has had intermittent flares of
  ulcerative pan colitis for 10 years. He has increasing
  diarrhea, blood per rectum, and fever. He responds
  to medical management.
QUESTION No. 139
Proctocolectomy is indicated for?

ANSWER
a.- Severe dysplasia on initial biopsy.
b.- Persistent stricture.
c.- Long periods of persistent disease.
d.- Length of colon involved.
QUESTION No. 140
On routine rectal examination, a 2cm mass is felt.
  Biopsy is a carcinoid initial operative treatment is?

ANSWER
a.- Abdominal perineal resection.
b.- Fulguration.
c.- Low anterior signoid resection.
d.- Transanal excision.
QUESTION No. 141
A 60 year old female has a 2 cm hot thyroid nodule and
  exhibits signs and symptoms of thyrotoxicosis,
  treatment is?

ANSWER
a- Propythiouracil and FNA.
b.- f3- blocker and immediate thyroidectomy.
c.- Radioactive therapy.
d.- Potassium iodide SSKI
CLINIC CASE
A ventilated patient has a tidal volumen of 800 and a
  respiratory rate of 12. You want to lower the tidal
  volumen to 600 and maintain the same amount of
  ventilation.

QUESTION No. 142
What will be the respiratory rate?
ANSWER
a- 16.
b.- 14.
c.- 18.
d.- 12
CASO CLINICO

Se trata de paciente masculino de 23 años de edad el
  cual ingresa a urgencias presentando orificio de
  entrada en flanco derecho, por arma de fuego, sin
  orificio de salida, a la exploración física se observa
  con FC 125 lpm, llenado capilar lento, FR 31 rpm,
  se observa ansioso con tendencia a la confusión, su
  gasto urinario fue de 20 ml/h.
PREGUNTA No. 143
Considerando las manifestaciones clínicas observadas
  así como las variables vitales, en que estado de
  choque se encuentra el paciente?

RESPUESTA
a.- Clase I.
b.- Clase II.
c.- Clase III.
d.- Clase IV.
PREGUNTA No. 144
Cual es la conducta para mantener el volumen
  circulante mas adecuada?

RESPUESTA
a.- Solución salina 10 ml/kg/hora.
b.- Solución hartman 15ml/Kg/hora.
c.- Crioprecipitados.
d.- Concentrado eritrocitario y cristaloide.
CASO CLINICO

Una hora después, el paciente presenta los siguientes
  constantes vitales, FC 105 lpm, FR 21 rpm, gasto
  urinario de 35 ml/h, se encuentra ansioso, pero no
  confundido, la hemoglobina fue de 12 y
  hematocrito de 37, plaquetas de 120,000, asi como
  DHL 312, AST 435, tiempos de coagulación
  levemente incrementados.
PREGUNTA No. 145
Cual es la conducta a seguir en este momento?

RESPUESTA
a.- Mantener observación estrecha.
b.- Preparar para laparatomia exploratoria.
c.- Incrementar volumen circulante con paquete
   globular.
d.- Vasopresores, volumen y LAPE.
PREGUNTA No. 146
Se realizo LAPE con reparación hepática con buena
  respuesta, fue necesario crioprecipitados, 5
  unidades de plaquetas y 7 concentrados
  eritrocitarios, se administro 3 gramos de
  gluconato de calcio, cual es la razón de esta
  indicación?
RESPUESTA
a.- Alteración de coagulación.
b.- Mantener estable la membrana.
c.- Evitar arritmias.
d.- Disminuir la adhesividad plaquetaria.
CASO CLINICO
Se trata de paciente femenino de 48 años de edad, la cual
  fue ingresada posterior a accidente automovilístico y
  volcadura, era copiloto y fue la única sobreviviente, se
  observa confundida, desorientada, agitada, se presenta
  diaforesis, con palidez de mucosas, su llenado capilar
  lento, FC de 138 lpm, FR 34 rpm, TA 100/65 mmHg,
  minutos después se estableció un Glasgow de 8,
  distensión abdominal e inestabilidad toracicca, su gasto
  urinario fue de 20 ml/h, Campos pulmonares con matidez
  a la percusión bibasal, no hay ingurgitación yugular, pero
  hay equimosis importante sobre area del cinturón de
  seguridad, en la radiografia portátil se observo fractura
  de 5 costillas del lado izquierdo.
PREGUNTA No 147
Cual es la medida inmediata mas adecuada en este
  momento?

RESPUESTA
a.- Intubación endotraqueal.
b.- Solución hartman, vasopresores.
c.- Solución salina hipertónica.
d.- Sedación y relajación muscular.
PREGUNTA No 148
Posterior al decisión anterior se administra 3 paquetes de
  concentrados eritrocitarios, 3 paquetes de plasma fresco
  y 2 litro de solución hartman, sin embargo el gasto
  urinario es de 25 ml/hr, la tensión arterial se mantiene en
  95/60 mmHg durante la primera hora de estancia, cual es
  la conducta a seguir mas adecuada?

RESPUESTA
a.- Albumina y expansores.
b.- Infusion de dopamina 5 µg/Kg/min.
c.- Infusion de dobutamina 10 µg/kg/min.
d.- Crioprecipitados y gluconato de calcio.
PREGUNTA No 149
Cual es el objetivo mas importante de la conducta
  farmacológica tomada anteriormente, que no sea en
  detrimento de los mecanismos de reparacion
  inmediata?

REPUESTA
a.- Mantener el gasto urinario.
b.- Incrementar la fracción de eyección.
c.- Mantener la tensión arterial permisible.
d.- Favorecer el inotropismo.
PREGUNTA No 150
El paciente presenta estabilidad en la tensión arterial 95/70
   mmHg y gasto urinario de 35 ml/hr, se prepara para
   laparatoria exploratoria, tomando en cuenta la decisión
   farmacológica anterior, cual de los siguiente efectos no
   favorece al caso?

RESPUESTA
a.- Incremento > del 25 % sobre la tensión arterial basal.
b.- Aumento del flujo sanguíneo renal.
c.- Disminución de la resistencia vascular periférica.
d.- Disminución de la demanda de O2 del miocardio.
CLINIC CASE

Thre weeks following a nissen fundoplication, the
  patient has a severe episode of retching following a
  heavy meal and has forceful vomiting. Two hours
  later, he has severe epigastric pain and a spiking
  temperature of 102 F.
QUESTION No. 151
Which of the following is the problem?

ANSWER
a.- The nissen has slipped.
b.- Funduplication disrruption has ocurred with
   perforation.
c.- A perforated ulcer has developed.
d.- Acute pancreatitis.
QUESTION No. 152
Submucosal spread of malignant neoplasms is
  common and has major operative consequences
  in?

ANSWER
a.- Esophageal cancer.
b.- Rectal cancer.
c.- Small bowel cancer.
d.- Colon cancer.
CLINIC CASE
A 55 year old patient presents with dysphagia. A barium
  swallow identifies a smooth filling defect with the
  mucosa intact in the distal esophagus.

QUESTION No. 153
Appropiate tratment is?

ANWSER
a.- Total esophagectomy
b.- Distal esophagectomy.
c.- Enucleation of the lesion.
d.- Endoscopic drainage.
QUESTION No. 155
Tratment of barrett´s esophagus with a nessen
  fundoplication is indicated for?

ANSWER
a.- Halting the progression of the disease.
b.- Eliminating the risk of cancer.
c.- Elimination of the stricture.
d.- Resolution of the pathologic finding.
CLINIC CASE

A 75 year old female is evaluated for reflux
 esophagitis. At endoscopy the mucosa is salmon
 pink with a definite demarcation between normal
 and abnormal esophageal mucosa. Biopsy indicates
 no dysplasia?
QUESTION No. 156
Initial tratmente is?

ANSWER
a.- Esophageal resection.
b.- Primary medical tratment.
c.- Nissen fundoplication.
d.- Ablation of abnormal mucosa.
CASO CLINICO
Se trata de paciente masculino de 19 años de edad el
  cual ingresa a urgencias posterior a accidente
  automovilístico, refiere cefalea y dolor lumbar leve,
  permaneció en observación, se coloco catéter
  periférico 18 fr y solución glucosada al 5 % para 8
  hrs, a las 4 horas posterior al ingreso refiere
  ansiedad, sensación de mareo, a la EF se observa
  palidez de tegumentos, diaforesis, torax sin datos
  patológicos, abdomen con ruidos peristálticos
  disminuidos, SV con TA 100/70 mmHg, FC 105 lpm,
  FR 32 rpm, no se apresia compromiso oseo.
PREGUNTA No. 157
Se realizan realiza QS, BH, RX de torax y abdomen, Hb
  10.1, Hto 30, Rx de abdomen con niveles
  hidroaereos, imagen despulida, cual es la conducta a
  seguir más adecuada?

RESPUESTA
a.- Solución hartman 10 ml/kg/hora.
b.- Solución salina 0.45 % 10 ml/kg/hora.
c.- Solución mixta 10 ml/kg/hora.
d.- Solución glucosada al 10 % 10 ml/Kg/hora.
PREGUNTA No. 158
Posteriomente se administra 3 concetrados eritrocitarios y 2
  paquetes de plasma fresco, la tensión arterial se mantiene
  estable en 100/70 mmHg sin embargo la urésis fue de 40
  ml/hora, se observa auscencia de ruidos peristálticos, se
  observa estado de conciencia adecuado, alerta y tranquilo,
  cual no es un objetivo primordial en este momento del
  caso?

RESPUESTA
a.- Elevar la tensión arterial media por arriba de 95 mmHg.
b.- Manterne la hemostasia con gluconato de calcio.
c.- Estabilizar hemostasis por sospecha de hemorragia.
d.- Mantener un adecuado volumen urinario.
CLINIC CASE

A 62 year ol male presents with dyspepsia. An
  UGI shows thckening of the rugal folds. At
  upper endoscopy, no mass or lesion is
  identified. The gastric wall is thickened but
  no ulceration is noted.
QUESTION No. 159
Following endoscopic biopsy, this patient is best
  treated by?

ANSWER
a.- Radiation.
b.- Chemotherapy.
c.- Chemotherapy and radiation.
d.- Subtotal gastrectomy
CASO CLINICO
Mujer de 34 años sin antecedentes patológicos, no
  hábitos tóxicos, no alergias a fármacos conocidas.
  Presentó cefalea tensional en tratamiento con ácido
  acetilsalicílico con episodios de urticaria aguda en
  relación con el consumo de marisco, tratado y
  remitido hace 24 hrs. 4 horas después apareció
  dolor faringeo, tumefacción facial, de manos y
  brazos, con disminución de la diuresis. A las pocas
  horas de la hospitalización presentó una alteración
  hemodinámica con taquicardia sinusal a 140
  pulsaciones por minuto e hipotensión.
PREGUNTA No. 160
Considerando el cuadro clínico cual es la conducta
  inmediata a seguir, la paciente se encentra con
  vías de acceso venoso?

RESPUESTA
a.- Administración de volumen con cristaloides.
b.- Administración de aminas vasoactivas.
c.- Colocación de catéter largo.
d.- Administración de epinefrina.
PREGUNTA No. 161
Posteriormente se deterioró el nivel de conciencia y se
  produjo un paro respiratorio que precisó intubación y
  ventilación mecánica. Se consiguió la estabilización
  hemodinámica con dopamina y expansores de
  volumen, crioprecipitados y plasma fresco, cual es la
  complicación más probable?

RESPUESTA
a.- Coagulación por hemodilución.
b.- Sindrome compartimental.
c.- Coagulación intravascular por consumo.
d.- Coagulopatia dilucional.
PREGUNTA No. 162

Considerando el cuadro clínico y la evolución del caso,
  cual de las siguientes causas es la menos probable?

RESPUESTA
a.- Reacciones a fármacos.
b.- Reacciones a alimentos.
c.- Reacciones transfusionales.
d.- Reacciones a antígenos inhalados.
CASO CLINICO
Paciente varón de 57 años con larga evolución en
  cuidados intensivos por (SDMO) y (SRIS)
  secundarios a proceso postquirúrgico. Fue
  gastrectomizado y desarrolló una doble fístula
  pleural y digestiva. Ha tenido simultáneamente
  diversos procesos infecciosos sistémicos (empiema,
  peritonitis, herida quirúrgica y respiratoria) tratados
  mediante antibiograma. Además ha presentado
  repercusión orgánica persistente con necesidad de
  soporte vasopresor para control hemodinámico y
  respiratorio.
RESPUESTA No. 163
Considerando las características actuales que se
  observan en el paciente que tipo de choque se
  observa actualmente?

RESPUESTA
a.- Choque distributivo.
b.- Choque restrictivo.
c.- Choque séptico.
d.- Choque oculto.
RESPUESTA No. 164
El día 11 se introduce tratamiento con linezolid por IV,
    600mg/12h por aislamiento de Enterococcus faecium
    resistente a meticilina, sensible a linezolid, enoxaparina
    40mg/24h, propofol 300mg/h. A las 48 h del inicio de linezolid
    se observan crisis comiciales en relación con movimientos
    tonicoclónicos preferentemente de extremidades superiores y
    tronco sin llegar a generalizarse, transitorios. cual es su
    impresión?
RESPUESTA
a.- Choque anafiláctico.
b.- Reaccion adversa a medicamento.
c.- Choque séptico.
d.- Síndrome serotoninergico maligno.
RESPUESTA No. 165
Se observa también hiperreflexia, discreta acidosis metabólica y
    empeoramiento grave de la fiebre, leucocitosis, marcadores de
    inflamación y estado hemodinámico (hipotensión). Ante la
    sospecha de RAM se sustituye meperidino por fentanilo y se
    añade midazolam, 10mg/h, en perfusión continua al esquema
    de sedoanalgesia para la mejora del cuadro y noradrenalina
    para el soporte hemodinámico. Considerando las variable
    vitales, hipotensión refractaria cual de las siguientes medidas
    es mas adecuada?
RESPUESTA
a.- Incrementar aminas vasoactiva.
b.- Hemodialisis.
c.- Retirar linezolid y administrar metilprednizolona.
d.- Concentrado eritocitario y plasma fresco congelado
RESPUESTA No. 166
En los cultivos se aisló Candida spp. Se administro anfotericina B
    en dosis de 4mg/kg/día. Linezolid fue también suspendido y se
    indico daptomicina, 350mg/24h. Tras el cambio de tratamiento
    las mioclonías e hiperreflexia mejoraron antes de 24 h, hasta
    desaparecer por completo al segundo día. La fiebre,
    leucocitosis     e     hipotensión     mejoraron     también
    progresivamente, aunque la acidosis metabólica persistió. Cual
    es la complicación mas probable?
RESPUESTA
a.- Dudoso por acidosis metabolica.
b.- Coagulopatia por dilución.
c.- Alergia al fármaco.
d.- Falla organica multiple.
RESPUESTA No. 167
El diagnóstico de síndrome serotoninérgico suele ser difícil de
   realizar por la presencia de síntomas inespecíficos y comunes a
   otras entidades. Esta problemática puede ser todavía mayor
   en el caso de un paciente crítico y polimedicado como el que
   nos ocupa, donde los síntomas pueden ser atribuidos a
   múltiples factores de la enfermedad o del abordaje
   farmacológico. Cual es la base principal para diagnosticar este
   caso?

RESPUESTA
a.- Inicio brusco.
b.- Recuperación tras suspensión del fármaco.
c.- La presencia de fiebre, hiperreflexia y acidosis metabolica.
d.- Criterios diagnósticos de Sternbach y Dunkley.
CASO CLINICO
Se trata de un varón de 74 años que ingresó por fiebre y
  deterioro del estado general de meses de evolución. Como
  antecedentes personales, cabe destacar gastritis crónica
  atrófica, cardiopatía reumática con afectación aórtica y mitral
  y fibrilación auricular con tratamiento anticoagulante. En la
  exploración física no había alteraciones significativas. En la
  analítica, a reseñar una creatinina de 4,17 mg/dL no conocida
  y un INR de 7. En los hemocultivos realizados, crecieron en
  2/3 L. monocytogenes. En el estudio de la bacteriemia, se
  solicitó un ecocardiograma transtorácico que puso de
  manifiesto una verruga en la válvula aórtica, con fracción de
  eyección deprimida, KDOQ III, cual es la conducta
  farmacológica mas adecuada a seguir?
PREGUNTA No. 168
Cual es la conducta farmacologica de primera instancia
  es la mas adecuada?

RESPUESTA
a.- Administrar ampicilina y vancomicina.
b.- Administrar ampicilina y gentamicina ajustada.
c.- Administrar imipenem y amikacina.
d.- Administrar ceftriaxona e imipenem.
PREGUNTA No. 169
En la RMN craneal, se observaron múltiples infartos cerebrales, e
  hígado de estasis. La evolución fue mala, desarrollando
  insuficiencia cardiaca por lo que se decidió realizar cirugía
  cardiaca. En la intervención, se encontraron vegetaciones y
  abscesos que afectaban tanto a la válvula aórtica como a la
  mitral, presento hipotensión refractaria, sin respuesta a aminas
  vasopresoras, hipotermina, piel marmórea. Cual es el
  diagnostico mas probable en este momento del caso?

RESPUESTA
a.- Choque cardiogenico.
b.- Choque séptico.
c.- Choque restrictivo.
d.- Choque oculto.
CLINIC CASE

A 18 year old white female presents with
 nausea, vomiting, and RLQ pain. At
 exploration, the appendix is normal. However,
 there is inflammation and edema of the distal
 ileum involving the appendiceal stump.
QUESTION No. 170
The operation of choice is?

ANSWER
a.- Appendectomy.
b.- Exploration only.
c.- Ileocolectomy.
d.- Appendectomy and drainage.
CLINIC CASE
A 46 year old male has had intermittent flares of ulcerative
  pan colitis for 10 year. He has increasing diarrhea per
  rectum, and fever. He responds to medical management.

QUESTION No. 171
The operation of choice is?

ANWSER
a.- Length of colon involved.
b.- Long periods of persistente disease.
c.- Severe dysplasia on initial biopsy.
d.- Persistent stricture.
PREGUNTA No. 172
Paciente masculino de 21 años toxicomano, con datos de
  isquemia miocardica secundaria a consumo de crack
  (cocaina base), cual es el objetivo mas importante para
  disminuir la posibilidad de compromiso miocardico?

RESPUESTA
a.- Bajar la demanda de O2.
b.- Disminuir la resistencia periférica.
c.- Aumentar la capacitancia venosa.
d.- Cronotropismo negativo.
PREGUNTA No. 173
Se trata de paciente masculino de 67 años de edad con IAM
  de 3 horas de evolución con acinesia de apex, que se
  manejo con MONA inicialmente, se esta evaluando la
  posibilidad de reperfusión de rescarte, cual es el criterio
  mas adecuado para su reperfusion de rescate?

RESPUESTA
a.- Las alteraciones del sitio de acinesia.
b.- La actividad eléctrica alterada.
c.- La edad del paciente.
d.- Las caracteristicas de biomarcadores.
CLINIC CASE
A 21 year old male was diagnosed with right eye optic
   neuritis last month, with successful recovery of his vision
   after treatment. The patient has never han any other
   neurolical symptoms of wekness, numbness, etc. MRI
   brain showed 3 white matter lesions.
PREGUNTA No. 174
The next step of action?
RESPUESTA
a.- Methotrexate.
b.- Start oral steroids.
c.- No further treatment.
d.- Interferon beta 1ª.
CASO CLINICO
Se trata de paciente masculino que presenta trastorno
   mental por consumo de cocaina y marihuana desde hace
   mas de 10 años, ha presentado dos ingreso por
   sobredosis, durante el presente año.
PREGUNTA No. 175
Cual es el núcleo mas importante que se encuentra
   involucrado en la adicción de las sustancia?
RESPUESTAS
a.- Nucleo tecmental ventral.
b.- Nucleo accumbens.
c.- Nucleo ceruleo.
d.- Nucleo del rafe caudal.
CLINIC CASE
A 34 year old female was incidentally diagnoses whit a
  unruptured middle cerebral artery aneurism.

QUESTION No. 176
Which of the following is a known risk factor for cerebral
  aneurysms?

ANSWER
a.- Cigarette smoking.
b.- Diabetes.
c.- Hyperthyroidism.
d.- Hyperlipidemia.
CLINIC CASE
A 34 year old female was incidentally diagnoses whit a
  unruptured middle cerebral artery aneurism.

QUESTION No. 177
Which of the following is a known risk factor for cerebral
  aneurysms?

ANSWER
a.- Cigarette smoking.
b.- Diabetes.
c.- Hyperthyroidism.
d.- Hyperlipidemia.
CLINIC CASE
A 66 year old man with hypertension, dyslipidemia and
  two recent unevaluated TIA´S presented with an acute
  onset of right sided weakness. On neurological exam
  the patient was alert, fully oriented, had fluent
  language, normal repetition and naming. His pupils
  were equal, round and reactive, his face was
  symmetric, he had intact facial sensation, his hearing
  was grossly normal, there was no evidence of
  dysarthria, and his tongue was midline. Strength was
  normal in the LUE and LLE. Strength was 4/5 with drift
  in the right upper extremity.
CASO CLINICO
Se trata de paciente femenino de 67 años de edad, que acude
   a consulta por cansancio, fatiga, dificultad para respirar,
   dolor torácico que disminuye con reposo, disnea de
   moderados esfuerzos, edema de miembros inferiores hasta
   2/3 de la pierna, a la exploración física se observo palidez de
   piel y mucosas, llenado capilar lento, estertores crepitantes
   leves, presencia de galope y trill. La paciente es hipertensa
   desde hace 20 años con moderado apego a tratamiento con
   captopril 50 mg/dia, hidroclorotiazida 25 mg/12 hrs. Ha
   presentado 2 crisis hipertensivas, asi como un evento de
   insuficiencia mesentérica, sus laboratorios EGO proteínuria,
   glucosuria, uratos, densidad urinaria disminuida, colesterol
   289 mg/dl, triglicéridos 720 mg/dl, creatinina 6.2 mg/dl,
   BUM 29, kalemia de 5.3, los signos vitales fueron TA
   160/105 mmHg, FC 96, FR 28, IMC 32.
PREGUNTA No. 178
Se realizo un ecocardiograma donde se observa una
  FE de < 40 %, durante su ingreso la paciente
  presento volumen urinario de 20 ml/hr.
  Considerando las características actuales cual es la
  clase funcional que presenta el caso?

RESPUESTA
a.- Clase funcional I.
b.- Clase funcional II.
c.- Clase funcional III.
d.- Clase funcional IV.
PREGUNTA No. 179
Considerando las características actuales cual es el
  K/DOQI que presenta el paciente, cual es el método
  mas adecuado para determinar el estadio de la
  IRC?

RESPUESTA
a.- Depuracion de creatinina de 24 hrs.
b.- Biopsia renal.
c.- Ultrasonografia renal.
d.- Urografia escretora
PREGUNTA No. 180
Cual es la conducta farmacológica mas adecuada
  considerando el K/DOQI y la clase funcional?

RESPUESTA
a.- Verapamilo.
b.- Diltiazem.
c.- Telmisartan.
d.- Isosorbide.
PREGUNTA No. 181
Cual es la conducta inmediata a seguir para mejorar la
  precarga disminuyendo los niveles de potasio?

RESPUESTA
a.- Furosemide.
b.- Dialisis peritoneal.
c.- Hemodialisis.
d.- Soluciones metabolizadas.
PREGUNTA No. 182
Como esperaría encontrar al calcio y fosforo en el
  paciente en la condición actual?

RESPUESTA
a.- Calcio bajo, fosforo bajo.
b.- Calcio alto, fosforo alta.
c.- Calcio bajo, fosforo alta.
d.- Calcio alta, fosforo baja.
PREGUNTA No. 183
Que Cockroft-Gault, MDRD y K/DOQI presenta el
  paciente?

RESPUESTA
a.- Cockroft-Gault 284, MDRD 67 y K/DOQI (III).
b.- Cockroft-Gault 384, MDRD 57 y K/DOQI (IV).
c.- Cockroft-Gault 484, MDRD 47 y K/DOQI (V).
d.- Cockroft-Gault 184, MDRD 37 y K/DOQI (II).
PREGUNTA No. 184
Cual de las siguientes patologías es la principal causa
  de IRC?

RESPUESTA
a.- Diabetes mellitus.
b.- Hipertensión arterial.
c.- Glomerulonefritis.
d.- Enfermedad renal poliquística.
PREGUNTA No. 185
Cuales factores de riesgo aceleran el deterioro del
  funcionamiento renal en el caso actual?

RESPUESTA
a.- Dislipidemia.
b.- Edad avanzada.
c.- Diabetes mellitus.
d.- Anemia.
CASO CLINICO
Se trata de masculino de 59 años de edad, ingresa a urgencias debido
   a dificultad para respirar, cansancio, fatiga, tos productiva desde
   hace 15 dias intensificandoce la noche previa al ingreso actual,
   cuenta con antecedentes de EPOC diagnosticado hace 10 años,
   posterior a tabaquismo (una cajetilla diaria), con tratamiento de
   bromuro de iprapropio, salbutamol y betametasona, además fue
   diagnosticado como hipertenso hace 2 años, inicio con cambios del
   estilo de vida y dieta con restricción de sodio, hace un año se
   considero incluir captopril que fue cambiado 3 meses después por
   incremento de tos, desde entoces toma losartan, hidralazina. A la
   EF se observa facies pletórica con hiperemia conjuntival, mucosas
   orales cianóticas moderadamente, se ausculta un soplo carotideo
   derecho, ingurgitación yugular grado II, estertores subcrepitantes
   bibasales, con resonancia tipanica a la percusión, los ruidos
   cardiacos levemente disminuidos, rítmico, abdomen con
   hepatomegalia 5 cm debajo del borde costal.
PREGUNTA No. 186
Considerando el estado actual del paciente cual es la
  conducta mas apropiada a seguir, para establecer
  la función cardiopulmonar?

RESPUESTA
a.- Espirometria.
b.- Ecocardiograma.
c.- Valor de natriuretico.
d.- Radiografia de torax.
PREGUNTA No. 187
Se observo una FE < 40 %, indice cardiotoraccico patológico,
  exudados algodonosos bilaterales, trama brocovascular
  incrementada, gases con PCO 49, PO2 85, potasio 6.3, Factor
  natriuretico disminuido, a la EF se auscultan estertores,
  sibilancias, hiperresonancia, cual es la conducta mas
  apropiada para disminuir la precarga?

RESPUESTA
a.- Retirar diurético ahorrador de potasio por diurético de asa.
b.- Iniciar con calcioantagonista dihidropiridinico de acción
    rápida.
c.- Incrementar la capacitancia vascular periférica con nitritos.
d.- Restriccion hídrica, dieta libre de sodio, diurético de asa.
CASO CLINICO
Femenino de 37 años de edad la cual acude a consulta
  debido a que no ha logrado embarazarse, refiere que
  lleva 4 años de intentándolo, con vida sexual activa
  desde hace 5 años con la misma pareja, no usa método
  anticonceptivo, como antecedentes refiere menarca a
  los 19 años, gesta 0, para 0, abortos 0, a la exploración
  física se observa con caracteres sexuales secundarios
  adecuados, a la exploración GO sin secreción vaginal
  blanquecino sin olor desagradable, a la especuloscopia
  con normal, acude con resultados de Papanicolaou con
  datos inflamatorios crónico, colposcopia negativo para
  VPH y perfil hormonal normal (FSH, LH, P4, E2).
PREGUNTA NO. 188
Tomando en cuenta el perfil hormonal de la paciente que
  intervención en primera instancia tomaría usted?

RESPUESTA:
a.- Iniciar clomifeno 5 mg cada 24 hrs durante 14 días.
b.- Solicitar una espermatobioscopia.
c.- Realiza USG de anexos para buscar causa obstructiva.
d.- Indicar medroxiprogesterona 25mg y etinilestradiol
   2mg im
CASO CLINICO
Femenino de 31 años de edad la acude a consulta
  debido a que presenta amenorrea secundaria,
  agrega que desea embarazarse, como antecedentes
  cuenta con menarca a 21 años con oligomenorrea,
  se observa con clasificación de Taner grado 3, refiere
  presencia de leucorrea frecuente ha recibido
  tratamiento, a la exploración observa Talla 1.47 mts,
  Peso 48 kg. Además cuello alado, torax en escudo.
PREGUNTA No. 189
Cuál es la condición más frecuente de origen genético
  que presenta con los datos observados en este
  caso.

RESPUESTA:
a.- Sindrome de Kallman.
b.- Sindrome de Turner.
c.- Sindrome de Klinifelter.
c.- Sindrome XXX.
CASO CLINICO

Se trata de paciente femenino de 21 años de edad la
  cual acude por falta de menstruación durante los
  últimos 6 meses, niega vida sexual activa, sus
  antecedentes GO son menarca 11 años, G: 0, a la
  exploración se encuentra caracteres sexuales
  normales para edad y sexo, se observa sobrepeso
  por IMC, niega flujo, refiere que en los 3 últimos
  años sus periodos menstruales han sido irregulares,
  niega cefalea u otros síntomas.
PREGUNTA No. 190
Cuál de los siguientes diagnósticos es el más
  probable?

RESPUESTA
a.- Microadenoma PRL productor.
b.- Adenoma GH productor.
c.- Microadenoma ACTH productor.
d.- MEN.
CASO CLINICO
Se trata de femenino de 21 años de edad la cual acude
  a consulta debido a que refiere mastalgia y sensación
  de plenitud mamaria, irritabilidad, inestabilidad
  emocional, refiere que desde los 17 años se presento
  más intensos los síntomas, agrega dismenorrea
  ocacional con rechazo al periodo menstrual, al
  interrogatorio usted observa irregularidades
  menstruales, la paciente presenta un IMC de 18.
PREGUNTA No. 191
Cuál es la conducta más adecuada para este caso?

RESPUESTA
a.- La conducta mas adecuada es iniciar un esquema de
   contraceptivo oral.
b.- Iniciar un inhibidor selectivo de la recaptura de
   Serotonina.
c.- Indicar la calendarización de la sintomatología
   previa a la terapia.
d.- Derivar a la paciente al servicio de psiquiatría.
CASO CLINICO
Femenino de 41 años de edad. Entre sus antecedentes
  mencionó el diagnóstico de tuberculosis intestinal tres
  años antes, siendo tratada durante 8 meses con
  tratamiento     específico    tetra-asociado,     periodos
  menstruales regulares y un parto por cesárea a los 32
  años. Se hospitalizo por cuadro de varios años de
  evolución, con frecuentes episodios de dolor abdominal
  difuso, distensión abdominal, flatulencia, nauseas,
  vómitos y constipación, mayor durante ciclos
  menstruales. El cuadro que se intensifico durante el
  último año, asociándose esporádicamente deposiciones
  con estrías de sangre viva y perdida de 4 a 5 Kg de peso.
CASO CLINICO
Al ingreso, paciente en regulares condiciones
  generales, presión arterial 120/80 mmHg, frecuencia
  cardiaca 88 latidos por minuto, temperatura 36,6 ºC.
  El abdomen se encontraba distendido, timpanizado a
  la percusión, chapoteo en flanco y fosa iliaca derecha
  y borborismos generalizados a la auscultación. Los
  laboratorios, rayos X de tórax, abdomen y ecografía
  abdominal de ingreso fueron normales, exceptuando
  la sangre oculta en heces que fue (+). Las revisiones
  ginecológica y proctológica no encontraron
  alteraciones.
PREGUNTA No. 192
Cuál es el estadio en el que se encuentra la paciente?

RESPUESTA
a.- Estadio I (Mínimo) - 1-5 puntos.
b.- Estadio II (Leve) - 6-15 puntos.
c.- Estadio III (Moderada) - 16-40 puntos.
d.- Estadio IV (Severa) - más 40 puntos.
Primer simulador enarm 2013 primera parte final
Primer simulador enarm 2013 primera parte final
Primer simulador enarm 2013 primera parte final
Primer simulador enarm 2013 primera parte final
Primer simulador enarm 2013 primera parte final
Primer simulador enarm 2013 primera parte final
Primer simulador enarm 2013 primera parte final
Primer simulador enarm 2013 primera parte final
Primer simulador enarm 2013 primera parte final
Primer simulador enarm 2013 primera parte final
Primer simulador enarm 2013 primera parte final
Primer simulador enarm 2013 primera parte final
Primer simulador enarm 2013 primera parte final
Primer simulador enarm 2013 primera parte final
Primer simulador enarm 2013 primera parte final
Primer simulador enarm 2013 primera parte final
Primer simulador enarm 2013 primera parte final
Primer simulador enarm 2013 primera parte final
Primer simulador enarm 2013 primera parte final
Primer simulador enarm 2013 primera parte final
Primer simulador enarm 2013 primera parte final
Primer simulador enarm 2013 primera parte final
Primer simulador enarm 2013 primera parte final
Primer simulador enarm 2013 primera parte final
Primer simulador enarm 2013 primera parte final
Primer simulador enarm 2013 primera parte final
Primer simulador enarm 2013 primera parte final
Primer simulador enarm 2013 primera parte final
Primer simulador enarm 2013 primera parte final
Primer simulador enarm 2013 primera parte final
Primer simulador enarm 2013 primera parte final
Primer simulador enarm 2013 primera parte final
Primer simulador enarm 2013 primera parte final
Primer simulador enarm 2013 primera parte final
Primer simulador enarm 2013 primera parte final
Primer simulador enarm 2013 primera parte final
Primer simulador enarm 2013 primera parte final
Primer simulador enarm 2013 primera parte final
Primer simulador enarm 2013 primera parte final
Primer simulador enarm 2013 primera parte final
Primer simulador enarm 2013 primera parte final
Primer simulador enarm 2013 primera parte final
Primer simulador enarm 2013 primera parte final
Primer simulador enarm 2013 primera parte final
Primer simulador enarm 2013 primera parte final
Primer simulador enarm 2013 primera parte final
Primer simulador enarm 2013 primera parte final
Primer simulador enarm 2013 primera parte final
Primer simulador enarm 2013 primera parte final
Primer simulador enarm 2013 primera parte final
Primer simulador enarm 2013 primera parte final
Primer simulador enarm 2013 primera parte final
Primer simulador enarm 2013 primera parte final
Primer simulador enarm 2013 primera parte final
Primer simulador enarm 2013 primera parte final
Primer simulador enarm 2013 primera parte final
Primer simulador enarm 2013 primera parte final
Primer simulador enarm 2013 primera parte final
Primer simulador enarm 2013 primera parte final
Primer simulador enarm 2013 primera parte final
Primer simulador enarm 2013 primera parte final
Primer simulador enarm 2013 primera parte final
Primer simulador enarm 2013 primera parte final
Primer simulador enarm 2013 primera parte final
Primer simulador enarm 2013 primera parte final
Primer simulador enarm 2013 primera parte final
Primer simulador enarm 2013 primera parte final
Primer simulador enarm 2013 primera parte final
Primer simulador enarm 2013 primera parte final
Primer simulador enarm 2013 primera parte final
Primer simulador enarm 2013 primera parte final
Primer simulador enarm 2013 primera parte final
Primer simulador enarm 2013 primera parte final
Primer simulador enarm 2013 primera parte final
Primer simulador enarm 2013 primera parte final
Primer simulador enarm 2013 primera parte final
Primer simulador enarm 2013 primera parte final
Primer simulador enarm 2013 primera parte final
Primer simulador enarm 2013 primera parte final
Primer simulador enarm 2013 primera parte final
Primer simulador enarm 2013 primera parte final
Primer simulador enarm 2013 primera parte final
Primer simulador enarm 2013 primera parte final
Primer simulador enarm 2013 primera parte final
Primer simulador enarm 2013 primera parte final
Primer simulador enarm 2013 primera parte final
Primer simulador enarm 2013 primera parte final
Primer simulador enarm 2013 primera parte final
Primer simulador enarm 2013 primera parte final
Primer simulador enarm 2013 primera parte final
Primer simulador enarm 2013 primera parte final
Primer simulador enarm 2013 primera parte final
Primer simulador enarm 2013 primera parte final
Primer simulador enarm 2013 primera parte final
Primer simulador enarm 2013 primera parte final
Primer simulador enarm 2013 primera parte final
Primer simulador enarm 2013 primera parte final

More Related Content

What's hot

Clasificación de la Herniacion cerebral.
Clasificación de la Herniacion cerebral.Clasificación de la Herniacion cerebral.
Clasificación de la Herniacion cerebral.Nery Josué Perdomo
 
Hipertensión pulmonar
Hipertensión pulmonar Hipertensión pulmonar
Hipertensión pulmonar UACH, Valdivia
 
Fiebre reumatica e Insuficiencia cardiaca en Pediatría
Fiebre reumatica e Insuficiencia cardiaca en PediatríaFiebre reumatica e Insuficiencia cardiaca en Pediatría
Fiebre reumatica e Insuficiencia cardiaca en PediatríaDiana Jazmín Ojeda
 
Clasificación de killip y kimball
Clasificación de killip y kimballClasificación de killip y kimball
Clasificación de killip y kimballAndreaAle96
 
Iv.13. valoracion de la volemia
Iv.13. valoracion de la volemiaIv.13. valoracion de la volemia
Iv.13. valoracion de la volemiaBioCritic
 
Sìndromes de herniaciòn cerebral
Sìndromes de herniaciòn cerebralSìndromes de herniaciòn cerebral
Sìndromes de herniaciòn cerebralMonica Salgado
 
Respuesta metabolica al trauma
Respuesta metabolica al traumaRespuesta metabolica al trauma
Respuesta metabolica al traumaMetalixir
 
Angina Vasoespástica o de Prinzmetal
Angina Vasoespástica o de PrinzmetalAngina Vasoespástica o de Prinzmetal
Angina Vasoespástica o de PrinzmetalCardioTeca
 
Insuficiencia mitral clase Dahia
Insuficiencia mitral clase DahiaInsuficiencia mitral clase Dahia
Insuficiencia mitral clase DahiaDahiana Ibarrola
 
Sindrome Coronario Agudo Sin Elevacion del Segmento ST (SICA SESST)
Sindrome Coronario Agudo Sin Elevacion del Segmento ST (SICA SESST)Sindrome Coronario Agudo Sin Elevacion del Segmento ST (SICA SESST)
Sindrome Coronario Agudo Sin Elevacion del Segmento ST (SICA SESST)Ricardo Mora MD
 
Evaluacion cardiovascular prequirurgica ok
Evaluacion cardiovascular prequirurgica okEvaluacion cardiovascular prequirurgica ok
Evaluacion cardiovascular prequirurgica okeddynoy velasquez
 
Shock hipovolémico y shock séptico
Shock hipovolémico y shock sépticoShock hipovolémico y shock séptico
Shock hipovolémico y shock sépticolaylahamad94
 
Caso de actividad electrica sin pulso
Caso de actividad electrica sin pulsoCaso de actividad electrica sin pulso
Caso de actividad electrica sin pulsoIvette Urcuyo
 
Insuficiencia Mitral
Insuficiencia Mitral Insuficiencia Mitral
Insuficiencia Mitral Noe2468
 

What's hot (20)

Clasificación de la Herniacion cerebral.
Clasificación de la Herniacion cerebral.Clasificación de la Herniacion cerebral.
Clasificación de la Herniacion cerebral.
 
Hipertensión pulmonar
Hipertensión pulmonar Hipertensión pulmonar
Hipertensión pulmonar
 
Angiopatia amiloide cerebral.
Angiopatia amiloide cerebral.Angiopatia amiloide cerebral.
Angiopatia amiloide cerebral.
 
Trali y taco
Trali y tacoTrali y taco
Trali y taco
 
Craneo hipertensivo
Craneo hipertensivo Craneo hipertensivo
Craneo hipertensivo
 
Fiebre reumatica e Insuficiencia cardiaca en Pediatría
Fiebre reumatica e Insuficiencia cardiaca en PediatríaFiebre reumatica e Insuficiencia cardiaca en Pediatría
Fiebre reumatica e Insuficiencia cardiaca en Pediatría
 
Clasificación de killip y kimball
Clasificación de killip y kimballClasificación de killip y kimball
Clasificación de killip y kimball
 
Iv.13. valoracion de la volemia
Iv.13. valoracion de la volemiaIv.13. valoracion de la volemia
Iv.13. valoracion de la volemia
 
Sìndromes de herniaciòn cerebral
Sìndromes de herniaciòn cerebralSìndromes de herniaciòn cerebral
Sìndromes de herniaciòn cerebral
 
Respuesta metabolica al trauma
Respuesta metabolica al traumaRespuesta metabolica al trauma
Respuesta metabolica al trauma
 
ESTENOSIS DE PILORO PRESENTACION
ESTENOSIS DE PILORO PRESENTACIONESTENOSIS DE PILORO PRESENTACION
ESTENOSIS DE PILORO PRESENTACION
 
Angina Vasoespástica o de Prinzmetal
Angina Vasoespástica o de PrinzmetalAngina Vasoespástica o de Prinzmetal
Angina Vasoespástica o de Prinzmetal
 
ACV Hemorragico
ACV HemorragicoACV Hemorragico
ACV Hemorragico
 
Estenosis Aortica
Estenosis AorticaEstenosis Aortica
Estenosis Aortica
 
Insuficiencia mitral clase Dahia
Insuficiencia mitral clase DahiaInsuficiencia mitral clase Dahia
Insuficiencia mitral clase Dahia
 
Sindrome Coronario Agudo Sin Elevacion del Segmento ST (SICA SESST)
Sindrome Coronario Agudo Sin Elevacion del Segmento ST (SICA SESST)Sindrome Coronario Agudo Sin Elevacion del Segmento ST (SICA SESST)
Sindrome Coronario Agudo Sin Elevacion del Segmento ST (SICA SESST)
 
Evaluacion cardiovascular prequirurgica ok
Evaluacion cardiovascular prequirurgica okEvaluacion cardiovascular prequirurgica ok
Evaluacion cardiovascular prequirurgica ok
 
Shock hipovolémico y shock séptico
Shock hipovolémico y shock sépticoShock hipovolémico y shock séptico
Shock hipovolémico y shock séptico
 
Caso de actividad electrica sin pulso
Caso de actividad electrica sin pulsoCaso de actividad electrica sin pulso
Caso de actividad electrica sin pulso
 
Insuficiencia Mitral
Insuficiencia Mitral Insuficiencia Mitral
Insuficiencia Mitral
 

Viewers also liked

Enarm 2145 preguntas
Enarm 2145 preguntasEnarm 2145 preguntas
Enarm 2145 preguntas3lsanto
 
Banco preguntas espejo 2014
Banco preguntas espejo 2014Banco preguntas espejo 2014
Banco preguntas espejo 2014Andy Guarnizo
 
Primer simulador segunda parte curso enarm cmn siglo xxi final
Primer simulador segunda parte curso enarm cmn siglo xxi finalPrimer simulador segunda parte curso enarm cmn siglo xxi final
Primer simulador segunda parte curso enarm cmn siglo xxi finalPharmed Solutions Institute
 
Banco de-preguntas-medicina-interna
Banco de-preguntas-medicina-internaBanco de-preguntas-medicina-interna
Banco de-preguntas-medicina-internaGeek Advisor Freddy
 
1. Examen Nacional Xxix 2005
1. Examen Nacional Xxix 20051. Examen Nacional Xxix 2005
1. Examen Nacional Xxix 2005enarm
 
Resolución examen residentado 2016 26 de junio 2016
Resolución examen residentado 2016   26 de junio 2016Resolución examen residentado 2016   26 de junio 2016
Resolución examen residentado 2016 26 de junio 2016Villamedic Group
 
Preguntas y respuestas_razonadas_de_enfermeria
Preguntas y respuestas_razonadas_de_enfermeriaPreguntas y respuestas_razonadas_de_enfermeria
Preguntas y respuestas_razonadas_de_enfermeriaOverallhealth En Salud
 
18698273 1-enarm-2006-contestado-y-comentado
18698273 1-enarm-2006-contestado-y-comentado18698273 1-enarm-2006-contestado-y-comentado
18698273 1-enarm-2006-contestado-y-comentadoangiolikkia
 
Temario enarm 2013
Temario enarm 2013Temario enarm 2013
Temario enarm 2013Casosenarm
 
Guia para examen profesional
Guia para examen profesionalGuia para examen profesional
Guia para examen profesionalANaFAbB mts
 
Hipoglucemia curso enarm siglo xxi 36246001
Hipoglucemia curso enarm siglo xxi 36246001Hipoglucemia curso enarm siglo xxi 36246001
Hipoglucemia curso enarm siglo xxi 36246001emilio2005angel1973
 
LA DEPRESION ENEMIGO SILENCIOSO
LA DEPRESION ENEMIGO SILENCIOSOLA DEPRESION ENEMIGO SILENCIOSO
LA DEPRESION ENEMIGO SILENCIOSORicardo Mojica
 
Cam Xxv 2006 Cd3
Cam Xxv 2006 Cd3Cam Xxv 2006 Cd3
Cam Xxv 2006 Cd3enarm
 

Viewers also liked (20)

Enarm 2145 preguntas
Enarm 2145 preguntasEnarm 2145 preguntas
Enarm 2145 preguntas
 
Banco preguntas espejo 2014
Banco preguntas espejo 2014Banco preguntas espejo 2014
Banco preguntas espejo 2014
 
Examen RM 2014B
Examen RM 2014BExamen RM 2014B
Examen RM 2014B
 
ENARM Obstetricia ginecologia
ENARM Obstetricia ginecologiaENARM Obstetricia ginecologia
ENARM Obstetricia ginecologia
 
Primer simulador segunda parte curso enarm cmn siglo xxi final
Primer simulador segunda parte curso enarm cmn siglo xxi finalPrimer simulador segunda parte curso enarm cmn siglo xxi final
Primer simulador segunda parte curso enarm cmn siglo xxi final
 
Banco de-preguntas-medicina-interna
Banco de-preguntas-medicina-internaBanco de-preguntas-medicina-interna
Banco de-preguntas-medicina-interna
 
1. Examen Nacional Xxix 2005
1. Examen Nacional Xxix 20051. Examen Nacional Xxix 2005
1. Examen Nacional Xxix 2005
 
Depresión para enarm
Depresión para enarmDepresión para enarm
Depresión para enarm
 
SEGUNDO SIMULADOR (EJEMPLO)
SEGUNDO SIMULADOR (EJEMPLO)SEGUNDO SIMULADOR (EJEMPLO)
SEGUNDO SIMULADOR (EJEMPLO)
 
Resolución examen residentado 2016 26 de junio 2016
Resolución examen residentado 2016   26 de junio 2016Resolución examen residentado 2016   26 de junio 2016
Resolución examen residentado 2016 26 de junio 2016
 
Preguntas y respuestas_razonadas_de_enfermeria
Preguntas y respuestas_razonadas_de_enfermeriaPreguntas y respuestas_razonadas_de_enfermeria
Preguntas y respuestas_razonadas_de_enfermeria
 
18698273 1-enarm-2006-contestado-y-comentado
18698273 1-enarm-2006-contestado-y-comentado18698273 1-enarm-2006-contestado-y-comentado
18698273 1-enarm-2006-contestado-y-comentado
 
Temario enarm 2013
Temario enarm 2013Temario enarm 2013
Temario enarm 2013
 
Enarm 2014
Enarm 2014Enarm 2014
Enarm 2014
 
Guia para examen profesional
Guia para examen profesionalGuia para examen profesional
Guia para examen profesional
 
Hipoglucemia curso enarm siglo xxi 36246001
Hipoglucemia curso enarm siglo xxi 36246001Hipoglucemia curso enarm siglo xxi 36246001
Hipoglucemia curso enarm siglo xxi 36246001
 
LA DEPRESION ENEMIGO SILENCIOSO
LA DEPRESION ENEMIGO SILENCIOSOLA DEPRESION ENEMIGO SILENCIOSO
LA DEPRESION ENEMIGO SILENCIOSO
 
Esquizofrenia para enarm
Esquizofrenia para enarmEsquizofrenia para enarm
Esquizofrenia para enarm
 
Cam Xxv 2006 Cd3
Cam Xxv 2006 Cd3Cam Xxv 2006 Cd3
Cam Xxv 2006 Cd3
 
Preguntas enarm c.c.l.
Preguntas enarm c.c.l.Preguntas enarm c.c.l.
Preguntas enarm c.c.l.
 

Similar to Primer simulador enarm 2013 primera parte final

Seminarios y guia casos clinicos 2 era 2012 bioquimica[1]
Seminarios y guia casos clinicos  2 era 2012   bioquimica[1]Seminarios y guia casos clinicos  2 era 2012   bioquimica[1]
Seminarios y guia casos clinicos 2 era 2012 bioquimica[1]Grupos de Estudio de Medicina
 
Revisión de tema tce
Revisión de tema tceRevisión de tema tce
Revisión de tema tceSocundianeste
 
CASOS CLINICOS 2013.pdfjajajakakajajajajajdkfudufududud
CASOS CLINICOS 2013.pdfjajajakakajajajajajdkfudufudududCASOS CLINICOS 2013.pdfjajajakakajajajajajdkfudufududud
CASOS CLINICOS 2013.pdfjajajakakajajajajajdkfudufudududAxlGerardo
 
Parte 1 de 3 essalud 2017
Parte 1 de 3 essalud 2017Parte 1 de 3 essalud 2017
Parte 1 de 3 essalud 2017Bruno Alcalde
 
Actualidad medica 2012
Actualidad medica 2012Actualidad medica 2012
Actualidad medica 2012exforgehct
 
Banco+de+preguntas+de+semiología+endocrino
Banco+de+preguntas+de+semiología+endocrinoBanco+de+preguntas+de+semiología+endocrino
Banco+de+preguntas+de+semiología+endocrinoTheOnlyOne171190
 
CUIDADOS DE ENFERMERÍA AL PACIENTE CON NEUROPROTECCIÓN.pptx
CUIDADOS DE ENFERMERÍA AL PACIENTE CON NEUROPROTECCIÓN.pptxCUIDADOS DE ENFERMERÍA AL PACIENTE CON NEUROPROTECCIÓN.pptx
CUIDADOS DE ENFERMERÍA AL PACIENTE CON NEUROPROTECCIÓN.pptxAngel Ramos Mayhua
 
Meningitis caso clinico 1
Meningitis caso clinico 1Meningitis caso clinico 1
Meningitis caso clinico 1gaiaaaa
 
TEC PED.pptx
TEC PED.pptxTEC PED.pptx
TEC PED.pptxJoaoOroz
 
Examen Simulacro N⁰ 4 parte B.pdf
Examen Simulacro N⁰ 4 parte B.pdfExamen Simulacro N⁰ 4 parte B.pdf
Examen Simulacro N⁰ 4 parte B.pdfssuser601b67
 
highlights-acc-2012.pdf
highlights-acc-2012.pdfhighlights-acc-2012.pdf
highlights-acc-2012.pdfjosi diaz
 

Similar to Primer simulador enarm 2013 primera parte final (20)

Simulador segundo ejemplo
Simulador segundo ejemploSimulador segundo ejemplo
Simulador segundo ejemplo
 
Lesion traumatica cerebral en prdiatria
Lesion traumatica cerebral en prdiatriaLesion traumatica cerebral en prdiatria
Lesion traumatica cerebral en prdiatria
 
BQ guia casos clnicos 2 era 2012 bioquimica[1]
BQ guia casos clnicos  2 era 2012   bioquimica[1]BQ guia casos clnicos  2 era 2012   bioquimica[1]
BQ guia casos clnicos 2 era 2012 bioquimica[1]
 
Seminarios y guia casos clinicos 2 era 2012 bioquimica[1]
Seminarios y guia casos clinicos  2 era 2012   bioquimica[1]Seminarios y guia casos clinicos  2 era 2012   bioquimica[1]
Seminarios y guia casos clinicos 2 era 2012 bioquimica[1]
 
Manual endocrinologia
Manual endocrinologiaManual endocrinologia
Manual endocrinologia
 
ceaces1.pdf
ceaces1.pdfceaces1.pdf
ceaces1.pdf
 
Tec 2008
Tec 2008Tec 2008
Tec 2008
 
Revisión de tema tce
Revisión de tema tceRevisión de tema tce
Revisión de tema tce
 
CASOS CLINICOS 2013.pdfjajajakakajajajajajdkfudufududud
CASOS CLINICOS 2013.pdfjajajakakajajajajajdkfudufudududCASOS CLINICOS 2013.pdfjajajakakajajajajajdkfudufududud
CASOS CLINICOS 2013.pdfjajajakakajajajajajdkfudufududud
 
Parte 1 de 3 essalud 2017
Parte 1 de 3 essalud 2017Parte 1 de 3 essalud 2017
Parte 1 de 3 essalud 2017
 
1
11
1
 
Actualidad medica 2012
Actualidad medica 2012Actualidad medica 2012
Actualidad medica 2012
 
Banco+de+preguntas+de+semiología+endocrino
Banco+de+preguntas+de+semiología+endocrinoBanco+de+preguntas+de+semiología+endocrino
Banco+de+preguntas+de+semiología+endocrino
 
CUIDADOS DE ENFERMERÍA AL PACIENTE CON NEUROPROTECCIÓN.pptx
CUIDADOS DE ENFERMERÍA AL PACIENTE CON NEUROPROTECCIÓN.pptxCUIDADOS DE ENFERMERÍA AL PACIENTE CON NEUROPROTECCIÓN.pptx
CUIDADOS DE ENFERMERÍA AL PACIENTE CON NEUROPROTECCIÓN.pptx
 
ACV UNC 2022.pptx
ACV UNC 2022.pptxACV UNC 2022.pptx
ACV UNC 2022.pptx
 
Meningitis caso clinico 1
Meningitis caso clinico 1Meningitis caso clinico 1
Meningitis caso clinico 1
 
TEC PED.pptx
TEC PED.pptxTEC PED.pptx
TEC PED.pptx
 
Examen Simulacro N⁰ 4 parte B.pdf
Examen Simulacro N⁰ 4 parte B.pdfExamen Simulacro N⁰ 4 parte B.pdf
Examen Simulacro N⁰ 4 parte B.pdf
 
Resultados de la práctica clínica
Resultados de la práctica clínicaResultados de la práctica clínica
Resultados de la práctica clínica
 
highlights-acc-2012.pdf
highlights-acc-2012.pdfhighlights-acc-2012.pdf
highlights-acc-2012.pdf
 

More from Pharmed Solutions Institute (20)

Varicela
VaricelaVaricela
Varicela
 
Sarampion
SarampionSarampion
Sarampion
 
Rubeola
RubeolaRubeola
Rubeola
 
Quemadurasbuena
QuemadurasbuenaQuemadurasbuena
Quemadurasbuena
 
Parotidit
ParotiditParotidit
Parotidit
 
Meningitisss
MeningitisssMeningitisss
Meningitisss
 
Leucemias
LeucemiasLeucemias
Leucemias
 
Influenza
InfluenzaInfluenza
Influenza
 
Infecciones respi
Infecciones respiInfecciones respi
Infecciones respi
 
Neumonías
NeumoníasNeumonías
Neumonías
 
Hepatitis
HepatitisHepatitis
Hepatitis
 
Escarlatina
EscarlatinaEscarlatina
Escarlatina
 
Vivora
VivoraVivora
Vivora
 
Araña
ArañaAraña
Araña
 
Alacraan
AlacraanAlacraan
Alacraan
 
Parasitos
ParasitosParasitos
Parasitos
 
Diarrea
DiarreaDiarrea
Diarrea
 
Vih sida
Vih sidaVih sida
Vih sida
 
Tuberculosis
TuberculosisTuberculosis
Tuberculosis
 
Síndrome de sjogren
Síndrome de sjogrenSíndrome de sjogren
Síndrome de sjogren
 

Primer simulador enarm 2013 primera parte final

  • 1. Primer Simulador del Curso ENARM 2013 CMN SIGLO XXI
  • 2. Instrucciones 1. Preparar hoja de respuesta impresa. 2. Activar el simulador aplicando play. 3. Escribir nombre y hora de inicio en la hoja de respuesta. 4. Circular el reactivo de cada respuesta en hoja de respuesta. 5. El simulador se activará a las 8:00 hrs y se desactivará a las 12:00 hrs, la primera parte. 6. La segunda parte se activara a las 14:00 hrs y de desactivará a las 18:00 hrs.
  • 3. Caso clínico No. 58 (ejemplo) Femenino de 51 años, originaria de Campeche, acude control mensual por HAS y DM 10 y 5 años respectivamente, sus SV fueron TA 155/95 mmHg, FC 51 lpm, FR 21 rpm. Glucosa 210 mg/dl, hemoglobina glucosilada 6, urea 17, creatinina 1,1 mg/dl, refiere mareo ocasional, cefalea global acompañada de sueño y fatiga, frio y mareo ocasional, disnea de leves a moderados esfuerzos, se observa edema de miembros inferiores. Su tratamiento es captopril, metoprolol y glibenclamida, se realiza ECG (ver imagen).
  • 4. Caso clínico No. 58 (ejemplo)
  • 5. Caso clínico No. 58 (ejemplo) PRENGUNTA Se realiza ECG, considerando las manifestaciones que se han presentado, que conducta terapéutica considera más apropiada realizar de forma mediata? RESPUESTA a.- Incrementar los fármacos hasta meta de <120/80 mmHg. b.- Retirar metoprolol continuar con captopril y glibenclamida. c.- Mantener glibenclamida, retirar metoprolol e iniciar losartan. d.- Agregar hidroclorotiazida al manejo actual
  • 6. Hoja de respuestas d 51 a b c d 101 d 52 a b c d 102 d 53 a b c d 103 d 54 a b c d 104 d 55 a b c d 105 d 56 a b c d 106 d 57 a b c d 107 d 58 a b c d 108 d 59 a b c d 109 d 60 a b c d 110
  • 7. Caso Clínico • Femenino de 52 años, acude por cefalea occipital, que se incrementa con esfuerzos y estrés laboral, agrega mareo, intolerancia a ruidos e luz. AHF padre con IAM a los 40 años, madre con DM, HAS. APP tabaquismo, alcoholismo, vida sedentaria y obesidad grado II. AGO: eclampsia hace 30 años y anticonceptivos por 15 años. EF: TA 145/109, 150/110, 145/105. FC 93 lpm, FR 23. Palidez, ingurgitación yugular, ruidos respiratorios subcrepitantes, edema maleolar ++.
  • 8. PREGUNTA No. 1 ¿Cuál de los siguientes criterios es el más útil para para establecer el diagnostico? RESPUESTA a.- Cifras tensiónales altas en repetidas ocasiones. b.- Sospecha de alteraciones de órgano blanco. c.- Factores de riesgo presentes. d.- Alteraciones de estudios paraclinicos.
  • 9. PREGUNTA No. 2 Se realizaron estudios de laboratorio y gabinete con los siguientes datos: glucosa 116, creatinina 1.6, acido úrico 9, colesterol 256, triglicéridos 380, EGO: proteinuria +, glucosuria +. Cual es su conducta diagnostica mas adecuada a seguir? RESPUESTA a.- Realizar Rx de torax. b.- Realizar ECG. c.- Realizar ECCG. d.- Prueba de esfuerzo.
  • 10. PREGUNTA No. 3 Cual de los siguientes datos observados en la rx es mas probable? (ver imagen) RESPUESTA a.- Cardiomegalia global. b.- Hipertrofia ventricular derecha. c.- Edema agudo pulmonar. d.- Hipertrofia ventricular izquierda.
  • 11.
  • 12. PREGUNTA No. 4 Cual de los siguientes datos observados en el ECG es más probable? (ver imagen) RESPUESTA a.- Hipertrofia predominio septum. b.- Hipertrofia ventricular izquierda. c.- Hipertrofia ventricular derecha. d.- Hipertrofia auricular izquierda.
  • 13.
  • 14. PREGUNTA No. 5 Considerando el fondo de ojo cual es el estadio con la clasificación de Keith - Wagener - Barkeren el que se encuentra? (ver imagen) RESPUESTA a.- Grado I. b.- Grado II. c.- Grado III. d.- Grado IV.
  • 15.
  • 16. PREGUNTA No. 6 Considerando la clasificación JNC 7 en que estadio se encuentra el paciente? RESPUESTA a.- Normal. b.- Prehipertensión. c.- Hipertensión estadio I. d.- Hipertension estadio II.
  • 17. PREGUNTA No. 7 • De los factor de riesgo presentes cual es el más importante para el desarrollo de HAS en este caso? RESPUESTA a.- Eclampsia previa. b.- Uso de estrógenos. c.- Familiar con IAM. d.- Familiar con HAS.
  • 18. PREGUNTA No. 8 Cuál de los siguientes esquemas eligiría como inicial presenta mayor utilidad para el paciente? RESPUESTA a.- Losartan y hidroclorotiazida. b.- Metoprolol, hidroclorotiazida y nifedipina. c.- Captopril, hidroclorotiazida y metoprolol. d.- Lisonapril y amlodipino.
  • 19. PREGUNTA No. 9 Cuáles son los lineamientos para el manejo en tercer nivel de los pacientes con hipertensión arterial de difícil control en el primer nivel de atención, presente en este caso? RESPUESTA a.- Falta de recursos para continuar con el manejo. b.- Datos sugestivos de daño a órgano blanco. c.- Requerir más de 3 fármacos. d.- Presencia de crisis hipertensivas.
  • 20. PREGUNTA No. 10 ¿Cuáles son los cambios bioquímicos es más probables por los datos clínicos en el caso descrito? RESPUESTA a.- Aumento de angiotensina II. b.- Aumento de endotelinas. c.- Aumento de noradrenalina. d.- Disminución de tromboxano A2
  • 21. PREGUNTA No. 11 De los siguientes cambios funcionales cual es predominantemente menos dependiente de los factores fisiopatogenicos? RESPUESTA a.- Hiperactividad simpática. b.- Aumento del volumen sanguíneo. c.- Alteración del FNA. d.- Expansión del volumen intracelular.
  • 22. PREGUNTA No. 12 Cuáles de las siguientes medidas es menos adecuada para garantizar el adecuado control, vigilancia y seguimiento en este paciente? RESPUESTA a.- Toma de presión arterial al menos 2 veces al mes. b.- Envió oportuno a segundo nivel al diagnostico inicial. c.- Evaluación oftalmológica para manejo de retinopatía. d.- Realización de ecocardiograma al menos una vez al año.
  • 23. PREGUNTA No. 13 Cuál de los resultados de los índices o clasificaciones siguientes es más firme para la referencia del paciente antes descrito a segundo nivel de atención? RESPUESTA a.- Clasificación de keith, Wagener y Barker. b.- Índice de KDOQI. c.- Índice de Sokolow-Lyon. d.- ATP III.
  • 24. PREGUNTA No. 14 Cuál de las siguientes complicaciones es mas probable esperar en el caso considerando sus manifestaciones clínicas, daño a órgano blanco y paraclinicos observados? RESPUESTA a.- Infarto agudo al miocardio. b.- Accidente vascular cerebral. c.- Insuficiencia cardiaca congestiva. d.- Insuficiencia renal crónica.
  • 25. PREGUNTA No. 15 2 años después la paciente continuo presentando evolución maligna de la retinopatia hipertensiva, su gasto urinario fue de 45 ml/hr, creatinina 6, Urea 85, Cuál de los siguientes cambios histológicos renales es menos probable observar? RESPUESTA a.- Dilatación de los podocitos. b.- Esclerosis glomerular. c.- Necrosis fibrinoide glomerular. d.- Fibrosis intersticial
  • 26. PREGUNTA No. 16 Cuáles son las siguientes alteraciones es menos frecuentes que se presentan de la retina en la hipertensión arterial? RESPUESTA a.- Papiledema del nervio óptico. b.- Amaurosis. c.- Manchas algodonosas. d.- Hemorragias.
  • 27. PREGUNTA No. 17 The pharmacodynamic effect what more useful for nephroprotection in treating hypertension? ANSWER a. - Reabsorption of K+. b. - AGT inhibitors c. - AGT II antagonism. d. - Blocking calcium channels.
  • 28. PREGUNTA No. 18 What is the most useful strategy for the rehabilitation of the current case nephrological disorders? ANSWER a. - Maintain strict blood pressure levels. b. - Monitoring K/DOQI semester. c. - Reduced weight, lipids and snuff. d. - Decrease sympathetic nervous system activity.
  • 29. PREGUNTA No. 19 Cuál es la estrategia predictiva más útil para seguir la evolución de las alteraciones oftalmológicas del caso actual? RESPUESTA a.- Mantener los niveles de tensión arterial estricto. b.- Monitorización de K/DOQI semestral. c.- Reducción de peso, lípidos y tabaco. d.- Disminuir la actividad del sistema nervioso simpatico.
  • 30. CASE REPORT Male is 78 years old who attends to consultation by joint pain in his left knee during the scan you identify that your blood pressure are as follows, TA 3 times, 145/90, 155/90 and 150/91 mmHg, the patient denied symptoms associated with tensional figure. The patient has a history of a transient ischemic episode 6 months ago.
  • 31. PREGUNTA No. 20 What is action to take to handle this case? ANSWER a. - Weight reduction and salt. b. - Hydralazine 25 mg every 24 hrs. c. - Captopril and hydralazine. d. - Enalapril and hydrochlorothiazide.
  • 32. PREGUNTA No. 21 What is the purpose of the BMI for the treatment of the patient? ANSWER a. - Keep the body index between 20 and 25 kg / m2. b. - Maintain systolic blood pressure at least 140 mmHg. c. - Keep diastolic blood pressure less than 90 mmHg?. d. - Reduce the risk of MI or stroke.
  • 33. PREGUNTA No. 22 What is the treatment of choice for providing protection of the patient considering the history of transient cerebral ischemia? ANSWER a. - Enalapril 10 to 20 mg. b. - Losartan 50 mg c. - Hydrochlorothiazide 12.5 to 25mg. d. - Amlodipine 5 to 10 mg.
  • 34. PREGUNTA No. 23 The patient was evaluated by a cardiologist and returned for continuous control, within the measures submitted by the same monitor blood pressure was supine, sitting and standing. What is the reason for this measure. ANSWER a. - A measure that guarantees the figures. b. - Search orthostatic symptoms. c. - Check the treatment effect. d. - Does not have a demonstrable effect.
  • 35. CLINIC CASE A 47 year old white female with diabetes mellitus type II presents to the clinic with an erythematous rash in her axillae and groin of one month duration. Over the counter cortisone has failed to clear her rash. She denies using any deodorants or creams. PREGUNTA No. 24 What is the best treatment option for this diagnosis? ANSWER a.- Erythromcin. b.- Trimethoprim-sulfamethoxazole. c.- Ketoconazole cream. d.- Ciprofloxacin.
  • 36. CASO CLINICO Neonato de 11 días de vida, previamente sana, ingresada por dificultad respiratoria, mala coloración, rechazo de tomas y decaimiento. Constantes vitales; TA de 95/65 mmHg, FR de 160 lpm, FR de 90 rpm y saturación de oxígeno del 94% con FiO2 del 30%. Se auscultan ruidos crepitantes e hipoventilación basal izquierda. La radiografía de tórax evidencia infiltrado basal izquierdo.
  • 37. PREGUNTA No. 25 Cual es la conducta a seguir? RESPUESTA a.- Amoxicilina mas gentamicina. b.- Trimetoprim mas sulfametoxazol. c.- Cetriaxona. d.- Vancomicina.
  • 38. CASO CLINICO RN de 36 SDG obtenido por cesarea por desprendimiento de placenta. Al nacimiento presenta Apgar 8/9, peso en -0,89 DE, longitud en +1,75 DE, sin datos del perímetro cefálico. Con 48 horas de vida ingresada por ictericia en su hospital de referencia presenta hipoglucemias no cetósicas (glucemias entre 25-45mg/dl), insulinemia máxima registrada: 7μU/ml coincidiendo con glucemia de 33mg/dl.
  • 39. PREGUNTA No. 26 Cual es la conducta a seguir para identificar la causa ó antecedentes de la hipoglucemia en el paciente? RESPUESTA a.- Verificar Diabetes Mellitus en la madre. b.- Realizar escaneo abdominal. c.- Verificar funcionamiento hepático. d.- Verificar funcionamiento suprarrenal.
  • 40. CASO CLINICO Se trata de recién nacido de 3 horas de nacido, el cual nace prematuro, de madre adolecente con cervicovaginitis y parto distócico, inicia con taquipnea, dificultad respiratoria, con aleteo nasal, con disociación toraco abdominal, con mala tolerancia a la vía oral, con hipotermia, Los datos radiográficos de control se observo persistencia de imágenes radiológicas en ambos campos pulmonares tipo infiltrado, con síndrome de fuga, el recuento de leucocitos fueron predominio eosinofilos.
  • 41. PREGUNTA No. 27 Considerando las condiciones clínicas así como antecedentes patológicos, cual es el agente etiológico más probable? RESPUESTA a- Clanmydia b.- Klepsiela c.- Candida d.- Garnerella
  • 42. PREGUNTA No. 28 Cual es el tratamiento farmacológico en espera de los resultados de cultivo? a.- Ampicilina y amikacina. b.- Ampicilina y gentamicina. c.- Ceftriaxona y vancomicina. d.- Imipenem y Cefotaxima.
  • 43. CASO CLINICO Se trata de paciente de 35 semanas de gestación de 5 días de nacido, con ruptura prematura de membrana el cual requirió reanimación intensiva debido a pagar 2/6, se observo fiebre materna durante el trabajo de parto, la madre vive en medio rural, al nacer presento apnea, crisis convulsivas, vomito, distermia, taquicardia, disnea y deterioro, hiporreactivo con piel marmórea, distención abdominal, acude por flacidez, falta de respuesta, sin respuesta al medio externo.
  • 44. PREGUNTA No. 29 Cuales de los diagnósticos diferenciales es el menos frecuente? RESPUESTA a.- Síndrome de adaptación del recién nacido. b.- Taquicardia transitoria del recién nacido c.- Hemorragia ventricular. d.- Hipertiroidismo congenito.
  • 45. CASO CLINICO Se encuentra paciente de 42 años de edad el cual se conoce hipertenso con buen apego a tratamiento, tiene antecedentes de endarteritis de takayasu, refiere cefalea global, con acufenos y fosfenos, refiere amaurosis fugax y dolor retrosternal ardoroso y opresivo, se observo soplo carotidea, disminución de la fuerza muscular, disminución de la sensibilidad en de miembros, disminución de pulsos, con hipotermia periférica, se observa desviación de choque de punta, galope, sin de edema pulmonar e imagen de ensanchamiento mediastinal. Sus constantes vitales fueron TA 210/119 mmHg, FC 123 lpm, FR 31 rpm.
  • 46. PREGUNTA No 30 Cual de las siguientes complicaciones es la mas probable que se encuentra en este caso? RESPUESTA a.- Insuficiencias cardiaca aguda. b.- Disección de la aorta. c.- Bloqueo AV de tercer grado. d.- Encefalopatia hipertensiva
  • 47. CASO CLINICO Masculino de 51 años con VIH, hepatitis C y alcoholismo de 30 años. Consume un litro de vodka al día durante 2 meses. El alcohol en sangre era 269mg/dL, refiere náuseas, diarrea y dolor abdominal. Mala dentadura y eritema, atribuido a dermatitis seborreica crónica. Laboratorios e imágenes normales.
  • 48. PREGUNTA No. 31 Cuál es la conducta terapéutica más importante a seguir en este caso a su ingreso? RESPUESTA a.- Tiamina, ácido fólico y multivitaminas. b.- Diazepam 20 mg/c 8 hrs. IV. c.- Solución mixta 10 ml/kg/c 8 hrs. d.- Carbamacepina 800 a 1200 mg en 24 hrs.
  • 49. PREGUNTA No. 32 Presenta agitación, agresividad, desorientación, respuesta alterada a estímulos y se presentaron signos de inestabilidad autonómica. RESPUESTA a.- Diacepam 20 mg y haloperidol 5 mg. b.- Haloperidol 10 mg y levomepromacina 25 mg. c.- Diacepam 20 mg y loracepam 4 mg. d.- Diacepam 20 mg, haloperidol 10 mg y metoprolol 50 mg.
  • 50. PREGUNTA No 33 Los síntomas mentales y autonómicos con leve mejoría, se observa "eritema sin brillo, de color rojo oscuro con escamas sueltas en cabeza, cara, cuello, pecho y brazos, que comorbilidad más probable presenta? RESPUESTA a.- Delirium tremens con síndrome de mala absorción. b.- Delirium tremens y psicosis de korsakof. c.- Delirium tremens y encefalopatía de wernicke. d.- Delirium tremens y pelagra.
  • 51. PREGUNTA No 34 Cual es el tratamiento a largo plazo más adecuado? RESPUESTA a.- Ácido valproico 250 mg, niacina 150 mg, tiamina 100 mg y niacina 150 mg. b.- Lamotrigina 100 mg, niacina 100 mg, ácido fólic0 30 mg y hierro aminoquelado 0.800 mg c.- Ácido valproico 250 mg, niacina 100 mg y sulfiram 250 mg cada 8 hrs. d.- Carbamacepina 300 mg cada 8 hrs, niacina 150 mg, tiamina 100 mg y riboflavina 4 mg. .
  • 52. PREGUNTA No 35 Luego del alta, volvió a beber y no llevo el tratamiento. fue readmitido para desintoxicación, presento la erupción eritematosa fotodistribuida. Cuál es la complicación más frecuente en el SNC ? RESPUESTA a.- Daño pontino. b.- Daño cerebelar. c.- Daño frontotemporal. d.- Daño diecencefalico.
  • 53. PREGUNTA No 36 Ingresa a urgencias 2 años después, sin tratamiento, con alcoholismo activo, a la EF disneico, diaforético, confuso, delirante, ingurgitación yugular, tos con expectoración asalmolenada, estertores crepitantes, galope, obesidad central, reflujo hepatoyugular y edema de miembros inferiores. Cuál es la conducta diagnostica inmediata más adecuada? RESPUESTA a.- Cuantificación de CD4, carga viral de VHC. b.- Ultrasonografia hepática y vías biliares. c.- Ecocardiograma transesofagico. d.- Radiografía de torax.
  • 54. PREGUNTA No 37 Considerando los estudios realizados cual de los siguientes hallazgos nos proporciona una orientación diagnostica mas adecuada? RESPUESTA a.- 200 CD4 y 400 copias /ml. b.- Hepatomegalia, con aumento de ecogenicidad. c.- Fracción de eyección < 30%. d.- Evidencia de edema pulmonar.
  • 55. CASO CLINICO Mujer caucásica de 54 años de edad con dependencia del alcohol y dismotilidad esofágica se presentó a urgencias con diarrea, vómitos y pérdida de 15 kg de peso en seis meses, con estado mental alterado. En la TAC abdominal sugiere cirrosis. Por laboratorio se reporta anemia macrocítica, trombocitopenia, hipoalbuminemia y concentraciones elevadas de aspartato y alanina aminotransferasas.
  • 56. PREGUNTA No. 38 Cual es la conducta a seguir más adecuada? RESPUESTA a.- Reposición de líquidos y suplementos vitamínicos. b.- Diazepam 10 mg más haloperidol 10 a 20 mg. c.- Concentrado eritrocitario y plasma fresco. d.- Enema, neomicina y dieta hipoproteica.
  • 57. PREGUNTA No 39 Recibió reposición de líquidos y suplementos vitamínicos, altas dosis de diazepam debido a su estado alterado con agitación. 3 días después se observa temblor en reposo, ataxia prominente e incapacitante, cuál opción sería más adecuada para una aproximación diagnostica? RESPUESTA a.- Realizar BH, QS, EGO, PFH. b.- Realizar resonancia magnética. c.- Electroencefalograma. d.- Electromiografía y Potenciales evocados.
  • 58. PREGUNTA No 40 IRM con hiperintensidades, el EEG: ondas lentas difusas. A la EF somnolienta, orientada en persona, perseverante y desorganizada, discurso pobre y afecto lábil además diarrea persistente y glositis. Cual es la conducta farmacológica mas adecuada? RESPUESTA a.- Rivastigmina y memantina. b.- Niacina, tiamina, riboflamina, cianocobalamina. c.- Lamotrigina, acido valproico, quetiapina. d.- Ziprasidona, biperiden, clonacepam.
  • 59. CASO CLINICO Paciente masculino de 61 años de edad hipertenso mal controlado y diabetes mellitus, acude a urgencias por cefalea intensa, vomitos intenso, el paciente presenta datos compatibles con delirium caracterizado por agitación psicomotriz, presento vomito importante, su alteraciones neurologías no fueron focalizados y se presento lentamente, durante la exploración clínica presento disartria y hemiparesia, los signos vitales TA 205/120 mmHg.
  • 60. PREGUNTA No. 41 cual es la complicación mas probable que presento el paciente? RESPUESTA a.- Hemorragia cerebral. b.- Encefalopatia hipertensiva. c.- Edema pulmonar. d.- Meningoencefalitis.
  • 61. PREGUNTA No. 42 Cual es la meta sobre la hipertensión arterial en la primera hora? RESPUESTA a.- Disminuir 25 % de la diastólica. b.- Disminuir 50 % de la diastólica. c.- Disminuir 25 % de la sistólica. d.- Disminuir 50 % de la sistólica.
  • 62. PREGUNTA No. 43 Cual es tratamiento de primera elección en este caso? RESPUESTA a.- Betabloqueadores. b.- Nitroglicerina c.- Nitroprusiato. c.- Calcioantagonistas.
  • 63. PREGUNTA No. 44 Se agrego al tratamiento diurético al manejo, el paciente presenta rahs cutáneo en alas de mariposa, cual es la causa de la presencia de este síndrome de lupus Like? RESPUESTA a.- Hidroclorotiazida. b.- Clorotiacida. c.- Furosemide. d.- Hidralacina.
  • 64. CASO CLINICO Inicio con cefalea, visión borrosa, con dolor toraccico opresivo, el paciente es diabético e hipertenso con mal apego a tratamiento, la diatolica se observo en 115, refirió disminución de la agudeza visual, con necrosis fribinoide, con hemorragia de la retina, y exudados algodonosos, sus antecedentes fueron tabaquismo, anemia hemolítica, con cardiomegalia sin datos de edema pulmonar. Uremia, Anteriormente presento hemorragia cerebral hace 2 años.
  • 65. PREGUNTA No. 45 El paciente recibió previamente al traslado recibió dihidropiridina sublingual, cual es el tratamiento mas adecuado en este caso? TRATAMIENTO a.- Nitroprusiato de sodio. b.- Nitroglicerina. c.- Diurético de asa. d.- Dihidropiridina.
  • 66. PREGUNTA No. 46 La paciente respondió bien al cabo de una semana de tratamiento elegido, el delirio y ataxia habían mejorado notablemente, afecto agradable y reactividad apropiada, cual es el diagnostico actual con la respuesta observada? RESPUESTA a.- Encefalopatía de Wernicke. b.- Psicosis de korsakoff. c.- Deficiencia severa de niacina. d.- Trastorno orgánico cerebral.
  • 67. CASO CLINICO Hombre caucásico de 61 años de edad, alcohólico con cardiomiopatía dilatada, previo al ingreso bebió con mala alimentación durante 3 meses continuos. A su ingreso recibió clordiazepóxido, así como la tiamina y ácido fólico todos los días. 3 días después recibió 5 mg de haloperidol IM por agitación, continuó con desorientación fluctuante y tuvo un episodio de hipoxia de etiología incierta.
  • 68. PREGUNTA No. 47 El paciente presento saturación del 90 %, rx de torax normal, presentaba erupción eritematosa con escamas en su puente nasal y la hiperpigmentación de la piel en áreas expuestas al sol, desorientado en lugar y fecha con marcha atáxica, cual es la conducta a seguir mas adecuada? RESPUESTA a.- Nicotinamida oral 100 mg cada 8 hrs. b.- Neuroléptico bloqueador D2 tipicos. c.- Diazepam 10 mg cada 8 hrs. d.- Antipsicotico bloqueador D2 y 5Ht2a.
  • 69. PREGUNTA No. 48 El tratamiento de la abstinencia al alcohol es un grave problema debido a su alta morbilidad y mortalidad, los pacientes desarrollan delirio y reciben altas dosis de benzodiacepinas. Cual de las siguientes condiciones es menos frecuente? RESPUESTA a.- Deshidratación y alteraciones electrolíticas. b.- Trauma craneal y hematoma subdural crónico. c.- Pancreatitis aguda y/o crónica. d.- Deficiencia grave de niacina.
  • 70. PREGUNTA No. 49 La vitamina B3 en el metabolismo funciona como una coenzima o co-sustrato en una amplia gama de reacciones redox, cual es la forma principal con la que actúa? RESPUESTA a.- Hidrógeno nicotinamida adenina dinucleotido. b.- Nicotinamida adenina fosfato. b.- Nicotinamida adenina dinucleotido fostato de hidrogeno. c.- Hidrógeno nicotinamida dinucleotido.
  • 71. PREGUNTA No. 50 La niacina se encuentra naturalmente en forma de nicotinamida y acido nicotínico donde 1mg de cada uno representa NE (niacina uno). A cuantos días de deficiencia de niacina se pueden presentar su sintomatología? RESPUESTA a.- 30 días aproximadamente. b.- 60 días aproximadamente. c.- 90 días aproximadamente. d.- 120 días aproximadamente.
  • 72. PREGUNTA No. 51 Cual es la prevalencia de deficiencia de niacina en los pacientes con dependencia de alcohol con cuidados a largo plazo y que murieron durante la hospitalización por la deficiencia niacina con tratamiento potencialmente reversible? RESPUESTA a.- 10 a 20 %. b.- 20 a 30 %. c.- 30 a 40 %. d.- 40 a 50 %.
  • 73. CASO CLINICO • Femenino de 21 años de edad la cual acude a urgencias secundaria a perdida del estado de alerta, refiere acompañantes que se encontraba en clases cuando cayo de su propia altura sin causa aparente, a la exploración física se observa mucosas orales deshidratadas, palidez generalizada, piel reseca, cabello quebradizo y de caída fácil, observa un estadio tanner no correspondiente a la edad y al interrogatorio refiere pirosis, disfagia y vómitos ocasionales con trazas de sangre.
  • 74. PREGUNTA No. 52 Considerando su impresión diagnostica al ingreso a urgencias cual es su abordaje terapéutico? RESPUESTA a.- Inhibidor selectivo de la recaptura de serotonina. b.- Administración de glucosa al 50 %. c.- Administración de solución mixta. d.- Omeprazol 40 mg/12 hrs IV.
  • 75. PREGUNTA No 53 Considerando el padecimiento cual es la alteración hidroelectrolitico cronica mas probable? RESPUESTA a.- Hiponatremia. b.- Hipercalcemia. c.- Hiperkalcemia. d.- Hipokalemia.
  • 76. PREGUNTA No. 54 Cual es la incidencia y prevalencia de este trastorno Mental en mujeres? RESPUESTA a.- 8 por 100 000 y 0.3%. b.- 10 por 100 000 y 3.0%. c.- 15 por 100 000 y 2 %. d.- 12 por 100 000 y 1.5 %
  • 77. PREGUNTA No 55 Considerando el eje mas frecuentemente alterado en esta patología que estudios seria confirmatorio? RESPUESTA a.- Perfil tiroideo. b.- Perfil suprarenal. c.- Perfil gonadal. d.- Perfil de crecimiento.
  • 78. PREGUNTA No. 56 Las anomalías en las glándulas suprarrenales y la glándula tiroides generalmente se invierte a la rehabilitación nutricional, cual es la segunda complicación mas frecuente de larga duración? RESPUESTA a.- Baja estatura. b.- Infertilidad. c.- Osteoporosis. d.- Amenorrea
  • 79. PREGUNTA No. 57 La paciente egresa por mejoría es enviada a psiquiatría, donde recibe tratamiento farmacológico, cual de los siguiente esquemas es el mas adecuado para el caso? RESPUESTA a.- Amitriptilina 150 mg en 24 horas. b.- Fluvoxamina 100 a 300 mg en 24 hrs. c.- Fluoxetina 20 a 80 mg por día. d.- Velanfaxina 50 a 200 mg en 24 hrs.
  • 80. PREGUNTA No. 58 La paciente regresa 8 semanas después de iniciado el tratamiento, refiere ansiedad con agitación leve, estreñimiento, cefalea pero no ha presentado mejoría del cuadro inicial, cual es su conducta ? RESPUESTA a.- Ajusta dosis según K/DOQI. b.- Cambian a un ISRS sedante. c.- Mantiene el tratamiento actual. d.- Ingresa a para TEC.
  • 81. PREGUNTA No. 59 Ingresa urgencias con crisis convulsivas, perdida de control de esfínteres, hiperreflexia, hipertermia, TA 160 lpm, 60 rpm, palidez generalizada, ideas delirantes, pH 7.1, pCO2 31, pO2 84, gasto urinario < 30ml, enzimas hepáticas y cardiacas elevadas, presentando un síndrome serotoninergico maligno cual es la conducta mas adecuada sobre el fármaco especifico? RESPUESTA a.- Vía aérea, oxigenación y ventilación mecánica. b.- El carbón activado con sorbitol y lavado gástrico, c.- Medidas generales de soporte y sintomáticas. d.- Diuresis forzada ó diálisis y plasmaféresis.
  • 82. CASO CLINICO Masculino de 24 años, se encuentra en urgencias por referir inestabilidad emocional, taquicardia y opresión en el pecho, temor a ser secuestrado debido a sus conocimiento especiales, los familiares agregan que desde los 18 años dejo la escuela comenzó a comportarse extraño, se extravió 3 semanas, regresando sucio, mencionando que fue a buscar a dios para hablar con él, a la EF se observa falta de contacto visual, mutista selectivo, disperso, con rumiaciones, su constantes vitales se encuentran normales.
  • 83. PREGUNTA No 60 Cual es la conducta terapéutica de segunda elección mas adecuada? RESPUESTA a.- Haloperidol 10 mg y levomepromacina 25 mg IM. b.- Risperidona 2mg cada 12 hrs. c.- Olanzapina 5 mg cada 12 hrs. d.- Haloperidol 5 mg cada 12 hrs.
  • 84. PREGUNTA No 61 Se yugulan síntomas motivo de ingreso se realiza cambio en la terapéutica para mayor apego así para disminuir la conducta autística, cual seria el fármaco a elegir? RESPUESTA a.- Haloperidol 10 mg en 24 hrs VO. b.- Levomepromacina 25 en 24 hrs VO. c.- Quetiapina 200 mg cada 12 hrs VO. d.- Risperidona 3 mg cada 12 hrs VO.
  • 85. PREGUNTA No. 62 Cual es el diagnostico mas probable? RESPUESTA a.- Esquizofrenia paranoide. b.- Esquizofrenia hebefrenica. c.- Esquizofrenia desorganizada. d.- Esquizofrenia simple.
  • 86. PREGUNTA No 63 El paciente regresa en 6 meses con un nuevo cuadro psicótico, se ingresa a urgencias, el familiar refiere que el paciente no quiere tomar el medicamento por los efectos adversos, de las siguiente manejos coadyuvantes es el mas adecuado para contrarrestar los efectos indeseables? RESPUESTA a.- Difenidramina 25 mg. b.- Trihexifenidilo 5 mg cada 8 horas c.- Clonacepam 2 mg cada 24 hrs d.- Biperiden 25 mg cada 24 hrs.
  • 87. PREGUNTA No 64 Considerando la naturaleza del fármaco agregado cual de los siguientes manifestaciones no son propias de este grupo farmacológico? RESPUESTA a.- Pupilas dilatadas, piel y mucosa oral secas. b.- Elevación de la temperatura corporal. c.- Bradicardia o bloqueos. d.- Distensión abdominal y retención urinaria.
  • 88. PREGUNTA No 65 Cual de las siguientes no es una contraindicación del Biperideno? RESPUESTA a.- Hipertrofia prostática benigna. b.- Temblor esencial idiopático. c.- Síndrome de QT largo. d.- Enfermedad de alzheimer.
  • 89. CASO CLINICO Se trata de paciente femenino de 81 años que ingresa a urgencias por quemadura de segundo grado al cocinar en casa, la paciente llego 6 horas después ya que se encontraba sola en casa, a la EF se percibe olor a urea, descuidada, desalineada, se observa síntomas afásicos, apraxicos y amnésicos. (ver imagen)
  • 90.
  • 91. PREGUNTA No. 66 Cual es el diagnostico mas posible con la clínica y gabinete? RESPUESTA a.- Demencia por tautopatia b.- Enfermedad de pick. c.- Demencia de cuerpo de lewy d.- Demencia por enfermedad de pequeños vasos.
  • 92. PREGUNTA No 67 El paciente muestra signos de demencia leve a moderada, cuál de los siguientes manejos tiene mayor utilidad sobre la reserva del núcleo de meynert, reduciendo la actividad glutamatergica? RESPUESTA a.- Memantina. b.- Galantamina. c.- Rivastigmina. d.- Donepecilo.
  • 93. PREGUNTA No. 68 Se realiza resonancia magnética cual de las siguientes características no es frecuente de este tipo de demencia? RESPUESTA a.- Atrofia generalizada. b.- Atrofia de predominio frontal. c.- Lesiones de núcleos basales. d.- Atrofia periventricular.
  • 94. PREGUNTA No 69 Considerando las alteraciones fisiopatológicas cual de las siguientes respuestas no corresponde a la actividad de las ovillos neurofibrilares y las aplacas amiloideas? RESPUESTA a.- Neurotoxicidad mediada por glutamato. b.- Liberación de ROS y NOS. c.- Depleción de GABA y Glicina. d.- Liberación factores inflamatorios.
  • 95. PREGUNTA No. 70 Acude a consulta por infección urinaria, al interrogatorio se observa descuidado con irritabilidad actitud alucinada, con tendencia a la somnolencia, incongruente, ideas irracionales y agresividad, perdida del núcleo semántico, se diagnostica delirio, cual es el mejor abordaje? RESPUESTA a.- Olanzapina 5mg dosis respuesta. b.- Haloperidol 5 mg dosis respuesta. c.- Risperidona 2 mg dosis respuesta. d.- Clonacepam 2 mg dosis respuesta.
  • 96. CASO CLINICO Se trata de femenino de 19 años con tristeza, llanto fácil, ideas de minusvalia, acudió por insomnio, agresividad y falta de control de impulsos, la madre refiere que es muy cambiante, desde niña era muy problemática, aislada y poco flexible, al examen mental se observa renuente, no cooperadora, con tendencia a la irritabilidad, con llanto fácil, ideas suicidas y homicidas, con cambios de expresión frecuentes.
  • 97. PREGUNTA No. 71 Se indico un ISRS, a las 6 semanas, la paciente refiere estar bien, con energía, quiere regresar a la escuela, al trabajo. La madre refiere que duerme poco, esta muy activa, desaparecieron los síntomas depresivos, considerando la respuesta al tratamiento cual es la conducta mas adecuada a seguir? RESPUESTA a.- Mantener el tratamiento. b.- Retirar el tratamiento. c.- Cambiar el tratamiento. d.- Reducir el tratamiento.
  • 98. PREGUNTA No. 72 3 meses después la paciente refiere sentirse mejor pero algo aburrida, la madre refiere que aun tiene insomnio, cambios de conducta, nuevamente presenta llanto con irritabilidad e inestabilidad con bajo control de impulsos, cual es la conducta a seguir? RESPUESTA a.- Llevar a dosis mínima toxica de eutimizante. b.- Agregar un eutimizante de diferente grupo. c.- Agregar antidepresivo triciclico. d.- Agregar un agonista antagonista parcial D2.
  • 99. PREGUNTA No 73 Cual es la complicación psiquiátrica mas frecuente considerando la edad de inicio así como el sexo de la paciente? RESPUESTA a.- Suicidio. b.- Toxicomanías. c.- Ansiedad. d.- Trastorno de pánico.
  • 100. CASO CLINICO Masculino de 43 años de edad el cual acude a urgencias en la tarde, refiere dolor torácico, dificultad respiratoria, sudoración, temblor de miembros superiores, mareo y sensación de desmayo, estas manifestaciones se ha presentado en otros momentos, el paciente cuenta con antecedentes de importancia, como IAM el padre, DM en la madre, a la exploración física se observa obesidad grado III, leve palidez de tegumentos, SV 150/95 mmHg, FC 102 lpm, FR 41 rmp.
  • 101. PREGUNTA No. 74 Cual es la complicación psiquiátrica mas frecuente considerando la edad de inicio así como el sexo de la paciente? RESPUESTA a.- Suicidio. b.- Toxicomanías. c.- Ansiedad. d.- Trastorno de pánico.
  • 102. CASO CLINICO • Male is 78 years old which comes in for joint pain in his left knee during the scan you identify that your blood pressure are as follows, TA 80/170mmHg, repeat the pressure test and remains such, the patient denied symptoms associated with tension figure. The patient has a history of a transient ischemic episode 6 months ago.
  • 103. QUESTION No. 75 What is action to take to handle this case? ANSWER a. - Weight reduction and salt. b. - Hydralazine 25 mg every 24 hrs. c. - Captopril and hydralazine. d. - Enalapril and lisonipril.
  • 104. QUESTION No 76 What is the purpose of the BMI for the treatment of the patient? ANSWER a. - Keep the body index between 20 and 25 kg / m2. b. - Maintain systolic blood pressure at least 140 mmHg. c. - Keep diastolic blood pressure less than 90 mmHg?. d. - Reduce the risk of MI or stroke.
  • 105. QUESTION N0. 77 What is the treatment of choice considering pharmacological history of the patient? ANSWER a. - Enalapril 10 to 20 mg. b. - Losartan 50 mg c. - Hydrochlorothiazide 12.5 to 25mg. d. - Amlodipine.
  • 106. QUESTION No 78 The patient is currently stable with blood pressure of 120/80 mmHg, which of the following milestones must perform to shipping to third level? ANSWER a. - Perform QS, BH, EGO. b.- Laboratories - more routine CXR and EKG. c. - Seek first target organ damage. d. - It is not necessary as it was a recent find.
  • 107. CASO CLINICO Se trata de masculino de 22 años de edad el cual es ingresado al servicio de urgencia debido a la presencia de cefalea intensa, palpitación, sensación de muerte inminente, diaforesis, refiere que ya le había ocurrido esto anteriormente pero no tan intenso, su tensión arterial fue de 140/100 mmHg. FC 115, FR 32, se observa agitado con dolor precordial resto de exploración sin datos por agregar, laboratorios de rutina sin datos patológicos.
  • 108. PREGUNTA No 79 Considerando la presencia del cuadro clínico cual es la conducta a seguir mas adecuada? RESPUESTA a.- Evaluar metanefrinas /24 hrs. b.- Medicina nuclar con MIBG. c.- Prueba de supresión con clonidina. d.- Prueba de supresión con glucagon.
  • 109. PREGUNTA No. 80 Posterior a la realización del estudio anterior, se observo una imagen isointensa y continuaron los síntomas, cual es la conducta a seguir? RESPUESTA a.- Realizar búsqueda en cuello, torax y abdomen. b.- Identificar CAS/CARS c.- Supresion con Fluhidrocortisona. d.- Realizar urografía escretora.
  • 110. CASO CLINICO • Female patient is 73 years old which has 20 years of hypertension and diabetes mellitus, with medical treatment but poor adherence pharmacologic and nonpharmacologic, is currently with the following scheme 100mg/day hydrochlorothiazide, furosemide 20 mg / day, enalapril 60 mg / day, amlodipine 10 mg / day acetylsalicylic acid 100 mg / day, glipizide 20 mg in two divided doses, currently goes for occasional dizziness with falling height twice, has lost twice to disregard the people. Physical examination shows patient oriented, adequate alertness, with slight desihratacion with plethoric facies, observed 5 cm jugular venous distention, pulmonary rales fields predominantly in the bases, desdoblamiendo heard the second heart sound, mild hepatomegaly, lower limb edema + + +, which are hot. Vital signs: BP 105/80mmHg, HR 109 bpm, FR 31rpm. ECG is performed and shown tele- ray. Calculated an EF> 45%.
  • 111. QUESTION No. 81 What are the clinical manifestations specific for diagnosis of the disease? ANSWER a. - Orthopnea, nocturnal dyspnea and cough. b. - Edema, fatigue and chest congestion. c. - Electrocardiogram and chest radiograph. d. - Jugular venous distention, hepatomegaly and ejection fraction.
  • 112. QUESTION No. 82 What is the risk factor that can lead to an unbalanced state in this case? ANSWER a. - Failure of treatment. b. - OSA. c. - Arrhythmia, anemia or bleeding. d. - Class antiarrhythmic Ia and Ic.
  • 113. QUESTION No 83 What are the clinical features most useful to perform a differential diagnosis of diastolic HF vs systolic HF? ANSWER a. - There are distinguishable. b. - Dyspnea and edema. c. - Tachycardia and tachypnea. d. - Hepatic congestion.
  • 114. QUESTION No. 84 What are the most useful diagnostic aids that have not been performed in patients to identify complications of the disease? ANSWER a.- Liver function test. b. - Serum electrolytes. c. - Stress tests. d. - Echocardiogram.
  • 115. QUESTION No. 85 What is the measure predominantly pharmacological intervene on neurohumoral response in sodium and fluid retention? ANSWER a. - Renin-angiotensin-aldosterone system. b. - Arginine vasopressin system. c. - Release of atrial natriuretic peptide. d. - Inhibits transport system Na + Cl -.
  • 116. QUESTION No 86 Before defining the pharmacological treatment of reversible causes which less likely is the case? ANSWER a. - Alcohol-induced myocardial depression. b. - Nonsteroidal anti-inflammatory. c. - State of elevated afterload. d. - Using calcium channel blocker.
  • 117. QUESTION No. 87 Which of neurohumoral changes that occur in heart failure is less likely in this case? ANSWER a. - Increased myocardial contractility. b. - increased heart rate. c. - Increase in venous tone. d. - Decreased effective central blood volume.
  • 118. CASO CLINICO Ingresa paciente de 39 años de edad el cual ingresa a urgencias 4 horas antes, por presencia súbita de dolor retrosternal acompañado de presión toracicca “sensación de mucho peso sobre su pecho” que se presento en la madrugada, nausea, dificultad para respirar, refiere que la noche anterior comió abundante y consumió alcohol hasta la embriaguez, al ingreso se observa confuso, diaforético con palidez y cianosis distal, ingurgitación yugular, se ausculta estertores cripitantes bibasales, tercer ruido, TA 90/60 mmHg, FC 132 lpm, FR 36 rpm, Temp. 36.3 grados. Gasto urinario de < 20 ml/h, se observaron los siguientes estudios
  • 119. PREGUNTA No. 88 Cual es la clase funcional de Kllip que presenta el caso? RESPUESTA a.- Clase funcional Killip I. b.- Clase funcional Killip II. c.- Clase funcional Killip III. d.- Clase funcional Killip IV.
  • 120. PREGUNTA No. 89 Cual es el pronostico para la vida en este caso considerando la clase funcional? a.- Tasa de mortalidad del 6%. b.- Tasa de mortalidad del 17%. c.- Tasa de mortalidad del 38%. d.- Tasa de mortalidad del 81%
  • 121. PREGUNTA No. 90 Se calculo los siguientes resultados (LC) de 2 L/m2/min. Considerando los elementos clínicos y el (LC), en la curva de starling cual es el diagnostico? RESPUESTA a.- Insuficiencia cardiaca. b.- Edema agudo pulmonar. c.- Choque cardiogenico. d.- Choque distributivo.
  • 122. PREGUNTA No. 91 Se realizó ecocardiograma transesofagico de urgencias, donde se reporta FE < 30%, cual es % de mortalidad a 6 meses del paciente? RESPUESTA a.- del 15 al 20 % b.- del 20 al 25 % c.- del 25 al 30 % d.- 30 al 35 %
  • 123. CASO CLINICO Mujer de 70 años de edad, hipertensa, diabética de reciente diagnostico durante un examen de rutina y cursando con bacteriuria asintomatica, procuró atención médica por tos seca, disnea, inapetencia y desánimo. La paciente sabía que era portadora de hipertensión arterial desde la edad de 37 años, de diabetes mellitus desde los 56 años y de hipertrigliceridemia. A los 56 años de edad, se quejó de palpitaciones taquicárdicas con duración de 20 minutos, sin síncope. Hacía uso de 75 mg de captopril, 50 mg de clortalidona, 600 mg de quinidina y 0,25 mg de digoxina. El examen físico reveló peso de 54 kg, altura de 1,49 m, presión arterial de 170/110 mmHg. En el examen físico, fue identificado soplo sistólico +/4+ en área mitral. Lo restante del examen físico no reveló alteraciones. El ECG mostró ritmo sinusal, sobrecarga ventricular izquierda, infradesnivel de ST en V5 y V6 y presencia de onda U.
  • 124. PREGUNTA No. 92 Cual de los siguientes fármacos tiene mayor evidencia de efecto cardioprotector en esta condición del paciente? RESPUESTA a.- IECA´s b.- ARA II. c.- Digoxina. d.- Clortalidona.
  • 125. CASO CLINICO Hombre de 62 años con HTA, IAM y DM tipo 2 diagnosticada. Es ingresado por pielonefritis con IR aguda (MDRD 21 ml/min), interpretándose en el contexto de la propia infección, agravada por el consumo de antiinflamatorios no esteroideos (AINE) y persistiendo al ser dado de alta, es remitido a la consulta de nefrología presentando IR grado 4 (MDRD 13,83 ml/min), proteinuria de 5 g/24 h y microhematuria persistente (urocultivos negativos y citología negativa para malignidad). En el resto de la analítica destaca: ANA, ANCA, anti-MBG negativos. Inmunoglobulinas, cadenas ligeras, complemento y proteinograma normales. Serologías VHC, VHB, VIH negativas. Ecografía abdominal: riñones de tamaño normal con cortical moderadamente ecogénica y sin evidencia de dilatación de la vía excretora. Fondo de ojo normal.
  • 126. PREGUNTA No. 93 Cuál es la conducta más adecuada a seguir en el caso. RESPUESTA a.- IECA´s b.- ARA II. c.- Dialisis. d.- Hemodialisis.
  • 127. CASO CLINICO Se trata de masculino de 46 años de edad el cual acude a consulta debido a que ha presentado un proceso infecciosos urinario ya tratado, refiere mejoría completa posterior a una semana de tratamiento antibiótico, a la exploración identifica un perímetro abdominal de 102 cms debido a ello realiza una glucosa periférica donde se obtiene 130 mg/dl.
  • 128. PREGUNTA No. 94 Cual es la conducta a seguir para sustentar el diagnostico? RESPUESTA a.- Curva de tolerancia a la glucosa. b.- Glucosa en ayuno. c.- Determinar hemoglobina glucosilada. d.- Glucosa pospandrial.
  • 129. CASO CLINICO Ingresa a urgencias masculino de 21 años de edad el cual se observa ansioso, agitado con actitud alucinada, a la exploración se encuentra diaforético con palidez generalizada, súbitamente el paciente pierde el estado de alerta y presenta parada cardiaca, se activa el código de alerta e inicia compresiones torácicas, los familiares solo refieren que lo golpearon en una fiesta, 10 minutos después presento los síntomas. Se observo el siguiente trazo del ECG. No hay respuesta, no se detecta pulso, ni constantes vitales. Se descarga con 360 j, se mantiene soporte por 20 minutos más.
  • 130. PREGUNTA No. 95 No se percibe pulso pero se observa el siguiente trazo, cual de las siguientes causas es la mas probable en este caso? RESPUESTA a.- Neumotorax a tensión. b.- Sobredosis por drogas. c.- Tromboembolismo pulmonar. d.- Hipovolemia.
  • 131. CASO CLINICO • Se trata de femenino de 28 años de edad que acude a planificación familiar para solicitar método, cuenta con antecedentes de importancia tabaquismo positivo, ingesta de alcohol social una vez al mes por lo menos, desconoce antecedentes de cáncer en la familia, refiere que tiene relaciones sexuales con su esposo de 3 a 4 veces por semana desde hace 3 meses que se caso, es su primera pareja sexual desde hace 5 años.
  • 132. PREGUNTA No. 96 Cuál es el método más apropiado para esta paciente? RESPUESTA a.- Inyectable mensual. b.- Hormonales oral. c.- Dispositivo intrauterino. d.- Implante subdermico.
  • 133. CASO CLINICO Paciente femenino de 28 años de edad la cual se encuentra en pos-aborto de 5 dias, refiere que ya no presenta sangrado, acude a solicitar método anticonceptico, refiere que no quiere tener mas hijos por lo menos en 3 años, tiene 2 hijos, anteriormente empleaba hormonales orales y durante este método se embarazo, no cuenta con antecedentes de cervicovaginitis ni EPI.
  • 134. PREGUNTA No. 97 Cual de los siguientes métodos de planificación familiar es el mas adecuado para este caso? RESPUESTA a.- Hormonal inyectado. b.- Implante hormonal. c.- Preservativo. d.- Dispositivo intrauterino.
  • 135. PREGUNTA No. 98 La paciente acude ha consulta por cambios del flujo menstrual, el cual refiere es obscuro y ha presentado manchado intermenstrual, a la exploración se observa moco cervical con sangre moderada, cual es la conducta a seguir? RESPUESTA a.- Retirar el DIU. b.- Indicar un AINES. c.- Indicar antibiótico. d.- Indicar DOC.
  • 136. PREGUNTA No. 99 La paciente acude 6 meses después por dolor abdominal en fosa iliaca izquierda que se irradia a hipocondrio y hacia tras, a la palpación se presenta dolor abdominal de predominio pélvico, acude además porque su DOC presento cambios inflamatorios inespecíficos, cual es la conducta a seguir por en este caso? RESPUESTA a.- Retirar DIU por probable EPI. b.- Retirar DIU por resultado de DOC. c.- Retirar DIU por probable embarazo. d.- Retirar DIU por cambio de método.
  • 137. CASO CLINICO Se trata de paciente masculino RN de 60 minutos, obtenido por cesarea iterativa, demás de presentar sufrimiento fetal, por disminución de actividad intrauterina y disminuciones de FC de hasta 110 lpm, se observa con movimientos muy finos, al inicio presento cianosis leve y breve principalmente periungeal y oral, se apreciaron secreciones orales abundantes, a la exploración física se auscultan estertores bilaterales normales, peristaltismo poco audible y temperatura de 36.1 grado, se apreció expulsión de secreciones con tinte meconial durante la exploración.
  • 138. PREGUNTA No. 100 Considerando el cuadro clínico cual es su conducta a seguir? RESPUESTA a.- Mantener al paciente en observación continua. b.- Buscar posibles patologías ocultas. c.- Realizar radiografia de torax. d.- No hay datos patológicos son adaptativo.
  • 139. PREGUNTA No. 101 Cual es la conducta terapéutica menos apropiada en este momento del caso? RESPUESTA a.- Mantener ayuno hasta identificar causa. b.- Colocar en ambiente neutro. c.- Administrar oxigeno por casco cefálico 40 %. d.- Gasometria y oximetría de pulso.
  • 140. PREGUNTA No. 102 Cual de los siguientes parámetros gasométrico no es normal esperarlo en el caso? RESPUESTA a.- pH < 7.34. b.- PaCO > 45 mmHg. c.- HCO3 > 40mEq/L. d.- SpO2 < 95.
  • 141. PREGUNTA No. 103 Se realizaron electrolitos séricos por continuar con rechazo al alimento y secreción moderada oral, todos los resultados resultaron dentro de parámetro normales, cual de las siguientes diagnósticos diferencias del acidosis respiratoria es el mas probable presentarse? RESPUESTA a.- Ventilación inadecuada alveolar. b.- Desordenes musculares. c.- Defectos pulmonares. d.- Trastornos de las vías aéreas.
  • 142. PREGUNTA No. 104 Considerando los valores gasométricos y de electrolitos, cual es la causa de la modificación del anion gap, del caso? RESPUESTA a.- Error de laboratorio. b.- Toxinas no identificadas. c.- Mielomas. d.- Exceso de soluciones.
  • 143. CASO CLINICO Se trata de paciente masculino de 15 horas de vida extrauterina, el paciente nació vía cesárea, nació a la 35 semanas de gestación, se administro maduradores a la madre debido a trabajo de parto distócico con ruptura de membranas 12 horas antes de la intervención quirúrgica, a la exploración física se observo taquipnea de 101 rpm, y signo moderados de dificultad respiratoria.
  • 144. PREGUNTA No. 105 Considerando un probable diagnostico clínico realizado, cual de los diagnósticos diferenciales es menos probable encontrar? RESPUESTA a.- Síndrome de adaptación pulmonar. b.- Síndrome de aspiración de meconio. c.- Enfermedad de membrana hialina. d.- Neumonía neonatal hospitalaria.
  • 145. PREGUNTA No. 106 Se decide realizar radiografía de tórax, cuales son los datos no es probable espera encontrar para confirmar el diagnostico? RESPUESTA a.- Broncograma aéreo. b.- Hilio congestivo. c.- Derrame cisurales. d.- Sobredistencion pulmonar.
  • 146. PREGUNTA No. 107 Cuales son los factores de riesgo inducido para la taquipnea transitoria del recién nacido, cual es la mas probable en el caso clínico? RESPUESA a.- Sexo masculino b.- Grande. c.- Cesárea. d.- Sedación materna.
  • 147. PREGUNTA No. 108 Cual es el fenómeno fisiopatológico no es la mas adecuada para el caso? RESPUESTA a.- Falta de absorción de líquido amniótico. b.- Falta administración de esteroides. c.- Deficiencia ligera de surfactante. d.- Edema pulmonar transitorio.
  • 148. PREGUNTA No. 109 Considerando el diagnostico actual, el cual muestra signos de dificultad respiratoria, resultados de laboratorio normales al momento, FiO2 < 0.40 descartando los diagnósticos diferencias, cuál es la conducta a seguir en este caso es menos adecuada? RESPUESTA a.- Monitoreo de signos vitales. b.- Monitoreo de gases por punción cada 6 horas. c.- Mantener glucosa y electrolitos. d.- O2 por casco al 70 %
  • 149. PREGUNTA No. 110 Tomando en cuenta la evolución del caso, cual es su pronostico? RESPUESTA a.- Bueno. c.- Malo. b.- Fatal. d.- Excelente.
  • 150. PREGUNTA No. 111 Considerando un probable diagnostico clínico realizado, cual de los diagnósticos diferenciales es menos probable encontrar? RESPUESTA a.- Síndrome de adaptación pulmonar. b.- Síndrome de aspiración de meconio. c.- Enfermedad de membrana hialina. d.- Neumonía neonatal hospitalaria.
  • 151. CASO CLINICO Un RN de 3,6 kg nació a las 37 semanas de gestación hijo de diabética con un agente hipoglicémico oral. Obtenido por cesárea electiva con Apgar de 6/9. Desarrollo taquipnea inmediatamente después de su nacimiento y requirió oxígeno suplementario. En la gasometria presento; fueron PO2 de 57 mm Hg, PCO2 de 52 mm Hg, y pH de 7,31. El niño se mantuvo en oxígeno por campana. A las 2 horas el paciente se encuentra hipotónico, cianótico, con saturación del 70 %. La Rx de observa rectificación, hiperclaridad e incremento de los espacios intercostales y congestión parahiliar.
  • 152. PREGUNTA No. 112 Cual es la medida inmediata a seguir? RESPUESTA a.- Intubación orotraqueal. b.- Alimentación por sonda orogastrica. c.- Mantener un ambiente neutro. d.- Realizar medidas de reanimación.
  • 153. CASO CLINICO • Paciente masculino 10 días de nacimiento, que se obtuvo en casa sin control prenatal, ingresa por diarrea, llanto, e irritabilidad, vomito en proyectil, en casa, refiere la madre que solo fue en una ocasión, al ingreso se observo hipotónico, llanto agudo, mal estado generalizado, con tono cervical no se observo rigidez, pero fontanela abombada, su peso fue de 2050 grs, laboratorios con 150,000 de plaquetas, antecedentes de rinorrea hialina, se presento 35 grados, irritabilidad, durante la exploración se observo pedaleo y chupeteo, dificultad respiratoria.
  • 154. PREGUNTA No 113 Para establecer el diagnostico y conducir su terapéutica para el caso? RESPUESTA a.- Hemocultivo. b.- Punción lumbar. c.- Tomografía. d.- Biometría hemática.
  • 155. PREGUNTA No. 114 Se encuentra en espera de resultados, sin embargo el paciente se va deteriorando, cual es la agente etiológico mas probable para dar una terapéutica dirigida? a.- H influenza b.- E Coli. c.- Listeria monocitogenes d.- Estreptocco beta hemolito.
  • 156. PREGUNTA No. 115 Cual es la conducta farmacológica más adecuada considerando la etiología? RESPUESTA a.- Ampicilina mas amikacina. b.- Ampicilina mas metronizadol. c.- Ampicilina mas vancomicina. d.- Ampicilina mas cefotaxima.
  • 157. CASO CLINICO Varón de 33 años que sufrió un traumatismo torácico severo tras atropello. Precisó ventilación mecánica y fármacos inotrópicos. Presentaba un soplo de insuficiencia aórtica. La TAC mostro neumotórax y fracturas costales bilaterales, sin datos de disección aórtica. El USG evidenció insuficiencia aórtica severa con dilatación ligera de la raíz aórtica, se observó un velo coronariano derecho roto, que protruía hacia el tracto de salida del ventrículo izquierdo, e insuficiencia aórtica severa. En la pared anterior de aorta ascendente proximal, había una imagen de seudoaneurisma con rotura de capas íntima y media y se observaban colgajos medio-intimales protruyendo hacia la luz.
  • 158. PREGUNTA No. 116 La triada de Beck consiste en: RESPUESTA a.- Hipotensión, ruidos cardiacos velados y distensión venas del cuello. b.- Hipertensión, edema facial y ruidos cardiacos velados. c.- Hipotensión, torax inestable y disfagia. d.- Ruidos cardiacos velados, distensión venas del cuello y disfonía.
  • 159. CASO CLINICO • Un hombre de 31 años de edad, con antecedentes de DM, alcoholismo y tabaquismo. Con desprendimiento total de retina de su ojo derecho y la fotocoagulación con láser de su ojo izquierdo. Examen mostró etapa final de retinopatía diabética proliferativa con desprendimiento de retina traccional. El ojo izquierdo tenía la neovascularización extensiva y la proliferación fibrovascular de la retina secundaria a la retinopatía diabética proliferativa avanzada.
  • 160. PREGUNTA No 117 Cual de los siguientes factores es de mayor importancia para el desarrollo de la retinopatía diabética? RESPUESTA a.- Más de 5 años de evolución de la DM. b.- Tabaquismo positivo intenso. c.- Progresión rápida de la enfermedad. d.- Depuración de creatinina menos de 60 ml/ minuto.
  • 161. CASO CLINICO Un paciente de 78 años, sin antecedentes de importancia y con un control periódico normal hace 6 meses, comienza hace 24 horas con dolor quemante, en la región torácica posterior derecha, irradiado a flanco e hipocondrio homolaterales. La semiología pulmonar es normal y no presenta contractura muscular. En el abdomen no hay visceromegalias y presenta leve dolor a la palpación superficial en hipocondrio derecho. La radiografía de tórax y de columna dorsal y la ecografía hepatobiliopancreática son normales.
  • 162. PREGUNTA No. 118 Cuál es el diagnóstico más probable? RESPUESTA a.- CCL. b.- Espondilitis anquilosante. c.- Herpes zoster. d.- Pleuritis.
  • 163. CASO CLINICO • Un paciente de 87 años con antecedentes de bronquitis crónica e insuficiencia cardiaca, ha sido diagnosticado de colecistitis aguda litiasica. Tras 4 dia de hospitalización en tratamiento con dieta absoluta, liquidos IV, piperacilina / tazobactam, el paciente continua con fiebre, dolor abdominal persistente y leucocitosis, desihidratacion, hipeglucemia, hipotension, y taquicardia.
  • 164. PREGUNTA No. 119 La actitud mas adecuada en este momento seria? a.- Colecistectomia urgente. b.- Drenaje biliar mediante colecistostomia percutanea. c.- Sustituir la piperacilina/tazobactam por metronidazol+cefotaxima. d.- Sustituir la piperacilina/tazobactam por amikacina+clindamicina.
  • 165. CASO CLÍNICO • Se trata de paciente de 24 años de edad la cual acude a solicitud de la pareja debido a que presenta secresion por meato urinario, de tipo ardoroso, sin embargo la paciente no presenta manifestaciones, ocasionalmente presenta descarga vaginal, la EF se observa leve descarga directa del cervix.
  • 166. PREGUNTA NO. 120 Cual es el diagnostico mas probable que presenta la pareja? RESPUESTA a.- Gonorrea. b.- Tricomoniasis. c.- Candida d.- Garnerella
  • 167. PREGUNTA NO. 121 Cual es el diagnostico mas probable que presenta la pareja? RESPUESTA a.- Espectinomicina 2 gr. b.- Penicilina procainica 4 millones previo probenicid 1 gr. c.- Ciprofloxacilina 2 grs dosis unica. d.- Ceftriaxona 2 gr. IM DU.
  • 168. Pregunta No. 122 Cual de los siguientes agentes infecciosos no es diferencial en este caso? RESPUESTA a.- Toxoplasma. b.- Ureplasma. c.- Sifilis. d.- Clanmidya
  • 169. Pregunta No. 123 Acude a consulta la pareja de la paciente, cual presenta secresión en meato urinario, durante la exploración observa hiperemia conjuntival. Cual es el agente mas probable que se presenta en la mucosa subconjuntival? RESPUESTA a.- Toxoplasma. b.- Ureplasma. c.- Sifilis. d.- Clanmidya
  • 170. Pregunta No. 124 Se trata de paciente femenino que acude por descarga vaginal frecuente, sin mal olor, la pareja no presenta sintomatologias, no es ardorosa, ni ulceraciones, se presume que presenta una cervicitis por mycoplasma, cual es el medio mas adecuado para cultivar este agente? RESPUESTA a.- Caldo de argirina. b.- Caldo de urea. c.- Medio agar. d.- Medio saburoe.
  • 171. Pregunta No. 125 Se trata de paciente femenino que acude por descarga vaginal frecuente, a la revisión vaginal se observa ulceración en cérvix, la secreción es amarillenta, con olor desagradable, leve dolor a la palpación y negativo a la prueba de aminas, se envía secreción a cultivo, cual es el fármaco de elección en espera de resultados? RESPUESTA a.- Ceftriaxona b.- Metronidazol c.- Fluconazol d.- 5 fluoracilo.
  • 172. Pregunta No. 126. Se trata de paciente femenino que acude por descarga vaginal frecuente, ardor vulvar con secresion de lesion ulcerosa, a la exploracion fisica se detecta cadena ganglionar inflamada, cual es su conducta a seguir? RESPUESTA a.- Aspirado de ganglios. b.- Cultivo de secresiones. c.- Realizar VDRL. d.- Colposcopia.
  • 173. Pregunta No. 127 Se trata de paciente femenino que acude por descarga vaginal frecuente, ardor vulvar, con bubon unilateral y ganglios palpables? RESPUESTA a.- Herpes genital b.- Linfogranuloma venerio. c.- Chancroide d.- Sifilis secundaria.
  • 174. Pregunta No. 128. Se trata de paciente femenino que acude por descarga vaginal frecuente, ardor vulvar intenso con vesículas múltiples, acudió 5 días después de iniciado, intento usar una crema, la cual incremento las lesiones, a la exploración física se observa lesión corticoestropeada cual es el diagnostico mas probable? RESPUESTA a.- Herpes tipo I b.- Herpes tipo II c.- Herpes genital. d.- Herpes zoster.
  • 175. Pregunta No. 129. Se trata de paciente femenino que acude por descarga vaginal frecuente, ardor y dolor en vulva, con inflamacion importante en ganglios, con bubon, por arriba de ligamento inginal, con el signo del surco dividido? RESPUESTA a.- Linfogranuloma venereo b.- Herpes genital. c.- Balanosis. d.- Chancroide.
  • 176. Pregunta No. 130. Cual de las siguiente complicaciones es la menos probable? RESPUESTA a.- Queratosis. b.- Elefantiasis c.- Cervicitis crónica. d.- Paralisis y demencia.
  • 177. Pregunta No. 131 Se trata de femenino de 10 años que presenta lesiones sugerentes de hiperqueratinocitis, abundante con lesiones que sobrepasan la piel, algunas lesiones miden mas de 1 cm, lesiones umblicadas, tiene un aspecto perlado, sin descamación, con prurito, cual es su impresión diagnostica? RESPUESTA c.- Queratotenoma. b.- Molusco contagioso. c.- Lesiones de ducrey. d.- Citomegalovirus.
  • 178. Pregunta No. 132 Se trata de femenino de 21 años de edad que presenta cansancio, fatiga, adinamia, con linfadenopatia, hígado con tamaño dentro de lo normal, esplenomegalia grado I, con fiebre, cual es el agente etiológico mas probable? RESPUESTA a.- Mononucleosis. b.- Citomegarovirus c.- Hepatitis B d.- VIH.
  • 179. Pregunta No. 133 Se trata de femenino de 28 años de edad la cual acude por cansancio, fatiga, anorexia, adinamia, diarrea, dolor a la palpación abdominal de predominio cuadrante superior derecho, los cultivos de secreciones resultaron negativos para los agentes de ETS habituales? RESPUESTA a.- Mononuclesosis b.- Virus herpes genital. c.- Citomegarovirus d.- Hepatitis B
  • 180. CLINIC CASE A 16 year old female familiar to you from previous yearly visitis was brought to your office today by her mother who is concerned about her frequent complaints of dizziness and weaknes. There is no hitory of syncope, but the patient often remarks that se feels as if she is about to “pass out”. Revies of systems is negative for fever, weight gain or loss, nigth sweats, chest pain, palpitations, increased work of breathing, vomiting or diarrhea. Weigth continues to track along the / 5th percentile; height remains at the 50 th percentile, Vitial signs are within normal limits for age, including orthostatic blood pressure measurements. The physical examination is unremarkable except for some excess wear on the upper incisor.
  • 181. QUESTION No. 134 Tests Na 134 mEq/L, K 2.8 mEq/L, Cl 83 mEq/L, HCO3 35 mEq/L, creatinina 0.6 mg/dL, Urea 12 mg/dL, pH7.5, urine pH 9.0 This patient´s physical examination and laboratory values are most consistent with of the following condictions? ANSWER a.- Zinc deficiency. b.- Renal tubular acidosis. c.- Anorexia nervosa. d.- Bulimia nervosa.
  • 182. CLINIC CASE An 8 year old female is brought to your office the weeked after july 4 th for evaluation of fever. The child was well until yesterday, when she complained of a headache. Overnight she developed a fever to 39.9 C. and began complaining of achiness “all over” Thay also want you to look at a red place on her upper arm. You note an irregular annular about 4 cm across at the widest, with partial central clearinq. The areas is not scaly, and the child denies pain or pruritus associated with the lesion.
  • 183. QUESTION No. 135 Antibiotic therapy doxycycline twice a day for 10 days, should be administered to prevent wich of the following complications of this patient´s infection? ANSWER a.- Encephalitis. b.- Liver failure. c.- Relapsing arthritis. d.- Rheumatic heart disease.
  • 184. CLINICAL CASE A 9 year old male sustains an inversion injury to his ankie while playing soccer. He is untable to bear any weigth on the affected ankle, which is markedly swollen. Tendernes is most pronunced over the lateral malielu, particulary the distal fibular swelling but are otherwise normal.
  • 185. QUESTION No. 136 Which of the following is the most likely result of this boy´s injury? ANSWER a.- Frature of the distal fibular epiphysis. b.- fracture of the 5th metatarsal. c.- Rupture of the peroneal tendon. d.- Sprain of the calcaneofibular ligamente.
  • 186. CLINIC CASE During the initial assessment of a therm newborn, you note empty scrotal sacs on goth the right and the left. A small mobile mass the size of a typical newborn testis is palpated along the left inguinal canal; gentle traction does not result entry of the mass into the left scrotal sac. No similar mass is present on the right side. The genital, rectal, and back examinations are otherwise unremarkable. A sonogram of the pelvis confirms the right testis in the inguinal canal. The left testis is in the abdomen. Orchiopexy is successful at 12 months of age.
  • 187. QUESTION NO. 137 This patient is most at risk for which of the following complications? ANSWER a.- Orchitis. b.- Testicular torsion. c.- Testicular cancer. d.- Testicular feminization.
  • 188. CLINIC CASE You are called to the nursery to evaluate o newborn infant. The infante is full-term but weighs only 2040 grams. The liver is enlarged, and “blueberry muffin” spots are evident in the skin. You obtain computed tomography of the brain, which demonstrates periventricular calcifications. You suspect congenital infection with cytomegalovirus.
  • 189. QUESTION No. 138 Which of the following is the most likely long-term complication of congenital CMV? ANSWER a.- Hearing loss. b.- Keratoconjunctivitis. c.- Hydrops fetalis. b.- Cataracts and trachoma.
  • 190. CLINIC CASE A 46 year old male has had intermittent flares of ulcerative pan colitis for 10 years. He has increasing diarrhea, blood per rectum, and fever. He responds to medical management.
  • 191. QUESTION No. 139 Proctocolectomy is indicated for? ANSWER a.- Severe dysplasia on initial biopsy. b.- Persistent stricture. c.- Long periods of persistent disease. d.- Length of colon involved.
  • 192. QUESTION No. 140 On routine rectal examination, a 2cm mass is felt. Biopsy is a carcinoid initial operative treatment is? ANSWER a.- Abdominal perineal resection. b.- Fulguration. c.- Low anterior signoid resection. d.- Transanal excision.
  • 193. QUESTION No. 141 A 60 year old female has a 2 cm hot thyroid nodule and exhibits signs and symptoms of thyrotoxicosis, treatment is? ANSWER a- Propythiouracil and FNA. b.- f3- blocker and immediate thyroidectomy. c.- Radioactive therapy. d.- Potassium iodide SSKI
  • 194. CLINIC CASE A ventilated patient has a tidal volumen of 800 and a respiratory rate of 12. You want to lower the tidal volumen to 600 and maintain the same amount of ventilation. QUESTION No. 142 What will be the respiratory rate? ANSWER a- 16. b.- 14. c.- 18. d.- 12
  • 195. CASO CLINICO Se trata de paciente masculino de 23 años de edad el cual ingresa a urgencias presentando orificio de entrada en flanco derecho, por arma de fuego, sin orificio de salida, a la exploración física se observa con FC 125 lpm, llenado capilar lento, FR 31 rpm, se observa ansioso con tendencia a la confusión, su gasto urinario fue de 20 ml/h.
  • 196. PREGUNTA No. 143 Considerando las manifestaciones clínicas observadas así como las variables vitales, en que estado de choque se encuentra el paciente? RESPUESTA a.- Clase I. b.- Clase II. c.- Clase III. d.- Clase IV.
  • 197. PREGUNTA No. 144 Cual es la conducta para mantener el volumen circulante mas adecuada? RESPUESTA a.- Solución salina 10 ml/kg/hora. b.- Solución hartman 15ml/Kg/hora. c.- Crioprecipitados. d.- Concentrado eritrocitario y cristaloide.
  • 198. CASO CLINICO Una hora después, el paciente presenta los siguientes constantes vitales, FC 105 lpm, FR 21 rpm, gasto urinario de 35 ml/h, se encuentra ansioso, pero no confundido, la hemoglobina fue de 12 y hematocrito de 37, plaquetas de 120,000, asi como DHL 312, AST 435, tiempos de coagulación levemente incrementados.
  • 199. PREGUNTA No. 145 Cual es la conducta a seguir en este momento? RESPUESTA a.- Mantener observación estrecha. b.- Preparar para laparatomia exploratoria. c.- Incrementar volumen circulante con paquete globular. d.- Vasopresores, volumen y LAPE.
  • 200. PREGUNTA No. 146 Se realizo LAPE con reparación hepática con buena respuesta, fue necesario crioprecipitados, 5 unidades de plaquetas y 7 concentrados eritrocitarios, se administro 3 gramos de gluconato de calcio, cual es la razón de esta indicación? RESPUESTA a.- Alteración de coagulación. b.- Mantener estable la membrana. c.- Evitar arritmias. d.- Disminuir la adhesividad plaquetaria.
  • 201. CASO CLINICO Se trata de paciente femenino de 48 años de edad, la cual fue ingresada posterior a accidente automovilístico y volcadura, era copiloto y fue la única sobreviviente, se observa confundida, desorientada, agitada, se presenta diaforesis, con palidez de mucosas, su llenado capilar lento, FC de 138 lpm, FR 34 rpm, TA 100/65 mmHg, minutos después se estableció un Glasgow de 8, distensión abdominal e inestabilidad toracicca, su gasto urinario fue de 20 ml/h, Campos pulmonares con matidez a la percusión bibasal, no hay ingurgitación yugular, pero hay equimosis importante sobre area del cinturón de seguridad, en la radiografia portátil se observo fractura de 5 costillas del lado izquierdo.
  • 202. PREGUNTA No 147 Cual es la medida inmediata mas adecuada en este momento? RESPUESTA a.- Intubación endotraqueal. b.- Solución hartman, vasopresores. c.- Solución salina hipertónica. d.- Sedación y relajación muscular.
  • 203. PREGUNTA No 148 Posterior al decisión anterior se administra 3 paquetes de concentrados eritrocitarios, 3 paquetes de plasma fresco y 2 litro de solución hartman, sin embargo el gasto urinario es de 25 ml/hr, la tensión arterial se mantiene en 95/60 mmHg durante la primera hora de estancia, cual es la conducta a seguir mas adecuada? RESPUESTA a.- Albumina y expansores. b.- Infusion de dopamina 5 µg/Kg/min. c.- Infusion de dobutamina 10 µg/kg/min. d.- Crioprecipitados y gluconato de calcio.
  • 204. PREGUNTA No 149 Cual es el objetivo mas importante de la conducta farmacológica tomada anteriormente, que no sea en detrimento de los mecanismos de reparacion inmediata? REPUESTA a.- Mantener el gasto urinario. b.- Incrementar la fracción de eyección. c.- Mantener la tensión arterial permisible. d.- Favorecer el inotropismo.
  • 205. PREGUNTA No 150 El paciente presenta estabilidad en la tensión arterial 95/70 mmHg y gasto urinario de 35 ml/hr, se prepara para laparatoria exploratoria, tomando en cuenta la decisión farmacológica anterior, cual de los siguiente efectos no favorece al caso? RESPUESTA a.- Incremento > del 25 % sobre la tensión arterial basal. b.- Aumento del flujo sanguíneo renal. c.- Disminución de la resistencia vascular periférica. d.- Disminución de la demanda de O2 del miocardio.
  • 206. CLINIC CASE Thre weeks following a nissen fundoplication, the patient has a severe episode of retching following a heavy meal and has forceful vomiting. Two hours later, he has severe epigastric pain and a spiking temperature of 102 F.
  • 207. QUESTION No. 151 Which of the following is the problem? ANSWER a.- The nissen has slipped. b.- Funduplication disrruption has ocurred with perforation. c.- A perforated ulcer has developed. d.- Acute pancreatitis.
  • 208. QUESTION No. 152 Submucosal spread of malignant neoplasms is common and has major operative consequences in? ANSWER a.- Esophageal cancer. b.- Rectal cancer. c.- Small bowel cancer. d.- Colon cancer.
  • 209. CLINIC CASE A 55 year old patient presents with dysphagia. A barium swallow identifies a smooth filling defect with the mucosa intact in the distal esophagus. QUESTION No. 153 Appropiate tratment is? ANWSER a.- Total esophagectomy b.- Distal esophagectomy. c.- Enucleation of the lesion. d.- Endoscopic drainage.
  • 210. QUESTION No. 155 Tratment of barrett´s esophagus with a nessen fundoplication is indicated for? ANSWER a.- Halting the progression of the disease. b.- Eliminating the risk of cancer. c.- Elimination of the stricture. d.- Resolution of the pathologic finding.
  • 211. CLINIC CASE A 75 year old female is evaluated for reflux esophagitis. At endoscopy the mucosa is salmon pink with a definite demarcation between normal and abnormal esophageal mucosa. Biopsy indicates no dysplasia?
  • 212. QUESTION No. 156 Initial tratmente is? ANSWER a.- Esophageal resection. b.- Primary medical tratment. c.- Nissen fundoplication. d.- Ablation of abnormal mucosa.
  • 213. CASO CLINICO Se trata de paciente masculino de 19 años de edad el cual ingresa a urgencias posterior a accidente automovilístico, refiere cefalea y dolor lumbar leve, permaneció en observación, se coloco catéter periférico 18 fr y solución glucosada al 5 % para 8 hrs, a las 4 horas posterior al ingreso refiere ansiedad, sensación de mareo, a la EF se observa palidez de tegumentos, diaforesis, torax sin datos patológicos, abdomen con ruidos peristálticos disminuidos, SV con TA 100/70 mmHg, FC 105 lpm, FR 32 rpm, no se apresia compromiso oseo.
  • 214. PREGUNTA No. 157 Se realizan realiza QS, BH, RX de torax y abdomen, Hb 10.1, Hto 30, Rx de abdomen con niveles hidroaereos, imagen despulida, cual es la conducta a seguir más adecuada? RESPUESTA a.- Solución hartman 10 ml/kg/hora. b.- Solución salina 0.45 % 10 ml/kg/hora. c.- Solución mixta 10 ml/kg/hora. d.- Solución glucosada al 10 % 10 ml/Kg/hora.
  • 215. PREGUNTA No. 158 Posteriomente se administra 3 concetrados eritrocitarios y 2 paquetes de plasma fresco, la tensión arterial se mantiene estable en 100/70 mmHg sin embargo la urésis fue de 40 ml/hora, se observa auscencia de ruidos peristálticos, se observa estado de conciencia adecuado, alerta y tranquilo, cual no es un objetivo primordial en este momento del caso? RESPUESTA a.- Elevar la tensión arterial media por arriba de 95 mmHg. b.- Manterne la hemostasia con gluconato de calcio. c.- Estabilizar hemostasis por sospecha de hemorragia. d.- Mantener un adecuado volumen urinario.
  • 216. CLINIC CASE A 62 year ol male presents with dyspepsia. An UGI shows thckening of the rugal folds. At upper endoscopy, no mass or lesion is identified. The gastric wall is thickened but no ulceration is noted.
  • 217. QUESTION No. 159 Following endoscopic biopsy, this patient is best treated by? ANSWER a.- Radiation. b.- Chemotherapy. c.- Chemotherapy and radiation. d.- Subtotal gastrectomy
  • 218. CASO CLINICO Mujer de 34 años sin antecedentes patológicos, no hábitos tóxicos, no alergias a fármacos conocidas. Presentó cefalea tensional en tratamiento con ácido acetilsalicílico con episodios de urticaria aguda en relación con el consumo de marisco, tratado y remitido hace 24 hrs. 4 horas después apareció dolor faringeo, tumefacción facial, de manos y brazos, con disminución de la diuresis. A las pocas horas de la hospitalización presentó una alteración hemodinámica con taquicardia sinusal a 140 pulsaciones por minuto e hipotensión.
  • 219. PREGUNTA No. 160 Considerando el cuadro clínico cual es la conducta inmediata a seguir, la paciente se encentra con vías de acceso venoso? RESPUESTA a.- Administración de volumen con cristaloides. b.- Administración de aminas vasoactivas. c.- Colocación de catéter largo. d.- Administración de epinefrina.
  • 220. PREGUNTA No. 161 Posteriormente se deterioró el nivel de conciencia y se produjo un paro respiratorio que precisó intubación y ventilación mecánica. Se consiguió la estabilización hemodinámica con dopamina y expansores de volumen, crioprecipitados y plasma fresco, cual es la complicación más probable? RESPUESTA a.- Coagulación por hemodilución. b.- Sindrome compartimental. c.- Coagulación intravascular por consumo. d.- Coagulopatia dilucional.
  • 221. PREGUNTA No. 162 Considerando el cuadro clínico y la evolución del caso, cual de las siguientes causas es la menos probable? RESPUESTA a.- Reacciones a fármacos. b.- Reacciones a alimentos. c.- Reacciones transfusionales. d.- Reacciones a antígenos inhalados.
  • 222. CASO CLINICO Paciente varón de 57 años con larga evolución en cuidados intensivos por (SDMO) y (SRIS) secundarios a proceso postquirúrgico. Fue gastrectomizado y desarrolló una doble fístula pleural y digestiva. Ha tenido simultáneamente diversos procesos infecciosos sistémicos (empiema, peritonitis, herida quirúrgica y respiratoria) tratados mediante antibiograma. Además ha presentado repercusión orgánica persistente con necesidad de soporte vasopresor para control hemodinámico y respiratorio.
  • 223. RESPUESTA No. 163 Considerando las características actuales que se observan en el paciente que tipo de choque se observa actualmente? RESPUESTA a.- Choque distributivo. b.- Choque restrictivo. c.- Choque séptico. d.- Choque oculto.
  • 224. RESPUESTA No. 164 El día 11 se introduce tratamiento con linezolid por IV, 600mg/12h por aislamiento de Enterococcus faecium resistente a meticilina, sensible a linezolid, enoxaparina 40mg/24h, propofol 300mg/h. A las 48 h del inicio de linezolid se observan crisis comiciales en relación con movimientos tonicoclónicos preferentemente de extremidades superiores y tronco sin llegar a generalizarse, transitorios. cual es su impresión? RESPUESTA a.- Choque anafiláctico. b.- Reaccion adversa a medicamento. c.- Choque séptico. d.- Síndrome serotoninergico maligno.
  • 225. RESPUESTA No. 165 Se observa también hiperreflexia, discreta acidosis metabólica y empeoramiento grave de la fiebre, leucocitosis, marcadores de inflamación y estado hemodinámico (hipotensión). Ante la sospecha de RAM se sustituye meperidino por fentanilo y se añade midazolam, 10mg/h, en perfusión continua al esquema de sedoanalgesia para la mejora del cuadro y noradrenalina para el soporte hemodinámico. Considerando las variable vitales, hipotensión refractaria cual de las siguientes medidas es mas adecuada? RESPUESTA a.- Incrementar aminas vasoactiva. b.- Hemodialisis. c.- Retirar linezolid y administrar metilprednizolona. d.- Concentrado eritocitario y plasma fresco congelado
  • 226. RESPUESTA No. 166 En los cultivos se aisló Candida spp. Se administro anfotericina B en dosis de 4mg/kg/día. Linezolid fue también suspendido y se indico daptomicina, 350mg/24h. Tras el cambio de tratamiento las mioclonías e hiperreflexia mejoraron antes de 24 h, hasta desaparecer por completo al segundo día. La fiebre, leucocitosis e hipotensión mejoraron también progresivamente, aunque la acidosis metabólica persistió. Cual es la complicación mas probable? RESPUESTA a.- Dudoso por acidosis metabolica. b.- Coagulopatia por dilución. c.- Alergia al fármaco. d.- Falla organica multiple.
  • 227. RESPUESTA No. 167 El diagnóstico de síndrome serotoninérgico suele ser difícil de realizar por la presencia de síntomas inespecíficos y comunes a otras entidades. Esta problemática puede ser todavía mayor en el caso de un paciente crítico y polimedicado como el que nos ocupa, donde los síntomas pueden ser atribuidos a múltiples factores de la enfermedad o del abordaje farmacológico. Cual es la base principal para diagnosticar este caso? RESPUESTA a.- Inicio brusco. b.- Recuperación tras suspensión del fármaco. c.- La presencia de fiebre, hiperreflexia y acidosis metabolica. d.- Criterios diagnósticos de Sternbach y Dunkley.
  • 228. CASO CLINICO Se trata de un varón de 74 años que ingresó por fiebre y deterioro del estado general de meses de evolución. Como antecedentes personales, cabe destacar gastritis crónica atrófica, cardiopatía reumática con afectación aórtica y mitral y fibrilación auricular con tratamiento anticoagulante. En la exploración física no había alteraciones significativas. En la analítica, a reseñar una creatinina de 4,17 mg/dL no conocida y un INR de 7. En los hemocultivos realizados, crecieron en 2/3 L. monocytogenes. En el estudio de la bacteriemia, se solicitó un ecocardiograma transtorácico que puso de manifiesto una verruga en la válvula aórtica, con fracción de eyección deprimida, KDOQ III, cual es la conducta farmacológica mas adecuada a seguir?
  • 229. PREGUNTA No. 168 Cual es la conducta farmacologica de primera instancia es la mas adecuada? RESPUESTA a.- Administrar ampicilina y vancomicina. b.- Administrar ampicilina y gentamicina ajustada. c.- Administrar imipenem y amikacina. d.- Administrar ceftriaxona e imipenem.
  • 230. PREGUNTA No. 169 En la RMN craneal, se observaron múltiples infartos cerebrales, e hígado de estasis. La evolución fue mala, desarrollando insuficiencia cardiaca por lo que se decidió realizar cirugía cardiaca. En la intervención, se encontraron vegetaciones y abscesos que afectaban tanto a la válvula aórtica como a la mitral, presento hipotensión refractaria, sin respuesta a aminas vasopresoras, hipotermina, piel marmórea. Cual es el diagnostico mas probable en este momento del caso? RESPUESTA a.- Choque cardiogenico. b.- Choque séptico. c.- Choque restrictivo. d.- Choque oculto.
  • 231. CLINIC CASE A 18 year old white female presents with nausea, vomiting, and RLQ pain. At exploration, the appendix is normal. However, there is inflammation and edema of the distal ileum involving the appendiceal stump.
  • 232. QUESTION No. 170 The operation of choice is? ANSWER a.- Appendectomy. b.- Exploration only. c.- Ileocolectomy. d.- Appendectomy and drainage.
  • 233. CLINIC CASE A 46 year old male has had intermittent flares of ulcerative pan colitis for 10 year. He has increasing diarrhea per rectum, and fever. He responds to medical management. QUESTION No. 171 The operation of choice is? ANWSER a.- Length of colon involved. b.- Long periods of persistente disease. c.- Severe dysplasia on initial biopsy. d.- Persistent stricture.
  • 234. PREGUNTA No. 172 Paciente masculino de 21 años toxicomano, con datos de isquemia miocardica secundaria a consumo de crack (cocaina base), cual es el objetivo mas importante para disminuir la posibilidad de compromiso miocardico? RESPUESTA a.- Bajar la demanda de O2. b.- Disminuir la resistencia periférica. c.- Aumentar la capacitancia venosa. d.- Cronotropismo negativo.
  • 235. PREGUNTA No. 173 Se trata de paciente masculino de 67 años de edad con IAM de 3 horas de evolución con acinesia de apex, que se manejo con MONA inicialmente, se esta evaluando la posibilidad de reperfusión de rescarte, cual es el criterio mas adecuado para su reperfusion de rescate? RESPUESTA a.- Las alteraciones del sitio de acinesia. b.- La actividad eléctrica alterada. c.- La edad del paciente. d.- Las caracteristicas de biomarcadores.
  • 236. CLINIC CASE A 21 year old male was diagnosed with right eye optic neuritis last month, with successful recovery of his vision after treatment. The patient has never han any other neurolical symptoms of wekness, numbness, etc. MRI brain showed 3 white matter lesions. PREGUNTA No. 174 The next step of action? RESPUESTA a.- Methotrexate. b.- Start oral steroids. c.- No further treatment. d.- Interferon beta 1ª.
  • 237. CASO CLINICO Se trata de paciente masculino que presenta trastorno mental por consumo de cocaina y marihuana desde hace mas de 10 años, ha presentado dos ingreso por sobredosis, durante el presente año. PREGUNTA No. 175 Cual es el núcleo mas importante que se encuentra involucrado en la adicción de las sustancia? RESPUESTAS a.- Nucleo tecmental ventral. b.- Nucleo accumbens. c.- Nucleo ceruleo. d.- Nucleo del rafe caudal.
  • 238. CLINIC CASE A 34 year old female was incidentally diagnoses whit a unruptured middle cerebral artery aneurism. QUESTION No. 176 Which of the following is a known risk factor for cerebral aneurysms? ANSWER a.- Cigarette smoking. b.- Diabetes. c.- Hyperthyroidism. d.- Hyperlipidemia.
  • 239. CLINIC CASE A 34 year old female was incidentally diagnoses whit a unruptured middle cerebral artery aneurism. QUESTION No. 177 Which of the following is a known risk factor for cerebral aneurysms? ANSWER a.- Cigarette smoking. b.- Diabetes. c.- Hyperthyroidism. d.- Hyperlipidemia.
  • 240. CLINIC CASE A 66 year old man with hypertension, dyslipidemia and two recent unevaluated TIA´S presented with an acute onset of right sided weakness. On neurological exam the patient was alert, fully oriented, had fluent language, normal repetition and naming. His pupils were equal, round and reactive, his face was symmetric, he had intact facial sensation, his hearing was grossly normal, there was no evidence of dysarthria, and his tongue was midline. Strength was normal in the LUE and LLE. Strength was 4/5 with drift in the right upper extremity.
  • 241.
  • 242. CASO CLINICO Se trata de paciente femenino de 67 años de edad, que acude a consulta por cansancio, fatiga, dificultad para respirar, dolor torácico que disminuye con reposo, disnea de moderados esfuerzos, edema de miembros inferiores hasta 2/3 de la pierna, a la exploración física se observo palidez de piel y mucosas, llenado capilar lento, estertores crepitantes leves, presencia de galope y trill. La paciente es hipertensa desde hace 20 años con moderado apego a tratamiento con captopril 50 mg/dia, hidroclorotiazida 25 mg/12 hrs. Ha presentado 2 crisis hipertensivas, asi como un evento de insuficiencia mesentérica, sus laboratorios EGO proteínuria, glucosuria, uratos, densidad urinaria disminuida, colesterol 289 mg/dl, triglicéridos 720 mg/dl, creatinina 6.2 mg/dl, BUM 29, kalemia de 5.3, los signos vitales fueron TA 160/105 mmHg, FC 96, FR 28, IMC 32.
  • 243. PREGUNTA No. 178 Se realizo un ecocardiograma donde se observa una FE de < 40 %, durante su ingreso la paciente presento volumen urinario de 20 ml/hr. Considerando las características actuales cual es la clase funcional que presenta el caso? RESPUESTA a.- Clase funcional I. b.- Clase funcional II. c.- Clase funcional III. d.- Clase funcional IV.
  • 244. PREGUNTA No. 179 Considerando las características actuales cual es el K/DOQI que presenta el paciente, cual es el método mas adecuado para determinar el estadio de la IRC? RESPUESTA a.- Depuracion de creatinina de 24 hrs. b.- Biopsia renal. c.- Ultrasonografia renal. d.- Urografia escretora
  • 245. PREGUNTA No. 180 Cual es la conducta farmacológica mas adecuada considerando el K/DOQI y la clase funcional? RESPUESTA a.- Verapamilo. b.- Diltiazem. c.- Telmisartan. d.- Isosorbide.
  • 246. PREGUNTA No. 181 Cual es la conducta inmediata a seguir para mejorar la precarga disminuyendo los niveles de potasio? RESPUESTA a.- Furosemide. b.- Dialisis peritoneal. c.- Hemodialisis. d.- Soluciones metabolizadas.
  • 247. PREGUNTA No. 182 Como esperaría encontrar al calcio y fosforo en el paciente en la condición actual? RESPUESTA a.- Calcio bajo, fosforo bajo. b.- Calcio alto, fosforo alta. c.- Calcio bajo, fosforo alta. d.- Calcio alta, fosforo baja.
  • 248. PREGUNTA No. 183 Que Cockroft-Gault, MDRD y K/DOQI presenta el paciente? RESPUESTA a.- Cockroft-Gault 284, MDRD 67 y K/DOQI (III). b.- Cockroft-Gault 384, MDRD 57 y K/DOQI (IV). c.- Cockroft-Gault 484, MDRD 47 y K/DOQI (V). d.- Cockroft-Gault 184, MDRD 37 y K/DOQI (II).
  • 249. PREGUNTA No. 184 Cual de las siguientes patologías es la principal causa de IRC? RESPUESTA a.- Diabetes mellitus. b.- Hipertensión arterial. c.- Glomerulonefritis. d.- Enfermedad renal poliquística.
  • 250. PREGUNTA No. 185 Cuales factores de riesgo aceleran el deterioro del funcionamiento renal en el caso actual? RESPUESTA a.- Dislipidemia. b.- Edad avanzada. c.- Diabetes mellitus. d.- Anemia.
  • 251. CASO CLINICO Se trata de masculino de 59 años de edad, ingresa a urgencias debido a dificultad para respirar, cansancio, fatiga, tos productiva desde hace 15 dias intensificandoce la noche previa al ingreso actual, cuenta con antecedentes de EPOC diagnosticado hace 10 años, posterior a tabaquismo (una cajetilla diaria), con tratamiento de bromuro de iprapropio, salbutamol y betametasona, además fue diagnosticado como hipertenso hace 2 años, inicio con cambios del estilo de vida y dieta con restricción de sodio, hace un año se considero incluir captopril que fue cambiado 3 meses después por incremento de tos, desde entoces toma losartan, hidralazina. A la EF se observa facies pletórica con hiperemia conjuntival, mucosas orales cianóticas moderadamente, se ausculta un soplo carotideo derecho, ingurgitación yugular grado II, estertores subcrepitantes bibasales, con resonancia tipanica a la percusión, los ruidos cardiacos levemente disminuidos, rítmico, abdomen con hepatomegalia 5 cm debajo del borde costal.
  • 252. PREGUNTA No. 186 Considerando el estado actual del paciente cual es la conducta mas apropiada a seguir, para establecer la función cardiopulmonar? RESPUESTA a.- Espirometria. b.- Ecocardiograma. c.- Valor de natriuretico. d.- Radiografia de torax.
  • 253. PREGUNTA No. 187 Se observo una FE < 40 %, indice cardiotoraccico patológico, exudados algodonosos bilaterales, trama brocovascular incrementada, gases con PCO 49, PO2 85, potasio 6.3, Factor natriuretico disminuido, a la EF se auscultan estertores, sibilancias, hiperresonancia, cual es la conducta mas apropiada para disminuir la precarga? RESPUESTA a.- Retirar diurético ahorrador de potasio por diurético de asa. b.- Iniciar con calcioantagonista dihidropiridinico de acción rápida. c.- Incrementar la capacitancia vascular periférica con nitritos. d.- Restriccion hídrica, dieta libre de sodio, diurético de asa.
  • 254. CASO CLINICO Femenino de 37 años de edad la cual acude a consulta debido a que no ha logrado embarazarse, refiere que lleva 4 años de intentándolo, con vida sexual activa desde hace 5 años con la misma pareja, no usa método anticonceptivo, como antecedentes refiere menarca a los 19 años, gesta 0, para 0, abortos 0, a la exploración física se observa con caracteres sexuales secundarios adecuados, a la exploración GO sin secreción vaginal blanquecino sin olor desagradable, a la especuloscopia con normal, acude con resultados de Papanicolaou con datos inflamatorios crónico, colposcopia negativo para VPH y perfil hormonal normal (FSH, LH, P4, E2).
  • 255. PREGUNTA NO. 188 Tomando en cuenta el perfil hormonal de la paciente que intervención en primera instancia tomaría usted? RESPUESTA: a.- Iniciar clomifeno 5 mg cada 24 hrs durante 14 días. b.- Solicitar una espermatobioscopia. c.- Realiza USG de anexos para buscar causa obstructiva. d.- Indicar medroxiprogesterona 25mg y etinilestradiol 2mg im
  • 256. CASO CLINICO Femenino de 31 años de edad la acude a consulta debido a que presenta amenorrea secundaria, agrega que desea embarazarse, como antecedentes cuenta con menarca a 21 años con oligomenorrea, se observa con clasificación de Taner grado 3, refiere presencia de leucorrea frecuente ha recibido tratamiento, a la exploración observa Talla 1.47 mts, Peso 48 kg. Además cuello alado, torax en escudo.
  • 257. PREGUNTA No. 189 Cuál es la condición más frecuente de origen genético que presenta con los datos observados en este caso. RESPUESTA: a.- Sindrome de Kallman. b.- Sindrome de Turner. c.- Sindrome de Klinifelter. c.- Sindrome XXX.
  • 258. CASO CLINICO Se trata de paciente femenino de 21 años de edad la cual acude por falta de menstruación durante los últimos 6 meses, niega vida sexual activa, sus antecedentes GO son menarca 11 años, G: 0, a la exploración se encuentra caracteres sexuales normales para edad y sexo, se observa sobrepeso por IMC, niega flujo, refiere que en los 3 últimos años sus periodos menstruales han sido irregulares, niega cefalea u otros síntomas.
  • 259. PREGUNTA No. 190 Cuál de los siguientes diagnósticos es el más probable? RESPUESTA a.- Microadenoma PRL productor. b.- Adenoma GH productor. c.- Microadenoma ACTH productor. d.- MEN.
  • 260. CASO CLINICO Se trata de femenino de 21 años de edad la cual acude a consulta debido a que refiere mastalgia y sensación de plenitud mamaria, irritabilidad, inestabilidad emocional, refiere que desde los 17 años se presento más intensos los síntomas, agrega dismenorrea ocacional con rechazo al periodo menstrual, al interrogatorio usted observa irregularidades menstruales, la paciente presenta un IMC de 18.
  • 261. PREGUNTA No. 191 Cuál es la conducta más adecuada para este caso? RESPUESTA a.- La conducta mas adecuada es iniciar un esquema de contraceptivo oral. b.- Iniciar un inhibidor selectivo de la recaptura de Serotonina. c.- Indicar la calendarización de la sintomatología previa a la terapia. d.- Derivar a la paciente al servicio de psiquiatría.
  • 262. CASO CLINICO Femenino de 41 años de edad. Entre sus antecedentes mencionó el diagnóstico de tuberculosis intestinal tres años antes, siendo tratada durante 8 meses con tratamiento específico tetra-asociado, periodos menstruales regulares y un parto por cesárea a los 32 años. Se hospitalizo por cuadro de varios años de evolución, con frecuentes episodios de dolor abdominal difuso, distensión abdominal, flatulencia, nauseas, vómitos y constipación, mayor durante ciclos menstruales. El cuadro que se intensifico durante el último año, asociándose esporádicamente deposiciones con estrías de sangre viva y perdida de 4 a 5 Kg de peso.
  • 263. CASO CLINICO Al ingreso, paciente en regulares condiciones generales, presión arterial 120/80 mmHg, frecuencia cardiaca 88 latidos por minuto, temperatura 36,6 ºC. El abdomen se encontraba distendido, timpanizado a la percusión, chapoteo en flanco y fosa iliaca derecha y borborismos generalizados a la auscultación. Los laboratorios, rayos X de tórax, abdomen y ecografía abdominal de ingreso fueron normales, exceptuando la sangre oculta en heces que fue (+). Las revisiones ginecológica y proctológica no encontraron alteraciones.
  • 264. PREGUNTA No. 192 Cuál es el estadio en el que se encuentra la paciente? RESPUESTA a.- Estadio I (Mínimo) - 1-5 puntos. b.- Estadio II (Leve) - 6-15 puntos. c.- Estadio III (Moderada) - 16-40 puntos. d.- Estadio IV (Severa) - más 40 puntos.

Editor's Notes

  1. 01/04/13 DR. ANGEL MAURICIO SORIANO PEREZ NEXTEL 36246001 CURSO ENARM CMN SIGLO XXI PHARMED SOLUTIONS INSTITUTE